Med Surg Final Exam Fall 2020

¡Supera tus tareas y exámenes ahora con Quizwiz!

Vitamin D

5-75 ng/mL

Neutrophils

55-70%

Hypercalcemia

>10.5 mg/dL

A client has Crohns disease. What type of anemia is this client most at risk for developing? A. folic acid deficiency B. fanconis anemia C. Hemolytic anemia D. vitamin B12 anemia

A

A client in the sickle cell crisis is dehydrated and in the emergency department. The nurse plans to start an IV. Which fluid choice is best? A. 0.45% normal saline B. 0.9% normal saline C. Dextrose 50% (D50) D. lactated ringers solution

A

what is the normal measurement of QRS complex in ECG A. less than 0.12 seconds B. 0.10-0.16 seconds C. 0.12-0.20 seconds D. 0.16-0.24 seconds

A

what is the normal position of the ST segment in a ECG A. isoelectric B. elevated C. depressed D. biphasic

A

when heart failure develops, what is the initial compensatory mechanism of the heart that maintains cardiac output? A. sympathetic stimulation B. parasympathetic stimulation C. renin-angiotensin activation system (RAAS) D. myocardial hypertrophy

A

when the nurse assesses a patient with cardiovascular disease (CVD) there is difficulty auscultating the first heart sound S1. what is the nurses best action? A. ask the pt to lean forward or roll to his or her left side B. instruct the pt to take a deep breath and hold it C. auscultate with the bell instead of the diaphragm D. ask the unlicensed assistive personnel to complete a 12 lead ECG immediately

A

which category of cardiovascular drugs blocks sympathetic stimulation to the heart and decreases the heart rate A. beta blockers B. catecholamines C. steroids D. benzodiazepines

A

which pt is a candidate for proportionate palliative sedation A. pt is having refactory symptoms of distress that are not responding to treatments B. pt is seeking options and alternatives to passive euthanasia C. pt is extremely anxious that pain and suffering will not be adequately addressed D. pt is convinved that established pallative protocols will hasten death

A

The pts potassium level is 2.5 mEq/L. Which clinical findings does the nurse expect to see when assessing this patient? Select all that appy: A. General skeletal muscle weakness B. Moist crackles and tachypnea C. Lethargy D. Decreased urine output E. Weak hand grasps F. Weak, thready pulse

A. General skeletal muscle weakness C. Lethargy E. Weak hand grasps F. Weak, thready pulse

The emergency department (ED) nurse is caring for a patient who was brought in for significant alcohol intoxication and minor trauma to the wrist. What will serial hematocrits for this pt likely show. A. Hemoconcentration B. Normal and stable hematocrits C. Progressively lower hematocrits D. Decreasing osmolarity

A. Hemoconcentration

A pt has had diarrhea for the past 2 days. Which acid base normalities would the nurse monitor for? Select all that apply: A. Over-elimination of bicarbonate B. Respiratory alkalosis C. Metabolic acidosis D. Under-elimination of hydrogen ions E. Over production of hydrogen ions F. Elevated potassium

A. Over-elimination of bicarbonate C. Metabolic acidosis

By which mechanisms does parathyroid hormone PTH increase serum calcium levels? Select all that apply: A. Releasing free calcium from the bones B. Increasing calcium excretion in the urine C. Stimulating kidney reabsorption of calcium D. Causing vitamin D activation E. Increasing calcium absorption in the GI tract F. Pulling calcium out of the cells

A. Releasing free calcium from the bones C. Stimulating kidney reabsorption of calcium D. Causing vitamin D activation E. Increasing calcium absorption in the GI tract

The charge nurse is reviewing IV therapy orders. What information must be included in each order? A. Specific type of solution B. Rate of administration C. Specific drug dose to be added to the solution D. Method for diluting drugs for the solution E. Specific type of administration equipment

A. Specific type of solution B. Rate of administration C. Specific drug dose to be added to the solution

A nurse prepares to administer prescribed regular and NPH insulin. Place the nurse's actions in the correct order to administer these medications. 1. Inspect bottles for expiration dates. 2. Gently roll the bottle of NPH between the hands. 3. Wash your hands. 4. Inject air into the regular insulin. 5. Withdraw the NPH insulin. 6. Withdraw the regular insulin. 7. Inject air into the NPH bottle. 8. Clean rubber stoppers with an alcohol swab.

ANS: 3, 1, 2, 8, 7, 4, 6, 5 After washing hands, it is important to inspect the bottles and then to roll the NPH to mix the insulin. Rubber stoppers should be cleaned with alcohol after rolling the NPH and before sticking a needle into either bottle. It is important to inject air into the NPH bottle before placing the needle in a regular insulin bottle to avoid mixing of regular and NPH insulin. The shorter-acting insulin is always drawn up first.

A preoperative nurse assesses a client who has type 1 diabetes mellitus prior to a surgical procedure. The client's blood glucose level is 160 mg/dL. Which action should the nurse take? a. Document the finding in the client's chart. b. Administer a bolus of regular insulin IV. c. Call the surgeon to cancel the procedure. d. Draw blood gases to assess the metabolic state.

ANS: A Clients who have type 1 diabetes and are having surgery have been found to have fewer complications, lower rates of infection, and better wound healing if blood glucose levels are maintained at between 140 and 180 mg/dL throughout the perioperative period. The nurse should document the finding and proceed with other operative care. The need for a bolus of insulin, canceling the procedure, or drawing arterial blood gases is not required.

After teaching a client who is recovering from pancreas transplantation, the nurse assesses the client's understanding. Which statement made by the client indicates a need for additional education? a. "If I develop an infection, I should stop taking my corticosteroid." b. "If I have pain over the transplant site, I will call the surgeon immediately." c. "I should avoid people who are ill or who have an infection." d. "I should take my cyclosporine exactly the way I was taught."

ANS: A Immunosuppressive agents should not be stopped without the consultation of the transplantation physician, even if an infection is present. Stopping immunosuppressive therapy endangers the transplanted organ. The other statements are correct. Pain over the graft site may indicate rejection. Anti-rejection drugs cause immunosuppression, and the client should avoid crowds and people who are ill. Changing the routine of anti-rejection medications may cause them to not work optimally.

An emergency department nurse assesses a client with ketoacidosis. Which clinical manifestation should the nurse correlate with this condition? a. Increased rate and depth of respiration b. Extremity tremors followed by seizure activity c. Oral temperature of 102° F (38.9° C) d. Severe orthostatic hypotension

ANS: A Ketoacidosis decreases the pH of the blood, stimulating the respiratory control areas of the brain to buffer the effects of increasing acidosis. The rate and depth of respiration are increased (Kussmaul respirations) in an attempt to excrete more acids by exhalation. Tremors, elevated temperature, and orthostatic hypotension are not associated with ketoacidosis.

A nurse cares for a client who has a family history of diabetes mellitus. The client states, "My father has type 1 diabetes mellitus. Will I develop this disease as well?" How should the nurse respond? a. "Your risk of diabetes is higher than the general population, but it may not occur." b. "No genetic risk is associated with the development of type 1 diabetes mellitus." c. "The risk for becoming a diabetic is 50% because of how it is inherited." d. "Female children do not inherit diabetes mellitus, but male children will."

ANS: A Risk for type 1 diabetes is determined by inheritance of genes coding for HLA-DR and HLA-DQ tissue types. Clients who have one parent with type 1 diabetes are at increased risk for its development. Diabetes (type 1) seems to require interaction between inherited risk and environmental factors, so not everyone with these genes develops diabetes. The other statements are not accurate.

After teaching a client with diabetes mellitus to inject insulin, the nurse assesses the client's understanding. Which statement made by the client indicates a need for additional teaching? a. "The lower abdomen is the best location because it is closest to the pancreas." b. "I can reach my thigh the best, so I will use the different areas of my thighs." c. "By rotating the sites in one area, my chance of having a reaction is decreased." d. "Changing injection sites from the thigh to the arm will change absorption rates."

ANS: A The abdominal site has the fastest rate of absorption because of blood vessels in the area, not because of its proximity to the pancreas. The other statements are accurate assessments of insulin administration.

A nurse assesses a client with diabetes mellitus and notes the client only responds to a sternal rub by moaning, has capillary blood glucose of 33 g/dL, and has an intravenous line that is infiltrated with 0.45% normal saline. Which action should the nurse take first? a. Administer 1 mg of intramuscular glucagon. b. Encourage the client to drink orange juice. c. Insert a new intravenous access line. d. Administer 25 mL dextrose 50% (D50) IV push.

ANS: A The client's blood glucose level is dangerously low. The nurse needs to administer glucagon IM immediately to increase the client's blood glucose level. The nurse should insert a new IV after administering the glucagon and can use the new IV site for future doses of D50 if the client's blood glucose level does not rise. Once the client is awake, orange juice may be administered orally along with a form of protein such as a peanut butter.

A nurse assesses a client who has diabetes mellitus and notes the client is awake and alert, but shaky, diaphoretic, and weak. Five minutes after administering a half-cup of orange juice, the client's clinical manifestations have not changed. Which action should the nurse take next? a. Administer another half-cup of orange juice. b. Administer a half-ampule of dextrose 50% intravenously. c. Administer 10 units of regular insulin subcutaneously. d. Administer 1 mg of glucagon intramuscularly.

ANS: A This client is experiencing mild hypoglycemia. For mild hypoglycemic manifestations, the nurse should administer oral glucose in the form of orange juice. If the symptoms do not resolve immediately, the treatment should be repeated. The client does not need intravenous dextrose, insulin, or glucagon.

A nurse is assessing an older client for the presence of infection. The client's temperature is 97.6° F (36.4° C). What response by the nurse is best? a. Assess the client for more specific signs. b. Conclude that an infection is not present. c. Document findings and continue to monitor. d. Request that the provider order blood cultures.

ANS: A Because older adults have decreased immune function, including reduced neutrophil function, fever may not be present during an episode of infection. The nurse should assess the client for specific signs of infection. Documentation needs to occur, but a more thorough assessment comes first. Blood cultures may or may not be needed depending on the results of further assessment.

The nurse caring for oncology clients knows that which form of metastasis is the most common? a. Bloodborne b. Direct invasion c. Lymphatic spread d. Via bone marrow

ANS: A Bloodborne metastasis is the most common way for cancer to metastasize. Direct invasion and lymphatic spread are other methods. Bone marrow is not a medium in which cancer spreads, although cancer can occur in the bone marrow.

A client has been on dialysis for many years and now is receiving a kidney transplant. The client experiences hyperacute rejection. What treatment does the nurse prepare to facilitate? a. Dialysis b. High-dose steroid administration c. Monoclonal antibody therapy d. Plasmapheresis

ANS: A Hyperacute rejection starts within minutes of transplantation and nothing will stop the process. The organ is removed. If the client survives, he or she will have to return to dialysis treatment. Steroids, monoclonal antibodies, and plasmapheresis are ineffective against this type of rejection.

A clinic nurse is working with an older client. What assessment is most important for preventing infections in this client? a. Assessing vaccination records for booster shot needs b. Encouraging the client to eat a nutritious diet c. Instructing the client to wash minor wounds carefully d. Teaching hand hygiene to prevent the spread of microbes

ANS: A Older adults may have insufficient antibodies that have already been produced against microbes to which they have been exposed. Therefore, older adults need booster shots for many vaccinations they received as younger people. A nutritious diet, proper wound care, and hand hygiene are relevant for all populations.

A client is taking prednisone to prevent transplant rejection. What instruction by the nurse is most important? a. "Avoid large crowds and people who are ill." b. "Check over-the-counter meds for acetaminophen." c. "Take this medicine exactly as prescribed." d. "You have a higher risk of developing cancer."

ANS: A Prednisone, like all steroids, decreases immune function. The client should be advised to avoid large crowds and people who are ill. Prednisone does not contain acetaminophen. All clients should be taught to take medications exactly as prescribed. A higher risk for cancer is seen with drugs from the calcineurin inhibitor category, such as tacrolimus (Prograf).

A nurse provides diabetic education at a public health fair. Which disorders should the nurse include as complications of diabetes mellitus? (Select all that apply.) a. Stroke b. Kidney failure c. Blindness d. Respiratory failure e. Cirrhosis

ANS: A, B, C Complications of diabetes mellitus are caused by macrovascular and microvascular changes. Macrovascular complications include coronary artery disease, cerebrovascular disease, and peripheral vascular disease. Microvascular complications include nephropathy, retinopathy, and neuropathy. Respiratory failure and cirrhosis are not complications of diabetes mellitus.

The nurse working with oncology clients understands that interacting factors affect cancer development. Which factors does this include? (Select all that apply.) a. Exposure to carcinogens b. Genetic predisposition c. Immune function d. Normal doubling time e. State of euploidy

ANS: A, B, C The three interacting factors needed for cancer development are exposure to carcinogens, genetic predisposition, and immune function.

For a person to be immunocompetent, which processes need to be functional and interact appropriately with each other? (Select all that apply.) a. Antibody-mediated immunity b. Cell-mediated immunity c. Inflammation d. Red blood cells e. White blood cells

ANS: A, B, C The three processes that need to be functional and interact with each other for a person to be immunocompetent are antibody-mediated immunity, cell-mediated immunity, and inflammation. Red and white blood cells are not processes.

A nurse collaborates with the interdisciplinary team to develop a plan of care for a client who is newly diagnosed with diabetes mellitus. Which team members should the nurse include in this interdisciplinary team meeting? (Select all that apply.) a. Registered dietitian b. Clinical pharmacist c. Occupational therapist d. Health care provider e. Speech-language pathologist

ANS: A, B, D When planning care for a client newly diagnosed with diabetes mellitus, the nurse should collaborate with a registered dietitian, clinical pharmacist, and health care provider. The focus of treatment for a newly diagnosed client would be nutrition, medication therapy, and education. The nurse could also consult with a diabetic educator. There is no need for occupational therapy or speech therapy at this time.

Which are steps in the process of making an antigen-specific antibody? (Select all that apply.) a. Antibody-antigen binding b. Invasion c. Opsonization d. Recognition e. Sensitization

ANS: A, B, D, E The seven steps in the process of making antigen-specific antibodies are: exposure/invasion, antigen recognition, sensitization, antibody production and release, antigen-antibody binding, antibody binding actions, and sustained immunity. Opsonization is the adherence of an antibody to the antigen, marking it for destruction.

A nurse teaches a client with diabetes mellitus about foot care. Which statements should the nurse include in this client's teaching? (Select all that apply.) a. "Do not walk around barefoot." b. "Soak your feet in a tub each evening." c. "Trim toenails straight across with a nail clipper." d. "Treat any blisters or sores with Epsom salts." e. "Wash your feet every other day."

ANS: A, C Clients who have diabetes mellitus are at high risk for wounds on the feet secondary to peripheral neuropathy and poor arterial circulation. The client should be instructed to not walk around barefoot or wear sandals with open toes. These actions place the client at higher risk for skin breakdown of the feet. The client should be instructed to trim toenails straight across with a nail clipper. Feet should be washed daily with lukewarm water and soap, but feet should not be soaked in the tub. The client should contact the provider immediately if blisters or sores appear and should not use home remedies to treat these wounds.

The nursing student learning about cancer development remembers characteristics of normal cells. Which characteristics does this include? (Select all that apply.) a. Differentiated function b. Large nucleus-to-cytoplasm ratio c. Loose adherence d. Nonmigratory e. Specific morphology

ANS: A, D, E Normal cells have the characteristics of differentiated function, nonmigratory, specific morphology, a smaller nucleus-to-cytoplasm ratio, tight adherence, and orderly and well-regulated growth.

After teaching a client with type 2 diabetes mellitus, the nurse assesses the client's understanding. Which statement made by the client indicates a need for additional teaching? a. "I need to have an annual appointment even if my glucose levels are in good control." b. "Since my diabetes is controlled with diet and exercise, I must be seen only if I am sick." c. "I can still develop complications even though I do not have to take insulin at this time." d. "If I have surgery or get very ill, I may have to receive insulin injections for a short time."

ANS: B Clients with diabetes need to be seen at least annually to monitor for long-term complications, including visual changes, microalbuminuria, and lipid analysis. The client may develop complications and may need insulin in the future.

A nurse reviews the medication list of a client with a 20-year history of diabetes mellitus. The client holds up the bottle of prescribed duloxetine (Cymbalta) and states, "My cousin has depression and is taking this drug. Do you think I'm depressed?" How should the nurse respond? a. "Many people with long-term diabetes become depressed after a while." b. "It's for peripheral neuropathy. Do you have burning pain in your feet or hands?" c. "This antidepressant also has anti-inflammatory properties for diabetic pain." d. "No. Many medications can be used for several different disorders."

ANS: B Damage along nerves causes peripheral neuropathy and leads to burning pain along the nerves. Many drugs, including duloxetine (Cymbalta), can be used to treat peripheral neuropathy. The nurse should assess the client for this condition and then should provide an explanation of why this drug is being used. This medication, although it is used for depression, is not being used for that reason in this case. Duloxetine does not have anti-inflammatory properties. Telling the client that many medications are used for different disorders does not provide the client with enough information to be useful.

A nurse cares for a client with diabetes mellitus who asks, "Why do I need to administer more than one injection of insulin each day?" How should the nurse respond? a. "You need to start with multiple injections until you become more proficient at self-injection." b. "A single dose of insulin each day would not match your blood insulin levels and your food intake patterns." c. "A regimen of a single dose of insulin injected each day would require that you eat fewer carbohydrates." d. "A single dose of insulin would be too large to be absorbed, predictably putting you at risk for insulin shock."

ANS: B Even when a single injection of insulin contains a combined dose of different-acting insulin types, the timing of the actions and the timing of food intake may not match well enough to prevent wide variations in blood glucose levels. One dose of insulin would not be appropriate even if the client decreased carbohydrate intake. Additional injections are not required to allow the client practice with injections, nor will one dose increase the client's risk of insulin shock.

A nurse assesses a client with diabetes mellitus. Which clinical manifestation should alert the nurse to decreased kidney function in this client? a. Urine specific gravity of 1.033 b. Presence of protein in the urine c. Elevated capillary blood glucose level d. Presence of ketone bodies in the urine

ANS: B Renal dysfunction often occurs in the client with diabetes. Proteinuria is a result of renal dysfunction. Specific gravity is elevated with dehydration. Elevated capillary blood glucose levels and ketones in the urine are consistent with diabetes mellitus but are not specific to renal function.

A nurse develops a dietary plan for a client with diabetes mellitus and new-onset microalbuminuria. Which component of the client's diet should the nurse decrease? a. Carbohydrates b. Proteins c. Fats d. Total calories

ANS: B Restriction of dietary protein to 0.8 g/kg of body weight per day is recommended for clients with microalbuminuria to delay progression to renal failure. The client's diet does not need to be decreased in carbohydrates, fats, or total calories.

A nurse teaches a client about self-monitoring of blood glucose levels. Which statement should the nurse include in this client's teaching to prevent bloodborne infections? a. "Wash your hands after completing each test." b. "Do not share your monitoring equipment." c. "Blot excess blood from the strip with a cotton ball." d. "Use gloves when monitoring your blood glucose."

ANS: B Small particles of blood can adhere to the monitoring device, and infection can be transported from one user to another. Hepatitis B in particular can survive in a dried state for about a week. The client should be taught to avoid sharing any equipment, including the lancet holder. The client should be taught to wash his or her hands before testing. The client would not need to blot excess blood away from the strip or wear gloves.

A nurse reviews laboratory results for a client with diabetes mellitus who is prescribed an intensified insulin regimen: • Fasting blood glucose: 75 mg/dL • Postprandial blood glucose: 200 mg/dL • Hemoglobin A1c level: 5.5% How should the nurse interpret these laboratory findings? a. Increased risk for developing ketoacidosis b. Good control of blood glucose c. Increased risk for developing hyperglycemia d. Signs of insulin resistance

ANS: B The client is maintaining blood glucose levels within the defined ranges for goals in an intensified regimen. Because the client's glycemic control is good, he or she is not at higher risk for ketoacidosis or hyperglycemia and is not showing signs of insulin resistance.

A nurse cares for a client who is prescribed pioglitazone (Actos). After 6 months of therapy, the client reports that his urine has become darker since starting the medication. Which action should the nurse take? a. Assess for pain or burning with urination. b. Review the client's liver function study results. c. Instruct the client to increase water intake. d. Test a sample of urine for occult blood.

ANS: B Thiazolidinediones (including pioglitazone) can affect liver function; liver function should be assessed at the start of therapy and at regular intervals while the client continues to take these drugs. Dark urine is one indicator of liver impairment because bilirubin is increased in the blood and is excreted in the urine. The nurse should check the client's most recent liver function studies. The nurse does not need to assess for pain or burning with urination and does not need to check the urine for occult blood. The client does not need to be told to increase water intake.

A nurse assesses a client who has diabetes mellitus. Which arterial blood gas values should the nurse identify as potential ketoacidosis in this client? a. pH 7.38, HCO3- 22 mEq/L, PCO2 38 mm Hg, PO2 98 mm Hg b. pH 7.28, HCO3- 18 mEq/L, PCO2 28 mm Hg, PO2 98 mm Hg c. pH 7.48, HCO3- 28 mEq/L, PCO2 38 mm Hg, PO2 98 mm Hg d. pH 7.32, HCO3- 22 mEq/L, PCO2 58 mm Hg, PO2 88 mm Hg

ANS: B When the lungs can no longer offset acidosis, the pH decreases to below normal. A client who has diabetic ketoacidosis would present with arterial blood gas values that show primary metabolic acidosis with decreased bicarbonate levels and a compensatory respiratory alkalosis with decreased carbon dioxide levels.

A nurse teaches a client with type 2 diabetes mellitus who is prescribed glipizide (Glucotrol). Which statement should the nurse include in this client's teaching? a. "Change positions slowly when you get out of bed." b. "Avoid taking nonsteroidal anti-inflammatory drugs (NSAIDs)." c. "If you miss a dose of this drug, you can double the next dose." d. "Discontinue the medication if you develop a urinary infection."

ANS: B NSAIDs potentiate the hypoglycemic effects of sulfonylurea agents. Glipizide is a sulfonylurea. The other statements are not applicable to glipizide.

A student nurse is learning about the types of different cells involved in the inflammatory response. Which principles does the student learn? (Select all that apply.) a. Basophils are only involved in the general inflammatory process. b. Eosinophils increase during allergic reactions and parasitic invasion. c. Macrophages can participate in many episodes of phagocytosis. d. Monocytes turn into macrophages after they enter body tissues. e. Neutrophils can only take part in one episode of phagocytosis.

ANS: B, C, D, E Eosinophils do increase during allergic and parasitic invasion. Macrophages participate in many episodes of phagocytosis. Monocytes turn into macrophages after they enter body tissues. Neutrophils only take part in one episode of phagocytosis. Basophils are involved in both the general inflammatory response and allergic or hypersensitivity responses.

After teaching a client who is newly diagnosed with type 2 diabetes mellitus, the nurse assesses the client's understanding. Which statement made by the client indicates a need for additional teaching? a. "I should increase my intake of vegetables with higher amounts of dietary fiber." b. "My intake of saturated fats should be no more than 10% of my total calorie intake." c. "I should decrease my intake of protein and eliminate carbohydrates from my diet." d. "My intake of water is not restricted by my treatment plan or medication regimen."

ANS: C The client should not completely eliminate carbohydrates from the diet, and should reduce protein if microalbuminuria is present. The client should increase dietary intake of complex carbohydrates, including vegetables, and decrease intake of fat. Water does not need to be restricted unless kidney failure is present.

A nurse assesses a client with diabetes mellitus 3 hours after a surgical procedure and notes the client's breath has a "fruity" odor. Which action should the nurse take? a. Encourage the client to use an incentive spirometer. b. Increase the client's intravenous fluid flow rate. c. Consult the provider to test for ketoacidosis. d. Perform meticulous pulmonary hygiene care.

ANS: C The stress of surgery increases the action of counterregulatory hormones and suppresses the action of insulin, predisposing the client to ketoacidosis and metabolic acidosis. One manifestation of ketoacidosis is a "fruity" odor to the breath. Documentation should occur after all assessments have been completed. Using an incentive spirometer, increasing IV fluids, and performing pulmonary hygiene will not address this client's problem.

A nurse is assessing a client for acute rejection of a kidney transplant. What assessment finding requires the most rapid communication with the provider? a. Blood urea nitrogen (BUN) of 18 mg/dL b. Cloudy, foul-smelling urine c. Creatinine of 3.9 mg/dL d. Urine output of 340 mL/8 hr

ANS: C A creatinine of 3.9 mg/dL is high, indicating possible dysfunction of the kidney. This is a possible sign of rejection. The BUN is normal, as is the urine output. Cloudy, foul-smelling urine would probably indicate a urinary tract infection.

When teaching a client recently diagnosed with type 1 diabetes mellitus, the client states, "I will never be able to stick myself with a needle." How should the nurse respond? a. "I can give your injections to you while you are here in the hospital." b. "Everyone gets used to giving themselves injections. It really does not hurt." c. "Your disease will not be managed properly if you refuse to administer the shots." d. "Tell me what it is about the injections that are concerning you."`

ANS: D Devote as much teaching time as possible to insulin injection and blood glucose monitoring. Clients with newly diagnosed diabetes are often fearful of giving themselves injections. If the client is worried about giving the injections, it is best to try to find out what specifically is causing the concern, so it can be addressed. Giving the injections for the client does not promote self-care ability. Telling the client that others give themselves injections may cause the client to feel bad. Stating that you don't know another way to manage the disease is dismissive of the client's concerns.

A nurse reviews the medication list of a client recovering from a computed tomography (CT) scan with IV contrast to rule out small bowel obstruction. Which medication should alert the nurse to contact the provider and withhold the prescribed dose? a. Pioglitazone (Actos) b. Glimepiride (Amaryl) c. Glipizide (Glucotrol) d. Metformin (Glucophage)

ANS: D Glucophage should not be administered when the kidneys are attempting to excrete IV contrast from the body. This combination would place the client at high risk for kidney failure. The nurse should hold the metformin dose and contact the provider. The other medications are safe to administer after receiving IV contrast.

A nurse teaches a client who is prescribed an insulin pump. Which statement should the nurse include in this client's discharge education? a. "Test your urine daily for ketones." b. "Use only buffered insulin in your pump." c. "Store the insulin in the freezer until you need it." d. "Change the needle every 3 days."

ANS: D Having the same needle remain in place through the skin for longer than 3 days drastically increases the risk for infection in or through the delivery system. Having an insulin pump does not require the client to test for ketones in the urine. Insulin should not be frozen. Insulin is not buffered.

A nurse reviews laboratory results for a client with diabetes mellitus who presents with polyuria, lethargy, and a blood glucose of 560 mg/dL. Which laboratory result should the nurse correlate with the client's polyuria? a. Serum sodium: 163 mEq/L b. Serum creatinine: 1.6 mg/dL c. Presence of urine ketone bodies d. Serum osmolarity: 375 mOsm/kg

ANS: D Hyperglycemia causes hyperosmolarity of extracellular fluid. This leads to polyuria from an osmotic diuresis. The client's serum osmolarity is high. The client's sodium would be expected to be high owing to dehydration. Serum creatinine and urine ketone bodies are not related to the polyuria.

A nurse reviews the laboratory results of a client who is receiving intravenous insulin. Which should alert the nurse to intervene immediately? a. Serum chloride level of 98 mmol/L b. Serum calcium level of 8.8 mg/dL c. Serum sodium level of 132 mmol/L d. Serum potassium level of 2.5 mmol/L

ANS: D Insulin activates the sodium-potassium ATPase pump, increasing the movement of potassium from the extracellular fluid into the intracellular fluid, resulting in hypokalemia. In hyperglycemia, hypokalemia can also result from excessive urine loss of potassium. The chloride level is normal. The calcium and sodium levels are slightly low, but this would not be related to hyperglycemia and insulin administration.

A nurse teaches a client with type 1 diabetes mellitus. Which statement should the nurse include in this client's teaching to decrease the client's insulin needs? a. "Limit your fluid intake to 2 liters a day." b. "Animal organ meat is high in insulin." c. "Limit your carbohydrate intake to 80 grams a day." d. "Walk at a moderate pace for 1 mile daily."

ANS: D Moderate exercise such as walking helps regulate blood glucose levels on a daily basis and results in lowered insulin requirements for clients with type 1 diabetes mellitus. Restricting fluids and eating organ meats will not reduce insulin needs. People with diabetes need at least 130 grams of carbohydrates each day.

After teaching a client with type 2 diabetes mellitus who is prescribed nateglinide (Starlix), the nurse assesses the client's understanding. Which statement made by the client indicates a correct understanding of the prescribed therapy? a. "I'll take this medicine during each of my meals." b. "I must take this medicine in the morning when I wake." c. "I will take this medicine before I go to bed." d. "I will take this medicine immediately before I eat."

ANS: D Nateglinide is an insulin secretagogue that is designed to increase meal-related insulin secretion. It should be taken immediately before each meal. The medication should not be taken without eating as it will decrease the client's blood glucose levels. The medication should be taken before meals instead of during meals.

A nurse assesses a client with diabetes mellitus who self-administers subcutaneous insulin. The nurse notes a spongy, swelling area at the site the client uses most frequently for insulin injection. Which action should the nurse take? a. Apply ice to the site to reduce inflammation. b. Consult the provider for a new administration route. c. Assess the client for other signs of cellulitis. d. Instruct the client to rotate sites for insulin injection.

ANS: D The client's tissue has been damaged from continuous use of the same site. The client should be educated to rotate sites. The damaged tissue is not caused by cellulitis or any type infection, and applying ice may cause more damage to the tissue. Insulin can only be administered subcutaneously and intravenously. It would not be appropriate or practical to change the administration route.

A nurse cares for a client experiencing diabetic ketoacidosis who presents with Kussmaul respirations. Which action should the nurse take? a. Administration of oxygen via face mask b. Intravenous administration of 10% glucose c. Implementation of seizure precautions d. Administration of intravenous insulin

ANS: D The rapid, deep respiratory efforts of Kussmaul respirations are the body's attempt to reduce the acids produced by using fat rather than glucose for fuel. Only the administration of insulin will reduce this type of respiration by assisting glucose to move into cells and to be used for fuel instead of fat. The client who is in ketoacidosis may not experience any respiratory impairment and therefore does not need additional oxygen. Giving the client glucose would be contraindicated. The client does not require seizure precautions.

A nurse cares for a client who has type 1 diabetes mellitus. The client asks, "Is it okay for me to have an occasional glass of wine?" How should the nurse respond? a. "Drinking any wine or alcohol will increase your insulin requirements." b. "Because of poor kidney function, people with diabetes should avoid alcohol." c. "You should not drink alcohol because it will make you hungry and overeat." d. "One glass of wine is okay with a meal and is counted as two fat exchanges."

ANS: D Under normal circumstances, blood glucose levels will not be affected by moderate use of alcohol when diabetes is well controlled. Because alcohol can induce hypoglycemia, it should be ingested with or shortly after a meal. One alcoholic beverage is substituted for two fat exchanges when caloric intake is calculated. Kidney function is not impacted by alcohol intake. Alcohol is not associated with increased hunger or overeating.

A student nurse asks the nursing instructor what "apoptosis" means. What response by the instructor is best? a. Growth by cells enlarging b. Having the normal number of chromosomes c. Inhibition of cell growth d. Programmed cell death

ANS: D Apoptosis is programmed cell death. With this characteristic, organs and tissues function with cells that are at their peak of performance. Growth by cells enlarging is hyperplasia. Having the normal number of chromosomes is euploidy. Inhibition of cell growth is contact inhibition.

Which statement about carcinogenesis is accurate? a. An initiated cell will always become clinical cancer. b. Cancer becomes a health problem once it is 1 cm in size. c. Normal hormones and proteins do not promote cancer growth. d. Tumor cells need to develop their own blood supply.

ANS: D Tumors need to develop their own blood supply through a process called angiogenesis. An initiated cell needs a promoter to continue its malignant path. Normal hormones and proteins in the body can act as promoters. A 1-cm tumor is a detectable size, but other events have to occur for it to become a health problem.

A client receiving a blood transfusion develops anxiety and low back pain. After stopping the transfusion, what action by the nurse is most important? A. documenting the events in the clients medical record B. double-checking the client and blood product identification C. placing the client on strict bedrest until the pain subsides D. reviewing the clients medical record for known allergies

B

A nurse assesses a wife who is caring for her husband. She has a braden scale score of 9. Which question should the nurse include in this assessment? A. do you have a bedpan at home? B. how are you coping with providing this care? C. what are you doing to prevent pediculosis? D. are you sharing a bed with your husband?

B

A nurse cares for a client with burn injuries from a house fire. The client is not consistently oriented and reports a headache. Which action should the nurse take? A. increase the clients oxygen and obtain blood gases B. draw blood for a carboxyhemoglobin level C. increase the clients intravenous fluid rate D. perform a thorough mini-mental state exam

B

A nurse evaluates the following data in a clients chart: Admission note-78 year old male with a past medical history of a-fib is admitted with chronic leg wound. Prescriptions-warfarin sodium (coumadin), sotalol (Betapace). Wound Care-vacuum assisted wound closure (VAC) treatment to leg wound Based on this information, which action should the nurse take first? A. Assess the clients vital signs and initiate continuous telemetry monitoring B. contact the provider and express concerns related to the wound treatment prescribed C. consult the wound care nurse to apply the VAC device D. obtain a prescription for a low-fat, high-protein diet with vitamin supplements

B

A nurse in a hematology clinic is working with four clients who have polycythemia vera. Which client should the nurse see first? A. client with a blood pressure of 180/98 mmHg B. Client who reports shortness of breath C. client who reports calf tenderness and swelling D. Client with a swollen and painful left great toe

B

A nurse is preparing to administer a blood transfusion. What action is most important? A. correctly identifying client using two identifiers B. ensuring informed consent is obtained if required C. hanging the blood product with ringers lactate D. staying with the client for the entire transfusion

B

A nurse performs a skin screening for a client who has numerous skin lesions. Which lesion does the nurse evaluate first? A. beige freckles on the backs of both hands B. Irregular blue mole with white specks on the lower leg C. large cluster of pustules in the right axilla D. thick, reddened papules covered by white scales

B

A nurse receives new prescriptions for a client with severe burn injuries who is receiving fluid resuscitation per the parkland formula. The clients urine output continues to range from 0.2 to 0.25 mL/kg/hr. Which prescription should the nurse question? A. increase intravenous fluids by 100 mL/hr B. Administer furosemide (Lasix) 40 mg IV push C. continue to monitor urine output hourly D.. Draw blood for serum electrolytes STAT

B

A nurse suspects a client has serum sickness. Wat laboratory result would the nurse correlate with this condition? A. blood urea nitrogen: 12 mg/dL B. creatinine: 3.2 mg/dL C. hemoglobin: 8.2 mg/dL D. White blood cell count: 12,000/mm3

B

A pt with suspected TB is admitted to the hospital. Along with a private room, which nursing intervention is appropriate RELATED TO isolation procedures? a. airborne and contact isolation for sputum only b. strict airborne precautions and use of specially fitted respirator face masks c. airborne isolation with surgical masks until diagnosis is confirmed d. only standard precautions are necessary until the diagnosis is confirmed

B

a nurse is in charge of the coronary ICU what client should the nurse see first? A. client on a nitroglycerin infusion after 5 mcg/min not titrated in the last 4 hours B. client who is 1 day post coronary artery bypass graft, blood pressure 180/100 mm Hg C. client who is 1 day post percutaneous coronary intervention, going home this morning D. client who is 2 days post coronary artery bypass graft, became dizzy this morning while walking

B

a nurse is preparing to admit a client on mechanical ventilation from the ED what action by the nurse takes priority? A. assessing that the ventilator settings are correct B. ensuring there is a bag valve mask in the room C. obtaining personal protective equipment D. planning to suction the client upon arrival to the room

B

a nurse is teaching a client about warfarin (coumadin). what assessment finding by the nurse indicates a possible barrier to self-management? A. poor visual acuity B. strict vegetarian C. refusal to stop smoking D. wants weight loss surgery

B

a nurse is triaging clients in the ED which client should the nurse prioritize to receive care first? A. 22 year old with painful and swollen wrist B. 45 year old reporting chest pain and diaphoresis C. 60 year old reporting difficulty swallowing and nausea D. 81 year old with respiratory rate of 28 breaths/min and temp of 101F

B

a student is caring for a client who suffered massive blood loss after trauma. how does the student correlate the blood loss with the clients mean arterial pressure (MAP)? A. it causes vasoconstriction and increased MAP B. lower blood volume lowers MAP C. there is no direct correlation to MAP D. it raises cardiac output and MAP

B

a student nurse is preparing to administer enoxaparin (lovenox) to a client. what action by the student requires immediate intervention by the supervising nurse? A. assessing the client platelet count B. choosing an 18 gauge, 2 inch needle C. not aspirating prior to injection D. swabbing the injection site with alcohol

B

a trauma client with multiple open wounds is brought to the emergency department in cardiac arrest. which action should the nurse take prior to providing advanced cardiac life support? A. contact the on call ortho surgeon B. don PPE C. notify the rapid response team D. obtain a complete history from the paramedic

B

after a hospitals ED has efficiently triaged, treated and transferred clients from a community disaster to appropriate units, the hospital incident command officer wants to stand down from the emergency plan. which question should the nursing supervisor ask at this time? A. are you sure no more victims are coming into the ED B. do all areas of the hospital have the supplies and personnel they need? C. have all ED staff had the chance to eat and rest recently? D. does the chief medical officer agree this disaster is under control?

B

in a hypovolemic pt, stretch receptors in the blood vessels sense a reduced volume or pressure and send fewer impulses to central nervous system. as a result, which signs/symptoms does the nurse expect to observe in the pt A. reddish mottling to skin and a blood pressure elevation B. cool, pale skin and tachycardia C. warm, flushed skin, with low BP D. pale pink skin with bradycardia

B

on a hot humid day, an emergency deparment nurse is caring for a client who is confused and has these vitals: temp 104.1 F (40.1 C), pulse 132 bpm, resp. 26 breaths/min, BP 106/66 mmHg. which action should the nurse take? A. encourage the client to drink cool water or sports drinks B. start an IV line and infuse 0.9% saline solution C. administer acetaminophen (tylenol) 650 mg orally D. encourage rest and re-assess in 15 mins

B

the emergency department team is performing cardiopulmonary resuscitation on a client when the clients spouse arrives at the emergency department. what action should the nurse take first? A. request that the clients spouse sits in the waiting room B. ask the spouse if he wishes to be present during the resuscitation C. suggest that the spouse begin to pray for the client D> refer the clients spouse to the hospitals crisis team

B

the health care provider orders orthostatic vital signs on a pt who experienced dizziness and feeling lightheaded. what is the nurses first action A. instruct the pt to change position to sitting or standing B. measure the BP when the pt is supine C. place the pt in a supine positing for at least 3 mins D. wait for 1 min before auscultating BP and counting the radial pulse

B

the home health nurse is evaluating a pt being treated for heart failure. which statement by the pt is the best indicator of hope and well-being as a desired psychological outcome? A. I'm taking the medication and following the drs orders B. I'm looking forward to dancing with my wife on our wedding anniversary C. I'm planning to go on a long trip; I'll never go back to the hospital again D. I want to thank you for all that you have done. I know you did your best.

B

the nurse is analyzing the ECG rhythm strips for assigned pts. what is the nurses first action? A. analyze the P waves B. determine the heart rate C. measure the QRS duration D. measure the PR interval

B

the nurse is assessing a pt with CVD what is the priority med-surg concept for this pt A. fluid and electrolyte balance B. perfusion C. gas exchange D. acid base balance

B

the nurse is assessing a pts ECG rhythm strip and checking the regularity of the atrial rhythm. what is the correct technique? A. place one caliper point on a QRS complex; place the other point on the percise spot on the next QRS complex B. place one caliper point on a P wave; place the other point on the percise spot on the next P wave C. place one caliper point at the beginning of the P wave; place the other point at the end of the P-R segment D. place one caliper point at the beginning of the QRS complex; place the other point where the S-T segment begins

B

the nurse is caring for a 92 year old post op pt who has a DNR order. when the nurse assesses the pt he is diaphoretic and hyperalert and reports mild left anterior chest pain with SOB. What should the nurse do first? A. sit with the pt, talk calmly and be gently present B. administer o2 and alert the rapid responce team C. notify the person who has durable power of attorney for health D. monitor for cardiac or respiratory arrest and call the family

B

the nurse is counseling a group of women about cholesterol-lowering drugs. which drug will decrease BP while decreasing triglycerides (TGs) increasing high density lipoprotein (HDL) and lowering low density lipoprotein (LDL) A. Ezetimibe B. Caduet C. Vytorin D. Advicor

B

a nursing student learns about modifiable risk factors for coronary artery disease. which factors does this include? (select all that apply) A. age B. hypertension C. obesity D. smoking E. stress

B, C, D, E

A pt's serum potassium value is below 2.8 mEq/L. The pt is also on digoxin. The nurse quickly assesses the pt for which cardiac problem before notifying the provider? A. Cardiac murmur B. Cardiac dysrhythmia C. CHF D. Cardiac tamponade

B. Cardiac dysrhythmia

Which clinical condition can result from hypocalcemia? A. Stimulated cardiac muscle contraction B. Increased intestinal and gastric motility C. Decreased peripheral nerve excitability D. Increased bone density

B. Increased intestinal and gastric motility

Which pt with the highest risk for acidosis must the nurse care for first? A. Pt in mild pain with a kidney stone B. Pt with COPD and pulse ox 88% on 2 L oxygen C. Pt who had a seixure prior to admission with pulse ox 91% D. Pt with a rectal tube in place for frequent diarrhea

B. Pt with COPD and pulse ox 88% on 2 L oxygen

A nurse assesses a young female client who is prescribed isotretinoin (Accutane) which question should the nurse ask prior to starting this therapy? A. do you spend a great deal of time in the sun? B. have you or any family members ever had skin cancer? C. which method of contraception are you using? D. do you drink alcoholic beverages?

C

a client is admitted with a PE the client is young, healthy and active and has no known risk factors for PE. what action by the nurse is most appropriate? A. encourage the client to walk 5 minutes each hour B. refer the client to smoking cessation classes C. teach the client about factor V leiden testing D. tell the client that sometimes no cause for disease is found

C

an ED nurse is caring for a client who is homeless. which action should the nurse take to gain the clients trust? A. speak in a quiet and monotone voice B. avoid eye contact with the client C. listen to the clients concerns and needs D. ask security to store the clients belongings

C

the UAP tells the nurse that the dying pt is manifesting a death rattle. which action would the nurse perform? A. instruct the UAP to initiate postmortem care B. notify the family that the pt has died C. turn the ppt on the side to reduce gurgling D. tell the UAP that this is expected and nothing can be done.

C

what is the normal measurement of the PR interval in a ECG? A. less than 0.11 seconds B. 0.06-0.10 seconds C. 0.12-0.20 seconds D. 0.16-0.26 seconds

C

To ensure the safety of a pt with metabolic alkalosis, which task is best delegated to the UAP? A. Watch the pt when he or she eats or drinks anything B. Sit with the pt to prevent wandering C. Assist the pt to the bathroom as needed D. Remove all sharp objects from the bedside table

C. Assist the pt to the bathroom as needed

The nurse is caring for a patient with excessive alcohol ingestion and salicylate intoxication. What is the most likely acid-base imbalance this patient will have? A. Bicarbonate under elimination B. Bicarbonate loss C. Metabolic acidosis D. Metabolic alkalosis

C. Metabolic acidosis

The nurse is caring for several older adult pts who are at risk for dehydration. Which task can be delegated to the unlicensed assistive personal? A. Withhold fluids if pts have bowel or bladder incontinence B. Assess for and report any difficulties that pts are having in swallowing C. Stay with pts while they drink fluids and note the exact amount digested D. Divide the total amount of fluids needed over a 24 hour period and note in medical record.

C. Stay with pts while they drink fluids and note the exact amount digested

A nurse assesses a client who has burn injuries and notes crackles in bilateral lung bases, a respiratory rate of 40 breaths/min, and a productive cough with blood-tinged sputum. Which action should the nurse take next? A. administer furosemide (Lasix) B. perform chest physiotherapy C. document and reassess in a hour D. place the client in an upright position

D

A nursing student is struggling to understand the process of graft vs host disease. What explanation by the nurse instructor is best? A. because of immunosuppression, the donor cells take over B. Its like a transfusion reaction because no perfect matches exist C. the clients cells are fighting donor cells for dominance D. the donors cells are actually attacking the clients cells

D

A nursing student wants to know why clients with COPD tend to be polycythemic. What response by the nurse instructor is best? A. it is due to side effects of medications for bronchodilation B. it is from overactive bone marrow in response to chronic disease C. it combats the anemia caused by an increased metabolic rate D. it compensates for tissue hypoxia caused by lung disease

D

After teaching a client how to care for a furuncle in the axilla, a nurse assesses the clients understanding. Which statement indicates the client correctly understands the teaching? A. I'll apply cortisone cream to reduce the inflammation B. I'll apply a clean dressing after squeezing out the pus C. I'll keep my arm down at my side to prevent spread D. I'll cleanse the area prior to applying antibiotic cream

D

a pt is at risk for heart failure but currently has no official medical diagnosis. while assessing the pts lungs the nurse hears profuse fine crackles. what does the nurse do next? A. report the finding to the health care provider B. document the finding as a baseline for later comparison C. give the pt low flow supplemental o2 D. ask the pt to cough and reauscultate the lungs

D

a pt is prescribed atrovastatin the nurse instructs the pt to watch for and report which side effect? A. nausea and vomiting B. cough C. headaches D. muscle cramps

D

a student nurse asks for an explanation of refractory hypoxemia. what answer by the nurse instructor is best? A. it is chronic hypoxemia that accompanies restrictive airway disease B. it is hypoxemia from lunch damage due to mechanical ventilation C. it is hypoxemia that continues even after the client is weaned from oxygen D. it is hypoxemia that persists even with 100% oxygen administeration

D

an ED nurse is caring for a client who has died from a suspected homicide. which action should the nurse take? A. remove all tubes and wires in preparation for the medical examiner B. limit the number of visitors to minimize the family's trauma C. consult the bereavement committee to follow up with the grieving family D. communicate the clients death to the family in a simple and concrete manner

D

during assessment of a pt with heart failure, the nurse notes that the pts pulses alternate in strength what does this assessment indicate to the nurse? A. pulses paradoxus B. orthostatic hypotension C. hypotension D. pulsus alternans

D

which blood pressure is considered normal for an adult pt over 60 years of age? A. 162/92 B. 150/94 C. 156/90 D. 144/88

D

which intervention should be done when performing postmortum care A. place the head of the bed at 30 degrees B. remove pillows from under the head C. remove dentures and carefully clean and store them D. place pads under the hips and around the perineum

D

which pt has an abnormal heart sound A. S1 in a 45 year old pt B. S2 in a 30 year old pt C. S3 in a 15 year old pt D. S3 in a 54 year old pt

D

The nurse is assessing a patient's IV site and identifies signs and symptoms of infiltration. What is the first action that the nurse implements for this patient? A. Elevates the extremity. B. Applies a sterile dressing if weeping from the tissue has occurred. C. Removes the IV access. D. Stops the infusion

D. Stops the infusion

A 65-year-old patient has been receiving IV fluids at 100mL/hr of D(sub5)1/2% normal saline (NS) for the past 3 days, along with IV antibiotic therapy. After receiving the new antibiotic, the patient reports chills and a headache. On assessment, the patient's temperature is elevated. What complication do these assessment finding indicate? A. Catheter-rel

a

A cluster of H5N1 bird influenza cases occurs. Which intervention is MOST appropriate for this outbreak of flu? a. administer 2 Vepacel injections 28 days apart b. avoid the use of antiviral drugs such as zanamivir c. give oral ABX as directed by HCP d. restrict fluids for all infected individuals

a

A critical concern for a pt returning to the unit after a surgical procedure is related to impaired oxygenation caused by inadequate ventilation. Which ABG value and assessment finding indicates to the nurse that O2 and incentive spirometry must be administered? a. PaO2 is 89 mm Hg with crackles b. PaO2 is 90 mm Hg with wheezing c. PCO2 is 38 mm Hg with clear lung sounds d. PCO2 is 45 mm Hg with atelectasis

a

A patient comes to the HCP's office for an annual physical. THe pt. reports having a persistent, nagging cough. Which question does the nurse ask first about this symptom? a. When did the cough start? b. Do you have a family history of lung cancer? c. Have you been running a fever? d. Do you have sneezing and congestion?

a

A pt enters the ED after being punched in the throat. What does the nurse monitor for? a. aphonia b. dry cough c. crepitus d. loss of gag reflex

a

A pt has an inner maxillary fixation. The nurse encourages the pt to ear which kind of food? a. milkshakes b. cottage cheese c. tea and toast d. tuna noodle casserole

a

A pt is diagnosed with pneumonia. During auscultation of the lower lung fields, the nurse hears hoarse crackles and identifies the pts problem of impaired oxygenation. What is the underlying physiologic condition associated with the patient's condition? a. hypoxemia b. hyperemia c. hypocapnia d. hypercapnia

a

A pt is scheduled to have a pulmonary function test. Which type of information does the nurse include in the nursing history so that PFT results can be appropriately determined? a. Age, gender, race, height, weight, and smoking status b. Occupational status, activity tolerance for activities of daily living c. Medication history and history of allergies to contrast media d. History of chronic medical conditions and surgical procedures

a

A pt reports throat soreness and swelling, purulent nasal drainage, post-nasal drip, fever, dental pain and ear pressure. Which disorder does the nurse suspect? a. bacterial rhinosinusitis b. tonsilitis c. viral rhinosinusitis d. pneumonia

a

An active 55-year-old schoolteacher with COPD taking prednisone asks if it is necessary to get a flu shot. What is the BEST response by the nurse? a. yes, flu shots are highly recommended for pts with chronic illness and/or pts receiving immunotherapy b. no, flu shots are only recommended for pts 60 years and older c. yes, it will help minimize the risk of triggering an exacerbation of COPD d. no, pts who are active, not living in a nursing home, and not health care providers do not need a flu shot

a

In the older adult, there are a decreased number of functional alveoli. To assist the pt to compensate for this change related to aging, what does the nurse do? a. Encourage the pt to ambulate and change postions b. Allow the ptto rest and sleep frequently c. Have face to face conversations when possible d. Obtain an order for supplemental oxygen

a

On post-op assessment, the nurse notes that the pt with a rhinoplasty repeatedly swallows. What is the nurse's FIRST action? a. examine the throat for bleeding b. provide ice chips to ease swallowing c. notify the HCP d. ask if the pt is hungry

a

The HCP orders the discontinuation of the NG tube for a pt with a total laryngectomy. Before discontinuing the tube, which action must be performed? a. the HCP and the nurse will assess the pt's ability to swallow b. reassure the pt that eating and swallowing will be painless and natural c. the nutritionist will evaluate the pt's nutritional status d. the pt will be offered a prn analgesic or an anxiolytic med

a

The nurse has just recieved a patient from the recovery room who is somewhat drowsy, but is capable of following instructions. Pulse oximetry has dropped form 95% to 90% What is the priority nursing intervention? a. Adminster oxygen at 2L/min by nasal cannula, then reassess b. Have the pt perform coughing and deep-breathing exercises, then reassess c. Administer naloxone (Narcan) to reverse narcotic sedation effect d. Withhold narcotic pain medication to reduce sedation effect.

a

The nurse is assessing a pt who has had a neck dissection with removal of muscle tissue, lymph nodes, and the 11th cranial nerve. Which assessment finding is anticipated because of the surgical procedure? a. shoulder drop with an increased limitation of movement b. asymmetrical eye movements and a change of visual acuity c. weak, hoarse voice d. facial swelling with discoloration and bruising around the eyes

a

The nurse is assessing a pt's skin at the site of radiation therapy to the neck. Which skin condition is expected in relation to the radiation treatments? a. red, tender, and peeling b. shiny, pale and tight c. puffy and edematous d. pale, dry and cool

a

The nurse is providing discharge instructions about pneumonia to a pt and family. Which discharge info must the nurse be sure to include? a. complete antibiotics as prescribed, rest, drink fluids, and minimize contact with crowds b. take all ABX as ordered, resume diet and all activities as before hospitalization c. no restrictions regarding activities, diet and rest because the pt is fully recovered when discharged d. continue ABX only until no further signs of pneumonia are present; avoid exposing immunosupressed individuals

a

Which patient is at most risk for development of rhinosinusitis? a. patient with deviated nasal septum b. pt with ear infection c. pt with an infected heart valve d. pt with cellulitis

a

Which pt with the highest risk for developing cancer of the larynx should be alerted about relevant lifestyle modifications to decrease this risk? a. 57-year-old male with alcoholism b. 18-year-old marijuana smoker c. 28-year-old female with diabetes d. 34-year-old male who snorts cocaine

a

he physician orders transtracheal oxygen therapy for a patient with respiratory difficulty. What does the nurse tell the patient's family is the purpose of this type of oxygen delivery sys- tem? a. Delivers oxygen directly into the lungs. b. Keeps the small air sacs open to improve gas exchange. c. Prevents the need for an endotracheal tube. d. Provides high humidity with oxygen deliv- ery

a a. Delivers oxygen directly into the lungs.

41. A patient required emergency intubation and currently has an artificial airway in place. Oxy- gen is being administered directly from the wall source. Why would warmed and humidi- fied oxygen be a more appropriate choice for this patient? a. Helps prevent tracheal damage b. Promotes thick secretions c. Is more comfortable for the patient d. Is less likely to cause oxygen toxicity

a a. Helps prevent tracheal damage

28. A patient requires long-term airway mainte- nance following surgery for cancer of the neck. The nurse is using a piece of equipment to explain the procedure and mechanism that are associated with this long-term therapy. Which piece of equipment does the nurse most likely use for this patient teaching session? a. Tracheostomy tube b. Nasal trumpet c. Endotracheal tube d. Nasal cannula

a a. Tracheostomy tube

An older adult pt often coughs and chokes while eating/trying to take medication. The pt insists that he is ok, but the nurse identifies the priority pt problem of risk for aspiration. Which nursing interventions are used to prevent aspiration pneumonia? SELECT ALL THAT APPLY a. head of the bed should always be elevated during feeding b. monitor the pt's ability to swallow small bites c. give thin liquids to drink in small, frequent amounts d. consult a nutritionist and obtain swallowing studies e. monitor the pt's ability to swallow saliva f. place the pt on NPO status until swallowing is normal

a b d e

Which conditions may cause pts to be at risk for aspiration pneumonia? SELECT ALL THAT APPLY a. continuous tube feedings b. bronchoscopy procedure c. MRI d. decreased LOC e. stroke f. chest tube

a b d e

Which factors contribute to sleep apnea? SELECT ALL THAT APPLY a. smoking b. a short neck c. athletic lifestyle d. small uvula e. enlarged tonsil/adenoids f. underweight for height and gender

a b e

Which instructions must the nurse give to a pt after rhinoplasty to prevent bleeding? SELECT ALL THAT APPLY a. limit/avoid straining during bowel movements (eg Valsalva maneuver) b. do not sniff upwards/blow your nose c. sneeze with your mouth closed for a few days after packing is removed d. forceful coughing should be done to keep airways clear e. avoid aspirin-containing products/NSAIDs f. use a humidifier to prevent mucosal drying

a b e f

The pt has a diagnosis of mild sleep apnea. Which interventions will the nurse teach the pt that may correct this condition? SELECT ALL THAT APPLY a. change sleeping positions b. use a CPAP every night c. look into a weight loss program d. a position fixing device can prevent tongue subluxation e. you may need surgery to remodel your posterior oropharynx f. a prescription for modafinil may help promote wakefulness during the day

a c d

The pt with a total laryngectomy has a laryngectomy button in place. What important teaching points must the nurse include when teaching the pt about a laryngectomy button? SELECT ALL THAT APPLY a. a laryngectomy button is shorter and softer than a laryngectomy tube b. a laryngectomy button comes in only one size c. a laryngectomy button is more comfortable than a laryngectomy tube d. a laryngectomy button requires use of an alternative form of communication e. a laryngectomy button requires the use of sterile procedure when it is changed f. a laryngectomy button is longer and narrow like an endotracheal tube

a c d

A 42-year-old pt is admitted with a diagnosis of coccidioidomycosis. Which statements about this diagnosis are accurate? SELECT ALL THAT APPLY a. symptoms of coccidioidomycosis resemble those of other respiratory infections b. coccidioidomycosis is a viral infection caused by the coccidioides organism c. The coccidioides organism is present in the soil, but is inactive d. coccidioidomycosis is sometimes misdiagnosed as flu/pneumonia e. most younger healthy adults recover from the infection without treatment f. severe coccidioidomycosis is treated with drugs such as fluconazole (Diflucan)

a c d e

The nurse is interviewing a pt to assess for risk factors related to head and neck cancer. Which questions are appropriate to include? SELECT ALL THAT APPLY a. how many servings/day of alcohol do you typical drink? b. have you had frequent episodes of acute/chronic visual problems? c. have you had a problem with sores in your mouth? d. when was the last time you saw your dentist? e. do you have recurrent laryngitis/frequent episodes of sore throat? f. how many packs/day do you smoke and for how many years?

a c e f

Which people are at GREATEST risk for developing TB in the US? SELECT ALL THAT APPLY a. an alcoholic homeless man who occasionally stays in a shelter b. a college student sharing a room in a dorm c. a person with immune dysfunction or HIV d. a homemaker who does volunteer work at a homeless shelter e. immigrants (especially those from the Phillipines/Mexico) f. an adult living in a crowded area such as a long-term-care facility

a c e f

Which pts are at risk for developing health care acquired pneumonia? SELECT ALL THAT APPLY a. confused pt b. pt with atrial fibrillation who is alert and oriented c. pt with gram negative colonization of the mouth d. pt with hyperthyroid disease e. malnourished pt f. pt with influenza

a c e f

A patient with COPD is likely to have which findings on assessment? Select all that apply. a. Body odor and unkempt hair. b. Sitting in a chair leaning forward with elbows on knees. c. unintentional weight gain. d. decreased appetite. e. unexplained weight loss f. crooked fingers

a. Body odor and unkempt hair. b. Sitting in a chair leaning forward with elbows on knees. d. decreased appetite. e. unexplained weight loss

In obtaining a history for a patient with COPD, which risk factors are related to potentially causing or triggering the disease process? Select all that apply. a. Cigarette smoking b. Occupational and air pollution c. Genetic tendencies d. Smokeless tobacco e. Occupation f. Food or drug allergies

a. Cigarette smoking b. Occupational and air pollution c. Genetic tendencies e. Occupation

A patient has developed pulmonary arterial hypertension. What is the goal of drug therapy for this patient? a. Dilate pulmonary vessels and prevent clot formation. b. Decrease pain and make the patient comfortable. c. Improve or maintain gas exchange. d. Maintain and manage pulmonary exacerbation.

a. Dilate pulmonary vessels and prevent clot formation

The nurse is providing discharge instructions to a patient with pulmonary fibrosis and the patient's family. What instructions are appropriate for this patient? Select all that apply. a. Using home oxygen b. Maintaining activity level as before. c. Preventing respiratory infections. d. Limiting fluid intake e. Energy conservation measures. f. Encouraging patient to complete all ADLs.

a. Using home oxygen c. Preventing respiratory infections. e. Energy conservation measures

A patient with COPD has meal-related dyspnea. To address this issue, which drug does the nurse offer the patient 30 mins before the meal? a. albuterol lb. guaifenesin c. fluticasone d. pantoprazole sodium

a. albuterol

61. An older adult patient is at risk for aspirating food or fluids. Which are the most appropriate nursing actions to prevent this problem? (Se- lect all that apply.) a. Provide close supervision if the patient is self-feeding. b. Instruct the patient to tilt the head back when swallowing. c. Obtain an order for a clear liquid diet and offer small but frequent amounts. d. Instruct the patient to tuck the chin down when swallowing. e. Place the patient in an upright position.

a. d. e a. Provide close supervision if the patient is self-feeding. d. Instruct the patient to tuck the chin down when swallowing. e. Place the patient in an upright position.

The nurse is helping a patient learn about managing her asthma. What does the nurse instruct the patient to do? a. keep a symptom diary to identify what triggers the asthma attack b. make an appointment with an allergist for allergy therapy c. take a low dose of aspirin every day for the anti-inflammatory action d. drink large amounts of clear fluid to keep mucus thin and watery.

a. keep a symptom diary to identify what triggers the asthma attack

Which are characteristics of asthma? Select all that apply. a. narrowed airway lumen due to inflammation b. increased eosinophils c. increased secretions d. intermittent bronchospasm e. loss of elastic recoil f. stimulation of disease process by allergies

a. narrowed airway lumen due to inflammation b. increased eosinophils d. intermittent bronchospasm f. stimulation of disease process by allergies

Which statement by the nursing student indi- cates an understanding of the deflation of the tracheostomy cuff? a. "The cuff is deflated to allow the patient to speak." b. "The cuff is deflated to permit suctioning more easily." c. "The cuff should never be deflated because the patient will choke." d. "The cuff should be deflated to facilitate access for tracheostomy care."

aa. "The cuff is deflated to allow the patient to speak."

primary prevention examples:

avoidance of known or potential carcinogens (skin protection during skin exposure) modifying associated factors (alcohol, fatty diet, low fiber, multiple sex partners) removal of at risk tissues (removing moles, colon polyps, removing breasts) chemoprevention (drugs, chemicals, natural nutrients, substances to disrupt one or more steps important to cancer development. goal is prevention, for healthy people with no known cancer risk, people with greater risk due to environmental exposure, people with precancerous lesions, people with history of cancer Vaccination- Gardasil, HPV, prevent viral infection.

65. A patient with a tracheostomy is unable to speak. He is not in acute distress, but is gestur- ing and trying to communicate with the nurse. Which nursing intervention is the best ap- proach in this situation? a. Rely on the family to interpret for the pa- tient. b. Ask questions that can be answered with a "yes" or "no" response. c. Obtain an immediate consult with the speech therapist. d. Encourage the patient to rest rather than struggle with communicatio

b b. Ask questions that can be answered with a "yes" or "no" response.

The nurse is preparing a community info packet about "bird flu". What info does the nurse include in the packet? SELECT ALL THAT APPLY a. in the event of an outbreak, do not eat any cooked/uncooked poultry products b. prepare a minimum of 2 weeks supply of food, water, and routine prescription drugs c. listen to public health announcements and early warning signs for disease outbreaks d. avoid traveling to areas where there has been a suspected outbreak of disease e. obtain a supply of antiviral drugs such as oseltamivir f. in the event of an outbreak, avoid going to public areas such as churches or schools

b c d f

For which complications does the nurse monitor when a pt is diagnosed with rhinosinusitis? SELECT ALL THAT APPLY a. pneumonia b. meningitis c. abscess d. TB e. cellulitis f. tonsilitis

b c e

2. Which conditions can increase the body's need for more oxygen? (Select all that apply.) a. Hypothyroid b. Sickle cell disease c. Infection in the blood d. Body temperature of 101° F e. Hemoglobin level of 8.7 g/dL

b, c, d, e b. Sickle cell disease c. Infection in the blood d. Body temperature of 101° F e. Hemoglobin level of 8.7 g/dL

8. The home health nurse has been caring for a patient with a chronic respiratory disorder. Today the patient seems confused when she is normally alert and oriented x 3. What is the priority nursing action? a. Notify the physician about the mental sta- tus change. b. Take vital signs and check the pulse oxim- eter readings. c. Ask the patient's family when this behavior started. d. Perform a mental status examinatio

b,b. Take vital signs and check the pulse oxim- eter readings.

Which substances from cigarette smoke have been implicated in the development of serious lung diseases? SATA a. Carbon dioxide b. Nicotine c. Tar d. Carbon monoxide e. Dust particles

b,c,d

Which factors or conditions cause a decreased (below normal) PETCO level due to abnormal ventilation? SATA a. Hyperthermia b. Hypotension c. Apnea d. Hyperventilation e. Hypothermia

b,c,e

Which respiratory changes occur as a result of aging? SATA a. Increased elasetic recoil b. Dilation of alveolar ducts c. Decreased ability to cough d. Alveolar surface tention increases e. Diffusion capacity decreases

b,c,e

The nurse is performing a respiratory assessment including pulse oximetry on several pts. Which conditions of situations may cause an artifically low reading? SATA a. Fever b. Anemia c. Receiving narcotic pain medications d. Peripheral artery disease e. History of respiratory disease such as cystic fibrosis or tuberculosis

b,d

The nurse is teaching a patient with COPD about his medications. Which statement by the patient indicates the need for additional teaching? a. "I will carry my albuterol with me at all times." b. "I will use my salmeterol whenever I start to feel short of breath." c. "I will check my heart rate before and after my exercise period." d. "I will use my ipratropium 4 times a day."

b. "I will use my salmeterol whenever I start to feel short of breath.

Which blood pH value does the nurse interpret as within normal limits? a. 7.27 b. 7.37 c. 7.47 d. 7.5

b. 7.37

A patient is undergoing diagnostic testing for possible cystic fibrosis. Which non-pulmonary assessment findings does the nurse expect to observe in a patient with CF? Select all that apply. a. Peripheral edema b. Abdominal distention c. Steatorrhea d. Constipation e. Gastroesophageal reflux f. Malnourished appearance

b. Abdominal distention c. Steatorrhea e. Gastroesophageal reflux f. Malnourished appearance

The nurse is working for a manufacturing company and is responsible for routine employee health issues. Which primary prevention is most important for those employees at high risk for occupational pulmonary disease? a. Screen all employees by use of chest x-ray films twice a year b. Advise employees not to smoke and to use masks and ventilation equipment c. Perform pulmonary function tests once a year on all employees. d. Refer at-risk employees to a social worker for information about pensions.

b. Advise employees not to smoke and to use masks and ventilation equipment

Which sites are commonly affected by lung cancer metastasis? Select all that apply. a. Heart. b. Bone c. Liver. d. Colon e. Brain f. Adrenal glands

b. Bone c. Liver. e. Brain f. Adrenal glands

A patient who is allergic to dogs experiences a sudden "asthma attack." Which assessment finding does the nurse expect for this patient? a. Slow, deep, pursed-lip respirations b. Breathlessness and difficulty completing sentences c. Clubbing of the fingers and cyanosis of the nail beds. d. Bradycardia and irregular pulse.

b. Breathlessness and difficulty completing sentences

A patient with respiratory difficulty has completed a pulmonary function test before starting any treatment. The peak expiratory flow (PEF) is 15%-20% below what is expected for the adult's age, gender, and size. the nurse anticipates this patient will need additional information about which topic? a. Further diagnostic tests to confirm pulmonary hypertension. b. How to manage asthma medications and identify triggers. c. Smoking cessation and its relationship to COPD. d. How to manage the acute episode of respiratory infection.

b. How to manage asthma medications and identify triggers.

A patient is diagnosed with cor pulmonale secondary to pumonary hypertension and is receiving an infusion of epoprostenol through a small portable IV pump. What is the critical priority for the patient? a. Strict aseptic technique must be used to prevent sepsis. b. Infusion must not be interrupted, even for a few minutes. c. The patient must have a daily dose of warfarin. d. the patient must be assessed for angina-like chest pain and fatigue.

b. Infusion must not be interrupted, even for a few minutes.

A patient has a history of COPD but is admitted for a surgical procedure that is unrelated to the respiratory system. To prevent any complications related to the patient's COPD, what action does the nurse take? a. Assess the patient's respiratory system every 8 hrs b. Monitor for signs and symptoms of pneumonia c. Give high-flow oxygen to maintain pulse oximetry readings. d. Instruct the patient to use a tissue if coughing or sneezing.

b. Monitor for signs and symptoms of pneumonia

A patient is receiving a chemotherapy agent for lung cancer. The nurse anticipates that the patient is likely to have which common side effect? a. Diarrhea b. Nausea c. Flatulence d. Constipation

b. Nausea

What is the priority medical-surgical concept for patients with noninfectious lower respiratory problems such as emphysema? a. perfusion b. gas exchange c. cellular regulation d. tissue integrity

b. gas exchange

Antigens

bad guys

high grade cancer

barely resemble the tissue from which they arose and are aggressive and spread rapidly.

42. A patient has an endotracheal tube and re- quires frequent suctioning for copious secre- tions. What is a complication of tracheal suc- tioning? a. Atelectasis b. Hypoxia c. Hypercarbia d. Bronchodilation

bb. Hypoxia

oma

benign mass involving epithelial or connective tissue (lipoma), or round cell tumors (histicytoma, lymphoma, melanoma)

Congestion

blood stagnation from decreased flow away from the site swelling

Angiogensis inhibitors s/e

bone marrow suppression, headache, GI distress, muscle/joint pain

cancers commonly arise in

bone marrow, skin, lining of the GI tract and ductal cells of the breast because they have the ability to divide. they all normally undergo cell division.

A 30 year old is admitted with severe coughing "fits" lasting several minutes. He tells you that he developed cold symptoms a little over a week ago. Which PRIORITY question would the nurse ask him? a. has your HCP prescribed ABX for your symptoms? b. on average, how often do you experience cold symptoms each winter? c. did you receive the usual childhood immunization when you were a child? d. do you smoke or did you ever smoke or use any tobacco products?

c

A pt had a rhinoplasty and is preparing for discharge home. A family member is instructed by the nurse to monitor the pt for postnasal drip by using a flashlight to look in the back of the throat. If the bleeding is noted, what does the nurse tell the family member to do? a. place ice packs on the back of the neck and apply pressure to the nose b. hyperextend the neck and apply pressure and ice packs as needed c. seek immediate medical attention for the bleeding d. monitor for 24 hrs if the bleeding appears to be a small amount

c

A pt has a positive skin test result for TB. What explanation does the nurse give to the pt? a. there is active disease, but you are not yet infectious to others b. there is active disease, and you need immediate treatment c. you have been infected, but this does not mean active disease is present d. a repeat skin test is necessary because the test could give a false-positive result

c

A pt has demonstrated anxiety since a diagnosis of neck cancer. The surgery and radiation therapy are completed. Which behavior indicates that a pt's fears are decreasing? a. repeatedly asks the same questions and seeks to re-validate all info b. states that he is less anxious but is irritable and tense whenever questioned c. makes a plan to contact the American Cancer Society Visitor Program d. makes a plan to share personal belongings with friends and family

c

An older adult pt asks the nurse how often one should receive the pneumococcal vaccine for pneumonia prevention. What is the nurse's BEST response? a. every year, when the pt is receiving the "flu shot" b. the standard is vaccination every 3 years c. it is usually given once 6-12 months after the Prevnar 13 vaccine d. there is no set schedule; it depends upon the pts history and risk factors

c

In a long term care facility caring for older adults and those who are immunocompromised , one employee and several pts have been diagnosed with influenza. What does the supervising nurse do to decrease risk of infection to other pts? a. ask employees who have the flu to stay home for at least 24 hrs b. place any pt with a sore throat, cough, or rhinorrhea into isolation for 1-2 wks c. ask employees with flu symptoms to stay home for up to 5 days after onset of symptoms d. recommend that all pts and employees be immediately vaccinated for the flu

c

In order to facilitate comfort and breathing for the pt with a laryngeal tumor, the nurse should use which position? a. Sim's b. supine c. Fowler's d. prone

c

In the event of a new severe acute respiratory syndrome (SARS) outbreak, what is the nurse's PRIMARY role? a. immediately report new cases of SARS to the CDC b. administer O2, standard ABX, and supportive therapies to pts c. prevent the spread of infection to other employees and pts d. initiate and strictly enforce contact isolation procedures

c

In the older adult with chronic pulmonary disease, there is a loss of elastic recoiling of the lung and decreased chest wall compliance. What is the result of this occurance? a. The thoracic area becomes shorter b. The pt has an increased activity tolerance c. There is an increase in anteroposterior ratio d. The pt has severe shortness of breath

c

Pulmonary function tests are scheduled for a pt w/ a history of smoking who reports dyspnea and chronic cough. What wil pt teaching information about this procedure include? a. Do not smoke for at least 2 weeks before the test b. Bronchodilator drugs may be withheld 2 days before the test c. The pt will breathe through the mouth and wear a nose clip during the test d. The patient will be expected to walk on a treadmill during the test.

c

The nurse is caring for a pt who had a constructive neck surgery and observes bright red blood spurting from the tissue flap that is covering the carotid artery. Which action must the nurse take FIRST? a. call the surgeon and alert the OR b. call the RRT c. apply immediate, direct pressure to the site d. apply a bulky sterile dressing and secure the airway

c

The nurse is caring for several pts who are at risk because of problems related to the upper airway. What are the PRIORITY assessments and actions for these pts? a. thickness of oral secretions; encourage ingestion of oral fluids b. anxiety and pain; provide reassurance and NSAIDs c. adequacy of oxygenation; ensure an unobstructed airway d. evidence of spinal cord injuries; obtain an order for x-rays

c

The nurse is inspecting a pt's chest and observes an increase in anteroposterior diameter of the chest. When is this an expected finding? a. With a pulmonary mass b. Upon deep inhalation c. In older patients d. With chest trauma

c

The nurse is palpating a pt's chest and identifies an increased tactile fremitus or vibration of the chest wall produced when the pt. speaks. What does the nurse do next? a. Observe for other findings associated with subcutaneous emphysema b. Document the observation as an expected normal finding c. Observe the patient for other findings associated with a pnemothorax d. Document the observation as a pleural friction rub

c

The pt with laryngeal trauma develops stridor. What is the nurse's HIGHEST priority intervention? a. apply O2 by nasal cannula b. obtain ABGs c. call RRT d. perform a maneuver to open the airway

c

The nurse is teaching a patient how to interpret peak expiratory flow (PEF) readings and to use this information to manage drug therapy at home. Which statement by the patient indicates a need for additional teaching? a. "If the reading is in the green zone, there is no need to increase the drug therapy." b. "Red is 50% below my 'personal best.' I should try a rescue drug and seek help." c. "If the reading is in the yellow zone. I should increase my use of my inhalers." d. "If frequent yellow readings occur, I should see my doctor for a change in medications."

c. "If the reading is in the yellow zone. I should increase my use of my inhalers."

A family member of a patient with COPD asks the nurse, "What is the purpose of making him cough on a routine basis?" What is the nurse's best response? a. "We have to check the color and consistency of his sputum." b. "We don't want him to feel embarrassed when coughing in public, so we actively encourage it." c. "It improves air exchange by increasing airflow in the larger airways." d. "If he cannot cough, the physician may elect to do a tracheostomy."

c. "It improves air exchange by increasing airflow in the larger airways."

A patient is having pain resulting from bone metastases caused by lung cancer. What is the most effective intervention for relieving the patient's pain? a. Support the patient through chemotherapy b. Handle and move the patient very gently c. Administer analgesics around the clock d. Reposition the patient, and use distraction

c. Administer analgesics around the clock

A patient has a chest tube in place. What does the water in the water seal chamber do when the system is functioning correctly? a. Bubbles vigorously and continuously b. Bubbles gently and continuously c. Fluctuates with the patient's respirations d. Stops fluctuation, and bubbling is not observed.

c. Fluctuates with the patient's respirations

A patient has COPD with chronic difficulty breathing. In planning this patient's care, what condition must the nurse acknowledge is present in this patient? a. Decreased need for calories and protein requirements since dyspnea causes activity intolerance. b. COPD has no effect on calorie and protein needs, meal tolerance, satiety, appetite, and weight. c. Increased metabolism and the need for additional calories and protein supplements. d. Anabolic state, which creates conditions for building body strength and muscle mass.

c. Increased metabolism and the need for additional calories and protein supplements.

Which statement is true about radiation therapy for lung cancer patients? a. It is given daily in "cycles" over the course of several months. b. It causes hair loss, nausea, and vomiting for the duration of treatment c. It causes dry skin at the radiation site, fatigue, and changes in appetite with nausea d. It is the best method of treatment for systemic metastatic disease.

c. It causes dry skin at the radiation site, fatigue, and changes in appetite with nausea

A patient with chronic bronchitis often shows signs of hypoxia. Which clinical manifestation is the priority to monitor in this patient? a. Chronic, non-productive dry cough b. Clubbing of fingers c. Large amounts of thick mucous d. Barrel chest

c. Large amounts of thick mucous

A patient had prolonged occupational exposure to petroleum distillates and subsequently developed a chronic lung disease. The patient is advised to seek frequent health examinations because there is a high risk for developing which respiratory disease condition? a. Tuberculosis b. Cystic fibrosis c. Lung cancer d. Pulmonary hypertension

c. Lung cancer

The patient has one gene allele for alpha-1 anti-trypsin (AAT) that is faulty and one that is normal. Which statement is true about this patient? a. The patient will have an alpha-1 anti-trypsin deficiency and is at risk for COPD. b. The patient will not be at risk for development of COPD. c. The patient will be a carrier for alpha-1 anti-trypsin deficiency. d. The patient will make enough alpha-1 anti-trypsin to avoid COPD even if exposed to smoking.

c. The patient will be a carrier for alpha-1 anti-trypsin deficiency.

genetic testing for cancer predisposition

can confirm or rule out a persons genetic risk for a few specific cancers. tests on blood are expensive and often not covered by insurance.

chemical carcinognisis

can occur from exposures to many known substances, drugs, and other products used in everyday life.

Neoplasia

cancer - out of control cell growth

Malignant cell indicates?

cancer, and can lead to death w/o intervention

Infectious

caused by some organism within the host

cancer classification

classified by the type of tissue from which they arise, or biologic behavior, anatomic site, and degree of differentiation.

grading

classifies cellular aspects of the cancer; needed because some cancer cells are more malignant than others. varying aggressiveness and sensitivitiy to treatment.

IgM

common, faster and more efficient

A pt diagnosed with TB agrees to take the medication as instructed and to complete the therapy. When does the nurse tell the pt is the BEST time to take the medication? a. before breakfast b. after breakfast c. midday d. bedtime

d

A pt has been compliant with drug therapy for TB and has returned as instructed for follow up. Which result indicates that the patient is NO LONGER infectious/communicable? a. negative chest xray b. no clinical symptoms c. negative skin test d. 3 negative sputum cultures

d

An older adult pt who is talking and laughing while eating begins to choke on a piece of meat. What is the INITIAL emergency management for this pt? a. several sharp blows between the scapula b. call the RRT c. nasotracheal suctioning d. abdominal thrusts (Heimlich maneuver)

d

What type of treatment has the HIGHEST cure rate for small cancers of the head and neck? a. surgery b. chemo c. laser surgery d. radiation therapy

d

The nurse is taking a report on a patient who had a pneumonectomy 4 days ago. Which question is the best to ask during the shift report? a. "Does the physician want us to continue encouraging use of the spirometer?" b. "How much drainage did you see in the Pleur-evac during your shift?" c. "Do we have a request to 'milk' the patient's chest tube?" d. "Does the surgeon want the patient placed on the operative or nonoperative side?"

d. "Does the surgeon want the patient placed on the operative or nonoperative side?"

A patient with pulmonary arterial hypertension is prescribed bosentan. For which side effect must the nurse monitor? a. Bradycardia b. Increased risk for blood clotting c. Decreased urine output d. Hypotension

d. Hypotension

Which intervention promotes comfort in dyspnea management for a patient with lung cancer? a. Administer morphine only when the patient requests it. b. Place the patient in a supine position with a pillow under the knees and legs c. Encourage coughing and deep-breathing and independent ambulation d. Provide supplemental oxygen via cannula or mask

d. Provide supplemental oxygen via cannula or mask

What principle guides the nurse when providing oxygen therapy for a patient with COPD? a. The patient depends on a high serum carbon dioxide level to stimulate the drive to breathe. b. The patient requires a low serum oxygen level for the stimulus to breathe to work. c. The patient who receives oxygen therapy ata high flow rate is at risk for a respiratory arrest. d. The patient should receive oxygen therapy at rates to reduce hypoxia and bring SpO2 levels up between 88%-92%.

d. The patient should receive oxygen therapy at rates to reduce hypoxia and bring SpO2 levels up between 88%-92%.

22. A patient with an oxygen delivery device would like to ambulate to the bathroom but the tubing is too short. Extension tubing is added. What is the maximum length of the tubing that can be added in order to deliver the amount of oxygen needed for that device? a. 25 feet b. 35 feet c. 45 feet d. 50 feet

dd. 50 feet

60. A patient has a cuffed tracheostomy tube with- out a pressure relief valve. To prevent tissue damage of the tracheal mucosa, what does the nurse do? a. Deflate the cuff every 2 to 4 hours and maintain as needed. b. Change the tracheostomy tube every 3 days or per hospital policy. c. Assess and record cuff pressures each shift using the occlusive technique. d. Assess and record cuff pressures each shift using minimal leak technique.

dd. Assess and record cuff pressures each shift using minimal leak technique.

49. A patient with an endotracheal tube in place has dry mucous membranes and lips related to the tube and the partial open mouth position. What techniques does the nurse use to provide this patient with frequent oral care? a. Cleanses the mouth with glycerin swabs. b. Provides alcohol-based mouth rinse and oral suction. c. Cleanses with a mixture of hydrogen per- oxide and water. d. Uses toothettes or a soft-bristled brush moistened in water.

dd. Uses toothettes or a soft-bristled brush moistened in water.

three interacting factors influence cancer development

exposure to carcinogens; genetic predisposition; immune function

Cell Mediated Response

fast reponse T-cell lymphocytes involved recognize self vs nonself

IgA

found usually in surface linings, mucous membranes

physical carcinogenesis

from physical agents or events causes cancer by the same mechanism as for chemical carcinogens. radiation and chronic irritation.

personal factors

immune function, age, genetic risk affect whether a person is likely to develop cancer.

Biologic Response Modifiers (BRMs)

immunotherapy that modifies the pt biological response to tumor cells

Effects that chemotherapy has on blood

impaired clotting, thrombocytopenia, and anemia

vascular endothelial growth factor (VEGR) s/e

impaired wound healing, bone marrow suppression, and HTN

Basophilia

increased basophils

Eosinophila

increased eosinophils

Artificial Immunity

preferred method in our patients due to the serious nature of diseases against which we vaccinate

Types of hormonal manipulation

steroids, enzyme inhibitors

contact inhibition

stopping of further rounds of cell division when dividing cell is completely surrounded and touched by other cells.

carcinogens

substances that change the activity of a cell so that the cell becomes a cancer cell.

oldest form of cancer treatement

surgery

Etiology

the actual cause of disease

Immunoglobulins IgG

the most common circulating immunoglobulin, intial response to pathogen, helps coat it to be recognized further

Passive Immunity

the mother passes on her antibodies in the Colostrum, the type of "Milk" provided for the first 24-48 hours

Chemical Barrier

the mucus and sebum produced inhibit pathogens

primary tumor

the original tumor; usually identified by the tissue from which it arose such as breast or lung cancer.

malignant transformation

the process of changing a normal cell into a cancer cell

Cancer Etiology and Genetic Risk. What factors causes cancer?

(COPD VD) 1) Chemical Carcinogenesis 2) Oncogene Activation 3) Physical Carcinogenesis 4) Dietary Factors 5) Virus Carcinogenesis 6) Personal factors

Leukocytes

(white blood cells) which are produced in the bone marrow, mount the inflammatory response

Specific Immunity

- 3rd line of defense - Accomplished mostly by two types of specialized lymphocytes

Inflammation

- General term for the body's non-specific response to any invader including parasite, bacteria, foreign body, bite - Inflammation does not = infection

Signs Of Inflammation

- Heat - Redness - Swelling - Pain - Loss of Function

Ionizing radiation is?

- Is found in natural elements such as the soil and rock like uranium. - X-rays

Basophils

- Least prevalent, least understood - Basophilic

Granulocytes

- Neutrophils - Eosinophils - Basophils

Lymphocyte

- Not phagocytic at all, involved in more specific immunity - B-Lymph and T-lymph - Lymphocytic

Eosinophils

- Phagocytic, primary cell involved in allergic and parasitic invasion - Eosinophilic

Colostrum

- Rich in antibodies, and is crucial to the offspring's early survival - Why we vaccinate when and how we do

Humoral Response

- Slower type of specific immunity - B-lymphocytes are involved, producing antibodies in responseto specific antigens

Hypoactivity

- Too little immune response - can cause minor illness

B - Cells

- active plasma cells which make immunoglobulins - start in bone marrow and end in bone marrow

Monocyte

- called Macrophaes or histiocytes - largest leukocyte - part of the body's clean up crew - Granlomatous

Memory Cells

- clone themselves - quickly recognize invaders in future in attacks

Exudates

- color - cellularity - protein count

T-Cells

- in spleen and lymph nodes - kill the invader or making it harmless to the host

Mechanical Barrier

- protects our bones and organs - physical barrier from pathogens & climate extremes

Malignant

- rapid growth - irregular margins - not contained or encapsulated - difficult to achieve clean margins w/ surgery - likely to spread or recur

Benign

- slow growing - regular margins - Encapsulated - easy to achieve complete margins w/surgical excision - unlikely to spread or recur

Benign cells vs Normal cells

-Benign cells are regular shape and tender to touch - Same characteristics as normal cells except for growth in which normal cells are regulated and benign cells grow by hyperplastic expansion

Modifying associated factors as a part of Primary prevention of cancer includes?

-Limiting intake of alcohol - Adding more fruits, vegetables, and whole grains to diet

Vitamin C

0.4-1.5 mg/dL

TSH

0.5-5mlU/L

Creatinine

0.8-1.3 mg/dL

International Normalized Ratio (INR)

0.9-1.2

Basophils

1%

Description of Malignant cell includes?

1) Abnormal 2) Serves no useful function 3) Harmful to normal body tissues

"Primary" cancer prevention include?

1) Avoidance of known or potential carcinogens 2) Removal of "at risk" tissues 3) Modifying associated factors 4) Chemoprevention 5) Vaccinations

Two names for cancer development include?

1) Carcinogenesis 2) Oncogenesis

Cellular Regulation controls what

1) Cell division making sure the right amount of cells are present at each organ or tissue to promote optimal function

Environment carcinogens include?

1) Chemical 2) Physical 3) Viral

Examples of carcinogens include?

1) Chemical agents 2) Physical agents 3) Viruses

Cancer Classification includes what?

1) Grading 2) Ploidy 3) Staging 4) TNM 5) Doubling time 6) Mitotic index

Three Important Factors that influence cancer development include?

1) Immune Factors 2) Genetic Risk 3) Exposure ( to carcinogens)

Personal factors include?

1) Immune function 2) Genetic Risk 3) Advance Age

Personal Factors ( that causes cancer) include?

1) Immune system 2) Genetic Risk 3) Advancing Age

Two types of Radiation include?

1) Ionizing radiation 2) Ultraviolet (UV)

Dietary Factors that influence cancer include?

1) Low fiber 2) High intake of red meats or animal fats 3) Preservatives 4) Additives 5) Foods high in fat 6) Alcohol

Examples of benign tumor cells are?

1) Moles 2) Skin tags 3) Endometriosis 4) Nasal polyps ( colon polyps) 5) Uterine fibroid tumors

Two Physical carcinogenesis include?

1) Radiation 3) Chronic Irritation

Eosinophils

1-2%

Non-Specifc Immunity

1. Species protection 2. Skin/integument 3. Inflammation

3 ways that BRM's work

1.) can have direct anti-tumor activity 2.) can interfere with cancer cell differentiation, transformation, metastasis 3.) Can increase immune function

Magnesium (Mg)

1.8-2.6 mEq/L

Vitamin B

130-700 ng/L

Sodium (Na)

136-145 mEq/L

The Integument

1st line of disease

Bilirubin

2-20 µmol/L

1L of water

2.2 lbs

Activated partial thromboplastin time (aPTT)

20-40 seconds

Bicarbonate (HCO3)

21-28 mEq/L

Normal Osmolarity of blood

270-300 mOsm/L

Inflammation

2nd line of defense

Total cholesterol

3-5.5 mmol/L

Potassium (K)

3.5-5.0 mEq/L

Vitamin A

30-65 µg/dL

Partial Pressure Carbon Dioxide (PaCO2)

35-45 mmHg

Albumin

35-50 g/L

HDL

40-80 mg/dL

Urine Level to excrete Toxins

400-600mL

WBC count

5,000-10,000/mm3

Arterial Blood pH

7.35-7.45

Fasting glucose level

70-100mg/dL

Partial Pressure Oxygen (PaO2)

75-100 mmHg

Blood Urea Nitrogen (BUN)

8-21 mg/dL

Environment exposure are responsible for what percent of cancers in the US?

80%

LDL

85-125 mg/dL

Calcium (Ca)

9.0-10.5 mg/dL

Chloride (Cl)

95-105 mEq/L

Hypomagnesemia

< 1.8 mEq/L

Hyponatremia

< 136 mEq/L

Hypokalemia

< 3.5 mEq/L

Hypotonic Fluids

< 300mOsm/L

A1C

< 6.5 %

Hypocalcemia

< 9 mg/dL

Hypernatremia

> 145 mEq/L

Hypermagnesemia

> 2.6 mEq/L

Hypertonic Fluids

> 300mOsm/L

Hyperkalemia

> 5 mEq/L

A client is having a bone marrow biopsy and is extremely anxious. What action by the nurse is best? A. assess client fears B. reassure the client this is a common test C. sedate the client prior to the procedure D. tell the client he or she will be asleep

A

A client presents to the emergency department in sickle cell crisis. What intervention by the nurse takes priority? A. administer oxygen B. apply an oximetry probe C. give pain medication D. start an IV line

A

A nurse is assessing a dark-skinned client for pallor. What action is best? A. assess the conjunctiva of the eye B. have the client open the hand widely C. look at the roof of the clients mouth D. palpate for areas of mild swelling

A

A nurse is caring for four clients with leukemia. After hand-off report, which client should the nurse see first? A. client who had two blood diarrhea stools this morning B. client who has been premeditated for nausea prior to chemotherapy C. client with a respiratory rate change from 18 to 22 breaths per min D. client with an unchanged lesion to the lower right lateral malleolus

A

A nurse prepares to discharge a client who has a wound and is prescribed home health care. Which information should the nurse include in the hand-off report to the home health nurse? A. recent wound assessment, including size and appearance B. insurance information for billing and coding purposes C. complete health history and physical assessment findings D. resources available to the client for wound care supplies

A

A nurse who manages client placements prepares to place four clients on a medical-surgical unit. Which client should be placed in isolation awaiting possible diagnosis of infection with methicillin-resistant Staphylococcus aureus(MRSA)? a.Client admitted from a nursing home with furuncles and folliculitisb. b. Client with a leg cut and other trauma from a motorcycle crash c.Client with a rash noticed after participating in sporting events d.Client transferred from intensive care with an elevated white blood cell count

A

A nurse works in an allergy clinic. What task performed by the nurse takes priority? a. Checking emergency equipment each morning b. Ensuring informed consent is obtained as needed c. Providing educational materials in several languages d. Teaching clients how to manage their allergies

A

An emergency room nurse assess a client who was rescued from a home fire. The client suddenly develops a loud, brassy cough. Which action should the nurse take first? A. apply oxygen and continuous pulse oximetry B. provide small quantities of ice chips and sips of water C. request a prescription for an antitussive medication D. ask the respiratory therapist to provide humidified air

A

The nurse is performing a 12 lead ECG on a pt with chest pain. because the positioning of the electrodes is crucial, how does the nurse place the ECG electrodes? A. four leads are placed on the limbs and six are placed on the chest B. the negative electrode is placed on the right leg C. four leads are placed on the limbs and four are placed on the chest D. the negative electrode is placed on the right arm and the positive electrode is placed on the left leg

A

The pt's ECG rhythm strip is irregular. Which method does the nurse use for an accurate assessment? a. 6-second strip method b. Memory method c. Big block method d. Commercial ECG rate ruler

A

Which test is the best tool for diagnosing heart failure? A. echocardiography B. pulmonary artery catheter C. radionuclide studies D. mitigated angiographic (MUGA) scan

A

a client appears dyspneic, but the O2 sat is 97% what action by the nurse is best? A. assess for other manifestations of hypoxia B. change the sensor on the pulse oximeter C. obtain a new oximeter from central supply D. tell the client to take slow, deep breaths

A

a client has been brought to the ED after being shot multiple times. what action should the nurse perform first? A. apply personal protective equipment B. notify local law enforcement officials C. obtain universal donor blood D. prepare the client for emergency surgery

A

a client has been diagnosed with a very large PE and has dropping blood pressure. what medication should the nurse anticipate the client will need as the priority? A. alteplase (activase) B. enoxaparin (lovenox) C. unfractionated heparin D. warfarin sodium (coumadin)

A

a client in shock is apprehensive and slightly confused. what action by the nurse is best? A. offer to remain with the client for awhile B. prepare to administer antianxiety med C. raise all four siderails on the clients bed D. tell the client everything possible is being done

A

a client in the ED has several broken ribs. what care measure will best promote comfort? A. allowing the client to choose the position in bed B. humidifying the supplemental oxygen C. offering frequent, small drinks of water D. providing warmed blankets

A

the nurse is caring for a terminally ill cancer pt who is near death. the pt reports an uncomfortable feeling of breathlessness. which therapy is the nurse most likely to administer A. 5mg of morphine B. 10 mg of lasix C. 2 L of O2 via nasal cannula D. albuterol via a metered-dose inhaler

A

The female pt is receiving isoniazid (INH) to treat TB. Which teaching points are essential for the nurse to review with the pt? SELECT ALL THAT APPLY a. do not take meds such as Maalox with this medication b. avoid drinking alcoholic beverages c. the urine will be orange in color d. take a multivitamin and B complex e. if going out in the sun, be sure to wear protective clothing and sunscreen f. this drug reduces the effectiveness of oral contraceptives

A B D

Benign cells result from ?

A small problem in cellular regulation

A nurse cares for a client with burn injuries during the resuscitation phase. Which actions are priorities during this phase? Select all that apply. A. administer analgesics B. prevent wound infections C. provide fluid replacement D. decrease core temperature E. initiate physical therapy

A, B, C

a client with a new PE is anxious. what nursing actions are most appropriate? select all that apply A. acknowledge the frightening nature of the illness B. delegate a back rub to the unlicensed assistive personnel (UAP) C. request a prescription for antianxiety medication E. stay with the client and speak in a quiet, calm voice

A, B, C, E

a nurse is caring for a client who is on mechanical ventilation. what actions will promote comfort in this client? select all that apply A. allow visitors at the clients bedside B. ensure the client can communicate if awake C. keep the TV turned to a favorite channel D. provide back and hand massages when turning E. turn the client every 2 hours or move

A, B, D, E

a nursing student studying acute coronary syndromes learns that the pain of a MI differs from stable angina in what ways? select all that apply A. accompanied by SOB B. feelings of fear or anxiety C. lasts less than 15 minutes D. no relief from taking nitroglycerin E. pain occurs without known cause

A, B, D, E

A nurse cares for a client with burn injuries who is experiencing anxiety and pain. Which nonpharmacological comfort measures should the nurse implement? Select all that apply. A. music as a distraction B. tactile stimulation C. massage to injury sites D. cold compresses E. increasing client control

A, B, E

A nurse cares for older adult clients in an long-term acute care facility. Which interventions should the nurse implement to prevent skin breakdown in these clients? select all that apply A. use a lift sheet when moving the client in bed B. avoid tape when applying dressings C. avoid whirlpool therapy D. use loose dressing on all wounds E. implement pressure-relieving devices

A, B, E

an ED nurse moves to a new city where heat related illnesses are common. which clients does the nurse anticipate being at higher risk for heat related illnesses? select all that apply. A. homeless individuals B. illicit drug users C. white people D. hockey players E. older adults

A, B, E

the nurse is caring for a client with suspected severe sepsis. what does the nurse prepare to do within 3 hours of the client being identified as being at risk? select all that apply A. administer antibiotics B. draw serum lactate levels C. infuse vasopressors D. measure central venous pressure E. obtain blood cultures

A, B, E

An older client asks the nurse why people my age have weaker immune system than younger people. What responses by the nurse are best? Select all that apply A. bone marrow produces fewer blood cells B. you may have decreased levels of circulating platelets C. you have lower levels of plasma proteins in the blood D. lymphocytes become more reactive to antigens E. spleen function declines after age 60

A, C

The student nurse studying shock understands that the common manifestations of this condition are directly related to which problems? (Select all that apply.) a. Anaerobic metabolism b. Hyperglycemia c. Hypotension d. Impaired renal perfusion e. Increased perfusion

A, C

the nursing student is studying hypersensitivitiy reactions. which reactions are correctly matched with their hypersensitivity types? Select all that apply A. Type 1 examples include hay fever and anaphylaxis B. type 2 medicated by action of immunoglobulin (IgM) C. type 3 immune complex deposits in blood vessel walls D. type 4 examples are poison ivy and transplant rejection E. type 5 examples include a positive tuberculosis test and sarcoidosis

A, C, D

A nurse plans care for a client with burn injuries. Which interventions should the nurse inclued in this clients plan of care to ensure adequate nutrition? Select all that apply. A. provide at least 5000 kcal/day B. start an oral diet on the first day C. administer a diet high in protein D. collaborate with a registered dietitian E. offer frequent high-calorie snacks

A, C, D, E

A nurse working with clients with sickle cell disease (SCD) teaches about self management to prevent exacerbations and sickle cell crises. What factors should clients be taught to avoid? select all that apply. A. Dehydration B. exercise C. extreme stress D. high altitudes E. pregnancy

A, C, D, E

A student studying leukemias learns the risk factors for developing this disorder. Which risk factors does this include? Select all that apply. A. chemical exposure B. genetically modified foods C. Ionizing radiation exposure D. vaccinations E. viral infections

A, C, E

A nurse cares for a client who reports pain related to eczematous dermatitis. Which nonpharmacologic comfort measures should the nurse implement? select all that apply. A. cool, moist compresses B. topical corticosteroids C. heating pad D. tepid bath with cornstarch E. back rub with baby oil

A, D

a nurse triages clients arriving at the hospital after a mass casualty. which clients are correctly classified? select all that apply A. 35 year old female with severe chest pain: red tag B. a 42 year old male with full thickness body urns: green tag C. a 55 year old female with a scalp laceration: black tag D. a 60 year old male with an open fracture with distal pulses: yellow tag E. an 88 year old male with shortness of breath and chest bruises: green tag

A, D

an africian american male is being seen for a blister on the right toe, what factors increase this pts risk for developing atherosclerosis? select all that apply A. 20 ear history of type 1 diabetes B sedentary lifestyle C. father with history of colon cancer D. 35 lb overweight E. grandmother who died after myocardial infarction F. drinking 2-3 diet sodas per day

A,B,D,E

a pt who was admitted for newly diagnosed heart failure is now being discharged the nurse instructs the pt and family on how to manage heart failure at home. what major self-management categories should the nurse include? select all that apply A. medications B. weight C. heart transplants D. activity E. diet F. what to do when symptoms get worse

A,B,D,E,F

the nurse is giving a community presentation about heart disease in women. what information does the nurse include in the presentation. select all that apply A. dyspnea on exertion may be the first and only symptom of heart failure B. symptoms are subtle or atypical C. pain is often relived by rest D. having a waist and abdominal obesity is a higher risk factor than having fat in buttocks and thighs E. pain always responds to nitro F. common symptoms include back pain, indigestion, nausea, vomiting, and anorexia

A,B,D,F

which bp readings require further assessment. select all that apply A. 90 systolic B. 139 systolic C. 115 systolic D. 66 diastolic E. 100 diastolic F. 96 diastolic

A,B,E,F

the nurse is assessing a pts nicotine dependence. which questions does the nurse ask for an accurate assessment? select all that apply A. how soon after you wake up in the morning do you smoke B. what kind of cigarettes do you smoke C. do you wake up in the middle of the night to smoke D. do you find it difficult not to smoke in places where smoking is prohibited E. do you smoke when you are ill F. what hapened the last time you tried to quit smoking

A,C,D,E

the pt has a diagnosis of angina. which assessment data would the nurse expect to find? select all that apply A. sudden onset of pain B. intermittent pain relieved with sitting upright C. substernal pain that may spread across chest, back and arms D. pain usually last less than 15 minutes E. sharp, stabbing pain that is moderate to severe F. pain releaved with rest

A,C,D,F

which are risk factors for cardiovascular disease (CVD) in women? select all that apply A. waist and abdominal obesity B. excess fat in the buttocks, hips and thighs C postmenopausal D. diabetes mellitus E. asian ethnicity F. elevated homocysteine level

A,C,D,F

8. The nurse is teaching a hospitalized client who is being discharged about how to care for a peripherally inserted central catheter (PICC) line. Which client statement indicates a need for further education? A. "I can continue my 20-mile (32-km) running schedule as I have for the past 10 years." B."I can still go about my normal activities of daily living." C."I have less chance of getting an infection because the line is not in my hand." D."The PICC line can stay in for months."

A. "I can continue my 20-mile (32-km) running schedule as I have for the past 10 years." The statement by the client stating that his or her normal running schedule can continue indicates a need for further education. Excessive physical activity can dislodge the PICC and should be avoided.Clients with PICCs should be able to perform normal activities of daily living. PICCs have low complication rates because the insertion site is in the upper extremity. The dry skin of the arm has fewer types and numbers of microorganisms, leading to lower rates of infection. PICC lines can be used long term (months).

Which nursing interventions apply to pts with hypercalcemia? Select all that apply: A. Administer IV NS B. Measure the abdominal girth C. Massage calves to encourage blood return to heart D. Monitor for ECG changes E. Provide adequate intake of Vitamin D F. During treatment, monitor for tetany

A. Administer IV NS B. Measure the abdominal girth D. Monitor for ECG changes

Which are appropriate interventions for a pt who has hypercalcemia? Select all that apply: A. Administer IV NS B. Administer hydrochlorothiazide HCTZ C. Ensure adequate hydration D. Administer calcium-based antacid for GI upset E. Discourage weight-bearing activity such as walking F. Provide continuous cardiac monitoring

A. Administer IV NS C. Ensure adequate hydration F. Provide continuous cardiac monitoring

A pt with renal failure that results in hypernatremia will require which interventions? Select all that apply A. Administration of furosemide B. Hemodialysis C. IV infusion of 0.9% sodium chloride D. Dietary sodium restriction E. Administration of potassium supplement F. Administration of democlocycline.

A. Administration of furosemide B. Hemodialysis D. Dietary sodium restriction

10. A client is being admitted to the burn unit from another hospital. The client has an intraosseous IV that was started 2 days ago, according to the client's medical record. What does the admitting nurse do first? A. Anticipate an order to discontinue the intraosseous IV and start an epidural IV. B. Call the previous hospital to verify the date. C. Immediately discontinue the intraosseous IV. D. Nothing; this is a long-term treatment.

A. Anticipate an order to discontinue the intraosseous IV and start an epidural IV. The admitting nurse would first anticipate an order to discontinue the intraosseous IV and start an epidural IV. The intraosseous route should be used only during the immediate period of resuscitation and should not be used for longer than 24 hours. Alternative IV routes, such as epidural access, should then be considered for pain management.The nurse should know what to do in this client's situation without contacting the previous hospital. Other client data, such as the date and time that the burn occurred, should validate the date and time of insertion of the IV. Discontinuing the IV is not the priority in this situation—the client is in a precarious fluid balance situation. One IV access should not be stopped until another is established. This type of IV is not used for long-term therapy; an action must be taken.

Which nursing interventions are implemented when caring for a patient with an implanted port? A. Before puncture, palpate the port to locate the septum. B. Use a large-bore needle to access the port. C. Flush the port before each use. D. Use a non-coring needle to access the port. E. Flush the port at least once a month

A. Before puncture, palpate the port to locate the septum.

The nurse assesses an acidotic pt's lower extremities for strength as part of the nursing shift assessment. What finding does the nurse expect to see? A. Bilateral weakness B. Weakness on the dominant side C. No change from baseline D. Cramping, but no weakness

A. Bilateral weakness

The nurse is assessing a pt with severe hypermagnesemia. Which assessment findings are associated with this electrolyte imbalance? A. Bradycardia and hypotension B. Tachycardia and weak palpable pulse C. Hypertension and irritability D. Irregular pulse and deep respirations

A. Bradycardia and hypotension

Which conditions cause a pt to be at risk for hypocalcemia? Select all that apply: A. Chrons Disease B. Acute pancreatitis C. Removal or destruction of parathyroid D. Immobility E. Use of digitalis F. GI wound drainage

A. Chrons Disease B. Acute pancreatitis C. Removal or destruction of parathyroid D. Immobility F. GI wound drainage

The provider has ordered therapy for a patient with low sodium and signs of hypervolemia? Which diauretic is best for this patient? A. Conivaptan B. Furosemide C. Hydrochlorothiazide D. Bumetanide

A. Conivaptan

A patient has been on prolonged steroid therapy. In assessing the patient for IV insertion, what finding does the nurse expect to see? A. Ecchymosis and possibly a hematoma B. Skin that is thick, tough, dry, and difficult to puncture C. Edema or puffiness, making visualization of veins difficult D. Rash with excoriation from scratching, which limits site selection

A. Ecchymosis and possibly a hematoma

A pts potassium level is high secondary to kidney failure. What laboratory changes does the nurse expect to see? Select all that apply: A. Elevated serum creatinine B. Decreased blood pH C. Elevated sodum D. Low to normal Hct E. Elevated Hgb F. Decreased BUN

A. Elevated serum creatinine B. Decreased blood pH D. Low to normal Hct

When using an intermittent administration set to deliver medications, how often does the Infusion Nurses Society recommend that the set be changed? A. Every 24 hours B. Every shift C. Every morning D. After every dose

A. Every 24 hours

Pts with which conditions are at greatest risk for deficient fluid volume? Select all that apply: A. Fever of 103 F (39.4( B. Extensive burns C. Thyroid crisis D. Water intoxication E. Continuous fistula drainage F. Diabetes insipidus

A. Fever of 103 F (39.4( B. Extensive burns C. Thyroid crisis E. Continuous fistula drainage F. Diabetes insipidus

A pt with CHF is receiving a loop diuretic. The nurse monitors for which electrolyte imbalance? Select all that apply: A. Hypocalcemia B. Hypercalcemia C. Hyponatremia D. Hypernatremia E. Hypokalemia F. Hyperkalemia

A. Hypocalcemia C. Hyponatremia E. Hypokalemia

A pt shows a positive Trosseau's or Chvostek's sign. The nurse prepares to give the pt which urgent treatment? A. IV Calcium B. Calcitonin C. IV potassium chloride D. Large doses of oral calcium

A. IV Calcium

A patient has a low potassium level, and the provider has ordered an IV infusion. Before starting an IV potassium infusion, what does the nurse assess? A. IV line patency B. Oxygen saturation level C. Baseline mental status D. Apical pulse

A. IV line patency

Which are major causes of hypomagnesemia? Select all that apply: A. Inadequate intake of magnesium B. Inadequate intake of sodium C. Use of potassium-sparing diuretics D. Decreased kidney excretion of magnesium E. Prescription of loop diuretics F. Cessation of alcohol intake

A. Inadequate intake of magnesium E. Prescription of loop diuretics

Which is a preventative measure for pts at risk for developing hypocalcemia? A. Increase daily dietary calcium and vitamin D intake B. Increase intake oh phosphorous C. Apply sunblock and wear protective clothing whenever outdoors D. Administer calcium-containing IV fluids to pts receiving multiple blood transfusions

A. Increase daily dietary calcium and vitamin D intake

A pt has hyperkalemia resulting from dehydration. Which additional laboratory findings does the nurse anticipate for this pt? A. Increased hematocrit and Hgb levels B. Decreased serum electrolyte levels C. Increased urine potassium levels D. Decreased serum creatinine

A. Increased hematocrit and Hgb levels

Which signs and symptoms would the nurse expect to assess in a pt with metabolic acidosis? Select all that apply: A. Kussmaul respirations B. Shallow, rapid respirations C. Warm, flushed skin D. Skin pale to cyanotic E. Elevated PaCO2 F. Decreased bicarbonate

A. Kussmaul respirations C. Warm, flushed skin F. Decreased bicarbonate

What are the consequences for a patient who does not meet the obligatory urine output? Select all that apply: A. Legal electrolyte imbalances B. Alkalosis C. Urine becomes diluted D. Toxic buildup of nitrogen E. Urine output increases F. Acidosis

A. Legal electrolyte imbalances D. Toxic buildup of nitrogen F. Acidosis

What impacts does sodium have on body function? Select all that apply: A. Maintains electroneutrality B. Maintains electrical membrane excitability C. Aids in carbohydrate and lipid metabolism D. Regulates water balance E. Low sodium stimulates secretion of aldosterone F. Regulates plasma osmlality

A. Maintains electroneutrality D. Regulates water balance E. Low sodium stimulates secretion of aldosterone F. Regulates plasma osmlality

6. A client who takes corticosteroids daily for rheumatoid arthritis requires insertion of an IV catheter to receive IV antibiotics for 5 days. Which type of IV catheter does the nurse teach the new graduate nurse to use for this client? A.Midline catheter B.Tunneled percutaneous central catheter C.Peripherally inserted central catheter D.Short peripheral catheter

A. Midline catheter For a client with fragile veins (which occur with long-term corticosteroid use) and the need for a catheter for 5 days, the midline catheter is the best choice.Tunneled central catheters usually are used for clients who require IV access for longer periods. Peripherally inserted central catheters usually are used for clients who require IV access for longer periods. A short peripheral catheter is likely to infiltrate before 5 days in a client with fragile veins, requiring reinsertion.

16. A client admitted to the intensive care unit is expected to remain for 3 weeks. The nurse has orders to start an IV. Which vascular access device is best for this client? A.Midline catheter B.Peripherally inserted central catheter (PICC) C.Short peripheral catheter D.Tunneled central catheter

A. Midline catheter Midline catheters are the best device for this client. These catheters are used for therapies lasting from 1 to 4 weeks.PICCs are typically used when IV therapy is expected to last for months. Short peripheral catheters are allowed to dwell (stay in) for 72 to 96 hours, but they then require removal and insertion at another venous site. Tunneled central catheters must be inserted by a health care provider. Nurses are typically not qualified to start tunneled central catheters.

The nurse is working in a long-term care facility where there are numerous patients who are immobile and at risk for dehydration. Which task is best to delegate to the unlicensed assistive personnel? A. Offer pts a choice of fluids every 1 hour B. Check pts at the beginning of the shift to see who is thirsty C. Give pts extra fluids around medication times

A. Offer pts a choice of fluids every 1 hour

The nurse is preparing to start an infusion of 10% dextrose. Why would the nurse infuse the solution through a central line? A. Osmolartiy of the solution could cause phlebitis or thrombosis. B. The patient could be at risk for fluid overload. C. Viscosity of the solution would slow the infusion. D. The solution should not be mixed with other drugs or solutions.

A. Osmolartiy of the solution could cause phlebitis or thrombosis.

Which statements correctly apply to acid-base balance in the body. Select all that apply: A. Renal mechanisms are stronger in regulating acid-base balance but slower to respond than respiratory mechanisms. B. The immediate binding of excess hydrogen ions occurs primarily in the RBC's. C. Combined acidosis is less severe than either metabolic acidosis or respiratory acidosis alone. D. Respiratory acidosis is caused by a patent airway E. Acid-base balance occurs through control of hydrogen ion production and elimination. F. Buffers are third-line defense against acid-base imbalances in the body

A. Renal mechanisms are stronger in regulating acid-base balance but slower to respond than respiratory mechanisms. B. The immediate binding of excess hydrogen ions occurs primarily in the RBC's. E. Acid-base balance occurs through control of hydrogen ion production and elimination.

Which conditions could cause a patient to develop acidosis? Select all that apply: A. Sepsis B. Hypovolemic shock C. Use of a mechanical ventilator D. Prolonged nasogastric suctioning E. Hypoventiliation F. Severe diarrhea

A. Sepsis B. Hypovolemic shock E. Hypoventiliation

Which serum value does the nurse expect to see for a patient with hyponatremia? A. Sodium less than 136 mEq/L B. Chloride less than 95 mEq/L C. Sodium less than 145 mEq/L D. Chloride less than 103 mEq/L

A. Sodium less than 136 mEq/L

The nurse is teaching a patient with hypokalemia about foods high in potassium? Which food items does the nurse recommend to this patient? A. Soybeans B. Lettuce C. Cantaloupe D. Potatoes E. Peaches F. Bananas

A. Soybeans C. Cantaloupe D. Potatoes F. Bananas

Which changes on a pts ECG reflect hyperkalemia? A. Tall peaked T waves B. Narrow peaked T waves C. Tall P waves D. Normal P-R interval

A. Tall peaked T waves

The pt who has undergone which surgical procedure is most at risk for hypocalcemia? A. Thyroidectomy B. Adrenalectomy C. Pancreatectomy D. Gastrectomy

A. Thyroidectomy

Which items does the nurse include in the documentation after completing the insertion of a PICC? A. Type of dressing applied B. Response of the family to IV access C. Type of IV access device used D. How long it took to place the IV access E. Location and vein that was used for insertion

A. Type of dressing applied C. Type of IV access device used E. Location and vein that was used for insertion

A pt with bilateral lower lobe pneumonia is diagnosed with respiratory acidosis based on ABG results. What is the likely cause of the pt's respiratory acidosis? A. Under-elimination of carbon dioxide from the lungs B. Buffering of ECF by ammonium C. Over-elimination of carbon dioxide from the lungs D. An increased bicarbonate level due to respiratory elimination of acid

A. Under-elimination of carbon dioxide from the lungs

Which medication usage could cause metabolic acidosis? A. aspirin overdose B. Overuse of antacids C. Prolonged use of antihistamines D. Vitamin overdose

A. aspirin overdose

A pt with anemia has completed a blood transfusion of 2 units of packed RBC's. Which imbalance should the nurse monitor for after the blood transfusion? A. metabolic alkalosis B. Respiratory acidosis C. Metabolic acidosis D. respiratory alkalosis

A. metabolic alkalosis

Which ABG results indicate that a pt's acid-base imbalance is a respiratory acidosis? Select all that apply: A. pH 7.31 B. PaCO2 58 mm Hg C. Bicarbonate 17 mEq/L D. PaO2 75 mm Hg E. Serum potassium 4.5 mEq/L F. PaCO2 31 mm Hg

A. pH 7.31 B. PaCO2 58 mm Hg D. PaO2 75 mm Hg

A pt is admitted to the hospital for diabetic ketoacidosis. Which ABG results should the nurse expect? Select all that apply: A. pH7.32 B. PaCo2 55mm Hg C. Bicarbonate 18 mEq/L D. pH 7.46 E. Bicarbonate 29 mEq/L F. PaCO2 44 mm Hg

A. pH7.32 C. Bicarbonate 18 mEq/L

A client asks the nurse if eating only preservative- and dye-free foods will decrease cancer risk. What response by the nurse is best? a. "Maybe; preservatives, dyes, and preparation methods may be risk factors." b. "No; research studies have never shown those things to cause cancer." c. "There are other things you can do that will more effectively lower your risk." d. "Yes; preservatives and dyes are well known to be carcinogens."

ANS: A Dietary factors related to cancer development are poorly understood, although dietary practices are suspected to alter cancer risk. Suspected dietary risk factors include low fiber intake and a high intake of red meat or animal fat. Preservatives, preparation methods, and additives (dyes, flavorings, sweeteners) may have cancer-promoting effects. It is correct to say that other things can lower risk more effectively, but this does not give the client concrete information about how to do so, and also does not answer the client's question.

A nursing student learning about antibody-mediated immunity learns that the cell with the most direct role in this process begins development in which tissue or organ? a. Bone marrow b. Spleen c. Thymus d. Tonsils

ANS: A The B cell is the primary cell in antibody-mediated immunity and is released from the bone marrow. These cells then travel to other organs and tissues, known as the secondary lymphoid tissues for B cells.

The student nurse is learning about the functions of different antibodies. Which principles does the student learn? (Select all that apply.) a. IgA is found in high concentrations in secretions from mucous membranes. b. IgD is present in the highest concentrations in mucous membranes. c. IgE is associated with antibody-mediated hypersensitivity reactions. d. IgG comprises the majority of the circulating antibody population. e. IgM is the first antibody formed by a newly sensitized B cell.

ANS: A, C, D, E Immunoglobulin A (IgA) is found in high concentrations in secretions from mucous membranes. Immunoglobulin E (IgE) is associated with antibody-mediated hypersensitivity reactions. The majority of the circulating antibody population consists of immunoglobulin G (IgG). The first antibody formed by a newly sensitized B cell is immunoglobulin M (IgM). Immunoglobulin D (IgD) is typically present in low concentrations.

A nurse assesses a client who is experiencing diabetic ketoacidosis (DKA). For which manifestations should the nurse monitor the client? (Select all that apply.) a. Deep and fast respirations b. Decreased urine output c. Tachycardia d. Dependent pulmonary crackles e. Orthostatic hypotension

ANS: A, C, E DKA leads to dehydration, which is manifested by tachycardia and orthostatic hypotension. Usually clients have Kussmaul respirations, which are fast and deep. Increased urinary output (polyuria) is severe. Because of diuresis and dehydration, peripheral edema and crackles do not occur.

The nurse assesses clients for the cardinal signs of inflammation. Which signs/symptoms does this include? (Select all that apply.) a. Edema b. Pulselessness c. Pallor d. Redness e. Warmth

ANS: A, D, E The five cardinal signs of inflammation include redness, warmth, pain, swelling, and decreased function.

A nurse assesses clients at a health fair. Which clients should the nurse counsel to be tested for diabetes? (Select all that apply.) a. 56-year-old African-American male b. Female with a 30-pound weight gain during pregnancy c. Male with a history of pancreatic trauma d. 48-year-old woman with a sedentary lifestyle e. Male with a body mass index greater than 25 kg/m2 f. 28-year-old female who gave birth to a baby weighing 9.2 pounds

ANS: A, D, E, F Risk factors for type 2 diabetes include certain ethnic/racial groups (African Americans, American Indians, Hispanics), obesity and physical inactivity, and giving birth to large babies. Pancreatic trauma and a 30-pound gestational weight gain are not risk factors.

A nurse cares for a client who is diagnosed with acute rejection 2 months after receiving a simultaneous pancreas-kidney transplant. The client states, "I was doing so well with my new organs, and the thought of having to go back to living on hemodialysis and taking insulin is so depressing." How should the nurse respond? a. "Following the drug regimen more closely would have prevented this." b. "One acute rejection episode does not mean that you will lose the new organs." c. "Dialysis is a viable treatment option for you and may save your life." d. "Since you are on the national registry, you can receive a second transplantation."

ANS: B An episode of acute rejection does not automatically mean that the client will lose the transplant. Pharmacologic manipulation of host immune responses at this time can limit damage to the organ and allow the graft to be maintained. The other statements either belittle the client or downplay his or her concerns. The client may not be a candidate for additional organ transplantation.

A nurse is teaching a client with diabetes mellitus who asks, "Why is it necessary to maintain my blood glucose levels no lower than about 60 mg/dL?" How should the nurse respond? a. "Glucose is the only fuel used by the body to produce the energy that it needs." b. "Your brain needs a constant supply of glucose because it cannot store it." c. "Without a minimum level of glucose, your body does not make red blood cells." d. "Glucose in the blood prevents the formation of lactic acid and prevents acidosis."

ANS: B Because the brain cannot synthesize or store significant amounts of glucose, a continuous supply from the body's circulation is needed to meet the fuel demands of the central nervous system. The nurse would want to educate the client to prevent hypoglycemia. The body can use other sources of fuel, including fat and protein, and glucose is not involved in the production of red blood cells. Glucose in the blood will encourage glucose metabolism but is not directly responsible for lactic acid formation.

A nurse teaches a client with diabetes mellitus about sick day management. Which statement should the nurse include in this client's teaching? a. "When ill, avoid eating or drinking to reduce vomiting and diarrhea." b. "Monitor your blood glucose levels at least every 4 hours while sick." c. "If vomiting, do not use insulin or take your oral antidiabetic agent." d. "Try to continue your prescribed exercise regimen even if you are sick."

ANS: B When ill, the client should monitor his or her blood glucose at least every 4 hours. The client should continue taking the medication regimen while ill. The client should continue to eat and drink as tolerated but should not exercise while sick.

A client has a leg wound that is in the second stage of the inflammatory response. For what manifestation does the nurse assess? a. Noticeable rubor b. Purulent drainage c. Swelling and pain d. Warmth at the site

ANS: B During the second phase of the inflammatory response, neutrophilia occurs, producing pus. Rubor (redness), swelling, pain, and warmth are cardinal signs of the general inflammatory process.

The nurse working in an organ transplantation program knows that which individual is typically the best donor of an organ? a. Child b. Identical twin c. Parent d. Same-sex sibling

ANS: B The recipient's immune system recognizes donated tissues as non-self except in the case of an identical twin, whose genetic makeup is identical to the recipient.

A nurse is participating in primary prevention efforts directed against cancer. In which activities is this nurse most likely to engage? (Select all that apply.) a. Demonstrating breast self-examination methods to women b. Instructing people on the use of chemoprevention c. Providing vaccinations against certain cancers d. Screening teenage girls for cervical cancer e. Teaching teens the dangers of tanning booths

ANS: B, C, E Primary prevention aims to prevent the occurrence of a disease or disorder, in this case cancer. Secondary prevention includes screening and early diagnosis. Primary prevention activities include teaching people about chemoprevention, providing approved vaccinations to prevent cancer, and teaching teens the dangers of tanning beds. Breast examinations and screening for cervical cancer are secondary prevention methods.

A nurse assesses a client who is being treated for hyperglycemic-hyperosmolar state (HHS). Which clinical manifestation indicates to the nurse that the therapy needs to be adjusted? a. Serum potassium level has increased. b. Blood osmolarity has decreased. c. Glasgow Coma Scale score is unchanged. d. Urine remains negative for ketone bodies.

ANS: C A slow but steady improvement in central nervous system functioning is the best indicator of therapy effectiveness for HHS. Lack of improvement in the level of consciousness may indicate inadequate rates of fluid replacement. The Glasgow Coma Scale assesses the client's state of consciousness against criteria of a scale including best eye, verbal, and motor responses. An increase in serum potassium, decreased blood osmolality, and urine negative for ketone bodies do not indicate adequacy of treatment.

A nurse assesses a client who has a 15-year history of diabetes and notes decreased tactile sensation in both feet. Which action should the nurse take first? a. Document the finding in the client's chart. b. Assess tactile sensation in the client's hands. c. Examine the client's feet for signs of injury. d. Notify the health care provider.

ANS: C Diabetic neuropathy is common when the disease is of long duration. The client is at great risk for injury in any area with decreased sensation because he or she is less able to feel injurious events. Feet are common locations for neuropathy and injury, so the nurse should inspect them for any signs of injury. After assessment, the nurse should document findings in the client's chart. Testing sensory perception in the hands may or may not be needed. The health care provider can be notified after assessment and documentation have been completed.

The nurse understands that which type of immunity is the longest acting? a. Artificial active b. Inflammatory c. Natural active d. Natural passive

ANS: C Natural active immunity is the most effective and longest acting type of immunity. Artificial and natural passive do not last as long. "Inflammatory" is not a type of immunity.

A nurse teaches a client with diabetes mellitus who is experiencing numbness and reduced sensation. Which statement should the nurse include in this client's teaching to prevent injury? a. "Examine your feet using a mirror every day." b. "Rotate your insulin injection sites every week." c. "Check your blood glucose level before each meal." d. "Use a bath thermometer to test the water temperature."

ANS: D Clients with diminished sensory perception can easily experience a burn injury when bathwater is too hot. Instead of checking the temperature of the water by feeling it, they should use a thermometer. Examining the feet daily does not prevent injury, although daily foot examinations are important to find problems so they can be addressed. Rotating insulin and checking blood glucose levels will not prevent injury.

After teaching a client who has diabetes mellitus and proliferative retinopathy, nephropathy, and peripheral neuropathy, the nurse assesses the client's understanding. Which statement made by the client indicates a correct understanding of the teaching? a. "I have so many complications; exercising is not recommended." b. "I will exercise more frequently because I have so many complications." c. "I used to run for exercise; I will start training for a marathon." d. "I should look into swimming or water aerobics to get my exercise."

ANS: D Exercise is not contraindicated for this client, although modifications based on existing pathology are necessary to prevent further injury. Swimming or water aerobics will give the client exercise without the worry of having the correct shoes or developing a foot injury. The client should not exercise too vigorously.

The nurse working with clients who have autoimmune diseases understands that what component of cell-mediated immunity is the problem? a. CD4+ cells b. Cytotoxic T cells c. Natural killer cells d. Suppressor T cells

ANS: D Suppressor T cells help prevent hypersensitivity to one's own cells, which is the basis for autoimmune disease. CD4+ cells are also known as helper/inducer cells, which secrete cytokines. Natural killer cells have direct cytotoxic effects on some non-self cells without first being sensitized. Suppressor T cells have an inhibitory action on the immune system. Cytotoxic T cells are effective against self cells infected by parasites such as viruses or protozoa.

Aneuploidy is?

Abnormal amount of chromosomes

Example of chemical carcinogens include?

Alcohol and tobacco

Mailgnant cells appearance is?

Anaplastic - meaning cells lost specific appearance from parenting cell

Species Resistance

Animals get some protection through their genetic make-up

Disease

Any alteration from normal healthy state of being

Fast growing tumor has a mitotic index of?

Anything greater then 10 %. The higher the number the faster the tumor is growing

Grading is need b/c some tumors are?

Are more malignant then others, varying in their aggressiveness and sensitivity to treatment

A client admitted for sickle cell crisis is distraught after learning her child also has the disease. What response by the nurse is best? A. both you and the father are equally responsible for passing it on B. I can see you are upset. I can stay here with you a while if you like C. Its not your fault; there is no way to know who will have this disease D. there are many good treatments for sickle cell disease these days

B

A client has a serum ferritin level of 8 ng/mL and microcytic red blood cells. What action by the nurse is best? A. encourage high-protein foods B. perform a hemoccult test on the clients stools C. offer frequent oral care D. prepare to administer cobalamin (Vitamin B12)

B

A client having severe allergy symptoms has received several doses of IV antihistamines. What action by the nurse is most important? A. assess the clients bedside glucose reading B. instruct the client not to get up without help C. monitor the client frequently for tachycardia D. record the clients intake, output, and weight

B

A nurse assess a client admitted with deep partial-thickness and full-thickness burns on the face, arms, and chest. Which assessment finding should alert the nurse to a potential complication? A. partial pressure of arterial oxygen (PaO2) of 80 mmHg B. urine output of 20 mL/hr C. productive cough with white pulmonary secretions D. core temperature of 100.6 F (38C)

B

A nurse assess a client and identifies that the client has pallor conjunctivae. Which focused assessment should the nurse complete next? A. partial thromboplastin time B. hemoglobin and hematocrit C. liver enzymes D. basic metabolic panel

B

A nurse assesses a client who has psoriasis. Which action should the nurse take first? A. don gloves and an isolation gown B. shake the clients hands and introduce self C. assess for signs and symptoms of infections D. ask the client if she might be pregnant

B

After teaching a client who is at risk for the formation of pressure ulcers, a nurse assess the clients understanding. Which dietary choice by the client indicates a good understanding of the teaching? A. low-fat diet with whole grains and cereals and vitamin supplements B. high-protein diet with vitamins and mineral supplements C. vegetarian diet with nutritional supplements and fish oil capsules D. low-fat, low-cholesterol, high-fiber, low-carbohydrate diet

B

During skin inspection of a client, a nurse observes lesions with wavy borders that are widespread across the client's chest. Which descriptors should the nurse use to document these observations? a. Clustered and annular b. Linear and circinate c. Diffuse and serpiginous d. Coalesced and circumscribed

B

The family of a neutropenic client reports the client "is not acting right." What action by the nurse is the priority? A. ask the client about pain B. assess the client for infection C. delegate taking a set of vital signs D. look at todays lab results

B

The nurse is caring for a client with an acute burn injury. Which action should the nurse take to prevent infection by autocontamination? A. use a disposable blood pressure cuff to avoid sharing with other clients B. change gloves between wound care on different parts of the clients body C. use the closed method of burn wound management for all wound care D. advocate for proper and consistent handwashing by all members of the staff

B

When is B-type natriuretic peptide (BNP) produced and released for a patient with heart failure? A. when a pt has an enlarged liver B. when a pt has fluid overload C. when a pts ejection fraction is lower than normal D. when a pt has ventricular hypertrophy

B

While assessing a client, a nurse detects a bluish tinge to the clients palms, soles, and mucous membranes, which action should the nurse take next? A. ask the client about current medications he or she is taking B. use pulse oximetry to assess the clients oxygen saturation C. auscultate the clients lung fields for adventitious sounds D. palpate the clients bilateral radial and pedal pulses

B

a client arrives in the ED after being in a car crash with fatalities. the client has a nearly amputated leg that is bleeding profusely. what action by the nurse takes priority? A. apply direct pressure to the bleeding B. ensure the client has a patent airway C. obtain consent for emergency surgery D. start two large-bore IV catheters

B

while assessing a clients lower extremities, a nurse notices that one leg is pale and cooler to the touch. Which assessment should the nurse perform next? A. ask about a family history of skin disorders B. palpate the clients pedal pulses bilaterally C. check for the presence of homans sign D. assess the clients skin for adequate skin turgor

B

a nurse is caring for clients in a busy ED which actions should the nurse take to ensure client and staff safety? select all that apply. A. leave the stretcher in the lowest position with rails down so that the client can access the bathroom B. use 2 identifiers before each intervention and before medication administration C. attempt de-escalation strategies for clients who demonstrate aggressive behaviors D. search the belongings of clients with altered mental status to gain essential medical information E. isolate clients who have immune suppression disorders to prevent hospital acquired infections

B, C, D

A client in the family practice clinic reports a 2-week history of an allergy to something. The nurse obtains the following assessment and lab data: physical assessment data lab results reports sore throat, runny nose, headache, posterior pharynx is reddened, nasal discharge is seen in the back of the throat, nasal discharge is creamy yellow in color, temperature 100.2F (37.9 C), red watery eyes, WBC count: 13,400/mm3, eosinophil count: 11.5%, neutrophil count: 82% About what medications and interventions does the nurse plan to teach this client? select all that apply A. elimination of any pets B. chlorpheniramine (chlor-trimation) C. future allergy scratch testing D. proper use of decongestant nose sprays E. taking the full dose of antibiotics

B, C, D, E

an emergency room nurse is caring for a trauma client. which interventions should the nurse perform during the primary survey? select all that apply A. Foley catherization B. needle decompression C. initiating IV fluids D. splinting open fractures E. endotracheal intubation F. removing wet clothing G. laceration repair

B, C, E, F

the nurse is assessing a pt with left sided heart failure. which assessment findings does the nurse expect to see in this pt? select all that apply A. ascites B. S3 heart sound C. paroxysmal nocturnal dyspena D. jugular venous distension E. oliguria during the day F. wheezes or crackles

B, C, E, F

a hospital prepares for a mass casualty event. which functions are correctly paired with the personnel role? select all that apply A. paramedic decides the number, acuity, and resource needs of clients B. hospital incident commander assumes overall leadership for implementing the emergency plan C. public information officer provides advanced life support during transportation to the hospital D. triage officer rapidly evaluates each client to determine priorities for treatment E. medical command physician serves as a liaison between the health care facility and the media

B, D

a nurse is caring for five clients for which clients would the nurse assess a high risk for developing a PE? select all that apply A. client who had a reaction to contrast dye yesterday B. client with a new spinal cord injury on a rotating bed C. middle aged man with an exacerbation of asthma D. older client who is 1 day post hip replacement surgery E. young obese client with a fractured femur

B, D, E

a hospital prepares to receive large numbers of casualties from a community disaster. which clients should the nurse identify as appropriate for discharge or transfer to another facility? select all that apply A. older adult in the medical decision unit for evaluation of chest pain B. client who had an open reduction and internal fixation of a femur fracture 3 days ago C. client admitted last night with community-acquired pneumonia D. infant who has a fever of unknown origin E. client on the medical unit for wound care

B, E

which factors can increase systemic arterial pressure? select all that apply A. decreased cardiac output B. increased heart rate C. increased peripheral vascular resistance D. increased stroke volume E. decreased BP F. decreased stroke volume

B,C,D

the nurse educates and advises a pt to follow the DASH diet. which instructions does the nurse give to the pt? select all that apply A. consume a dietary pattern that emphasizes intake of lean protein B. consume low fat dairy products, poultry, and fish C. lower sodium intake to no more than 2400 mg per day D. engage in aerobic physical activity 6-7 times/week E. limit intake of sweets and red meats F. eat legumes, nontropical vegetable oils and nuts

B,C,E,F

what different pathophys conditions can the healthy heart adapt to? select all that apply A. menses B. stress C. GERD D infection E hemorrhage F. kidney stones

B,D,E

the nurse is assessing a 62 year old native hawaiian woman. she is post menopausal, has had diabetes for 10 years, has smoked one pack a day of cigrettes for 20 years, walks twice a week for 30 mins, is an administrator, and describes her lifestyle as sedentary. for her weight and height she has a BMI of 32. which risk factors for this pt is controllable for CAD? select all that apply. A. ethnic background B. smoking C. age D. obsity E. postmenopausal F. sedentary lifestyle

B,D,F

The nurse is teaching the pt about hypokalemia. Which statement by the pt indicates a correct understanding of the treatment of hypokalemia? A. "My wife does all the cooking. She shops for food high in calcium." B. "When I take the liquid potassium in the evening, I'll eat a snack beforehand. " C. "I will avoid bananas, orange juice, and salt substitutes". D. "I hate being struck with needles all the time to monitor how much sugar I can eat"

B. "When I take the liquid potassium in the evening, I'll eat a snack beforehand. "

On admission, a pt with pulmonary edema weighed 151 lbs; now the pts weight is 149 lbs. Assuming the pt was weighed both times with the same clothing, on the same scale, and at the same time of day, how many millimeters of fluid does the nurse estimate the pt has lost? A. 500 B. 1000 C. 2000 D. 2500

B. 1000

Which potassium levels are within normal limits? Select all that apply: A. 2.9 mmol/L B. 3.5 mmol/L C. 4.5 mmol/L D. 5.0 mmol/L E. 6.0 mmol/L

B. 3.5 mmol/L C. 4.5 mmol/L D. 5.0 mmol/L

What is the minimum amount of urine output per day needed to excrete toxic waste products? A. 200-300 mL B. 400-600 mL C. 500-1000 mL D. 1000-1500 mL

B. 400-600 mL

Which pt is at greatest risk for chronic hypocalcemia? A. 38 yr old man with chronic kidney disease B. 50 yr old postmenopausal woman C. 62 yr old man with type 2 diabetes D. 78 yr old woman with dehydration

B. 50 yr old postmenopausal woman

The nurse is reviewing the laboratory calcium level results for a patient. Which value indicates mild hypocalcemia? A. 5.0 mg/dL B. 8.0 mg/dL C. 10.0 mg/dL D. 12.0 mg/dL

B. 8.0 mg/dL

Which factors affect the amount and distribution of body fluids? Select all that apply: A. Race B. Age C. Gender D. Height E. Body fat F. Weight

B. Age C. Gender E. Body fat F. Weight

The UAP notifies the nurse that the pt with emphysema receiving oxygen at 2L via nasal cannula is short of breath after morning care. What is the nurse's first action? A. Notify the health care provider immediately B. Ask the UAP to check the pt's SaO2 level C. Instruct the UAP to check vital signs D. Document the incident in the pt's chart

B. Ask the UAP to check the pt's SaO2 level

The nurse is testing the muscle strength of a pt at risk for acid-base imbalance. Which technique does the nurse use to test arm strength? A. Asks the pt to hold the arms straight out in front, and the nurse observes for drift B. Asks the pt to squeeze the nurse's hands C. Asks the pt to pick up an object that weighs at least 10 lbs D. Tries to separate the pt's clasped hands

B. Asks the pt to squeeze the nurse's hands

The nurse caring for a pt with hypercalcemia anticipates orders for which medications? Select all that apply: A. Magnesium Sulfate B. Calcitonin C. Furosemide D. Plicamycin E. Calcium gluconate F. Aluminum hyroxide

B. Calcitonin C. Furosemide D. Plicamycin

21. A client is admitted to the cardiothoracic surgical intensive care unit after cardiac bypass surgery. The client is still sedated on a ventilator and has an arterial catheter in the right wrist. What assessment does the nurse make to determine patency of the client's arterial line? A.Blood pressure B.Capillary refill and pulse C.Neurologic function D.Questioning the client about the pain level at the site

B. Capillary refill and pulse Capillary refill and pulse should be assessed to ensure that the arterial line is not occluding the artery.Blood pressure and neurologic function are not pertinent to the client's arterial line. Although the client's comfort level is important with an arterial line, it is not a determinant of patency of the line.

The electrolyte Magnesium is responsible for which functions? Select all that apply: A. Formation of hydrochloric acid B. Carbohydrate metabolism C. Contraction of skeletal muscle D. Regulation of intracellular osmolarity E. Vitamin activation F. Blood coagulation

B. Carbohydrate metabolism C. Contraction of skeletal muscle E. Vitamin activation F. Blood coagulation

The nurse administering potassium to a pt carefully monitors the infusion because of the risk for which condition? A. Pulmonary edema B. Cardiac Dysrhythmia C. Postural hypotension D. Renal Failure

B. Cardiac Dysrhythmia

A pt with hypocalcemia needs supplemental diet therapy. Which foods does the nurse recommend, providing both calcium and vitamin D? Select all that apply: A. Tofu B. Cheese C. Eggs D. Broccoli E. Milk F. Salmon

B. Cheese E. Milk F. Salmon

What is the term for a difference in concentration of particles that is greater on one side of a permeable membrane than on the other side? A. Hydrostatic Pressure B. Concentration gradient C. Passive transport D. Active transport

B. Concentration gradient

7. The nurse checking an IV fluid order questions its accuracy. What does the nurse do first? A.Asks the charge nurse about the order B.Contacts the health care provider who ordered it C.Contacts the pharmacy for clarification D.Starts the fluid as ordered, with plans to check it later

B. Contacts the health care provider who ordered it First, the nurse contacts the health care provider who ordered it. The nurse is legally and professionally responsible for accuracy and has the duty to verify the order with the health care provider who ordered it.The nurse can consult the charge nurse, but this is not the definitive action that the nurse should take. Contacting the pharmacy is not the best action that the nurse should take. Giving (or starting) the fluid when the order is questionable is not appropriate and could possibly harm the client.

What are the functions of potassium in the body? Select all that apply: A. Regulates hydration status B. Controls intracellular osmolality and volume C. Stimulates the secretion of antidiuretic hormone ADH D. Functions as the major cation of ICF E. Regulated glucose use and storage F. Helps maintain normal cardiac rhythym

B. Controls intracellular osmolality and volume D. Functions as the major cation of ICF E. Regulated glucose use and storage F. Helps maintain normal cardiac rhythym

A pt has a new onset of shallow and slow respirations. While the pt's body attempts to compensate, what happens to the pt's pH level? A. Increases B. Decreases C. Stabilizes D. Fluctuates

B. Decreases

Which foods will the nurse instruct a patient with kidney disease and hyperkalemia to avoid? Select all that apply: A. Canned apricots B. Dried beans C. Potatoes D. Cabbage E. Cantaloupe F. Canned sausage

B. Dried beans C. Potatoes E. Cantaloupe F. Canned sausage

A pt has a low pH level. Which other concurrent change does the nurse expect to see in this patient? A. Elevated serum sodium level B. Elevated serum potassium C. Decreased serum chloride level D. Decreased serum calcium level

B. Elevated serum potassium

The nurse assessing a pt notes a bounding pulse quality, neck vein distention when supine, presence of crackles in the lungs, and increasing peripheral edema. What fluid disorder do these findings reflect? A. Fluid Volume deficit B. Fluid volume excess C. Fluid homeostasis D. Fluid dehydration

B. Fluid volume excess

The nurse is caring for a patient with a Groshong catheter. According to the manufacturer's recommendations, which technique does the nurse use in maintaining this type of catheter? A. Flush the catheter with heparin. B. Flush the catheter with saline. C. Avoid flushing the catheter. D. Aspirate the catheter to remove clots.

B. Flush the catheter with saline.

A pt is at risk for acid-balance imbalance. Which laboratory value indicates that the pt has metabolic acidosis? A. PaCO2 = 55 mm Hg B. HCO3- = 17 mEq/L C. Lactate = 2.5 mmol/L D. pH = 7.35

B. HCO3- = 17 mEq/L

What are the hallmark laboratory findings of respiratory alkalosis? A. High pH and low PaO2 B. High pH and low PaCO2 C. High pH and high bicarbonate D. High pH and high PaCO2

B. High pH and low PaCO2

A hospitalized pt who is known to be homeless has been diagnosed with severe malnutrition, end-stage renal disease, and anemia. He is transfused with three units of packed RBC's. Which potential electrolyte imbalance does the nurse anticipate could occur in this pt? A. Hypernatremia B. Hyperkalemia C. Hypercalcemia D. Hypermagnesemia

B. Hyperkalemia

A pt who has advanced muscular dystrophy may develop which complications related to the disease? select all that apply : A. Hyperventilation B. Hypoventilation C. Underproduction of bicarbonate D. Respiratory acidosis E. Under elimination of hydrogen ions F. Decreased chest wall movement

B. Hypoventilation D. Respiratory acidosis F. Decreased chest wall movement

Which statement made by the pt indicates that he or she may have an alkaline condition? A. I am more and more tired and can't concentrate B. I have tingling in my fingers and toes C. My feet and ankles are swollen D. I am short of breath all of the time

B. I have tingling in my fingers and toes

Which component has a high content of potassium and phosphorous? A. ECF B. ICF C. ECF and the Intravascular space D. ICF and lymph fluid

B. ICF

The nurse is caring for a pt with metabolic alkalosis secondary to diuretic medication. Which equipment does the nurse obtain to administer the correct therapy to the pt? A. Oxygen tubing and cannula or mask B. IV catheter and IV start kit C. Foley catheter and drainage bag D. Antiemetic drug and emesis basin

B. IV catheter and IV start kit

The nurse is caring for an older adult pt whose serum sodium level is 150 mEq/L. The nurse assesses the pt for which common signs and symptoms associated with this sodium level? Select all that apply A. Intact recall of recent events B. Increased pulse rate C. Rigidity of extremities D. Hyperactivity E. Muscle weakness F. Difficulty palpating peripheral pulses

B. Increased pulse rate E. Muscle weakness F. Difficulty palpating peripheral pulses

The nurse is admitting a patient with acute kidney injury to the medical unit. Which ABG results would she expect for this patient? A. Respiratory acidosis B. Metabolic acidosis C. Respiratory Alkalosis D. Metabolic Alkalosis

B. Metabolic acidosis

A patient who has pancreatitis with nausea and vomiting will likely have which related alterations in acid-base balance. Select all that apply: A. Overproduction of hydrogen ions B. Metabolic acidosis C. Serum pH value that is directly related to the concentration of hydrogen ions D. Underproduction of bicarbonate E. Metabolic alkalosis F. Respiratory Acidosis

B. Metabolic acidosis C. Serum pH value that is directly related to the concentration of hydrogen ions D. Underproduction of bicarbonate

A pt with hypokalemia is likely to have which conditions? Select all that apply. A. Liver failure B. Metabolic alkalosis C. Cushing's Syndrome D. Hypothyroidism E. Paralytic Ileus F. Kidney failure

B. Metabolic alkalosis C. Cushing's Syndrome E. Paralytic Ileus

The pt with a serum magnesium level fo 2.9 mEq/L develops bradycardia with a prolonged P-R interval and a widened QRS. What is the nurse's best first action? A. Start an IV with 5% dextrose with 100 mL/hr B. Notify the health care provider immediately C. Auscultate the pt's apical heart rate D. Prepare to administer supplemental magnesium by IV

B. Notify the health care provider immediately

A pt has an elevated potassium level. Which assessment findings are associated with hyperkalemia? Select all that apply: A. Wheezing on exhalation B. Numbness in hands and feet and around the mouth C. Frequent, watery stools D. Irregular heart rate E. Circumoral cyanosis F. Muscle weakness

B. Numbness in hands and feet and around the mouth C. Frequent, watery stools D. Irregular heart rate F. Muscle weakness

The patient has severe hypokalemia (2.4 mEq/L). For which intestinal complication does the nurse monitor? A. Hypoactive bowel sounds B. Paralytic Ileus C. Nausea D. Constipation

B. Paralytic Ileus

Which pts require assessment related to inadequate chest expansion that would place them at risk for respiratory acidosis? Select all that apply: A. Pt with lordosis B. Pt with emphysema C. Severly obese pt on prolonged bedrest D. Pt in the first trimester of pregnancy E. Pt 2 days postoperative for laparoscopic cholecystectomy F. Pt with ascites

B. Pt with emphysema C. Severly obese pt on prolonged bedrest F. Pt with ascites

Which patient is most likely to develop respiratory acidosis? A. Pt who is anxious and breathing rapidly B. Pt with multiple rib fractures C. Pt with IV normal saline bolus D. Pt with increased urinary output

B. Pt with multiple rib fractures

3. Which statement is true about the special needs of older adults receiving IV therapy? A.Placement of the catheter on the back of the client's dominant hand is preferred. B.Skin integrity can be compromised easily by the application of tape or dressings. C.To avoid rolling the veins, a greater angle of 25 degrees between the skin and the catheter will improve success with venipuncture. D.When the catheter is inserted into the forearm, excess hair should be shaved before insertion.

B. Skin integrity can be compromised easily by the application of tape or dressings. Skin in older adults tends to be thin. Tape or dressings used with IV therapy can compromise skin integrity.Placement on the back of the dominant hand is contraindicated because hand movement can increase the risk of catheter dislodgement. An angle smaller than 25 degrees is required for venipuncture success in older adults. This technique is less likely to puncture through the older adult client's vein. Clipping, and not shaving, the hair around the insertion site typically is necessary only for younger men.

The nurse is attempting to insert a peripheral IV when the patient reports tingling and a feeling like "pins and needles." What does the nurse do next? A. Change to a short-winged butterfly needle. B. Stop immediately, remove the catheter, and choose a new site. C. Ask the patient to wiggle the fingers to stimulate circulation. D. Pause the procedure and gently massage the fingers.

B. Stop immediately, remove the catheter, and choose a new site.

A patient has a peripherally inserted central catheter (PICC) placed and is ordered to receive IV cisplatin (Plantinol). The drug has infiltrated into the tissue and redness is observed in the right lower side of the neck. What is the nurse's first action? A. Apply cold compresses to the site of swelling B. Stop the infusion and disconnect the IV line from the administration set. C. Aspirate the drus from the IV access device D. Monitor the patient and document.

B. Stop the infusion and disconnect the IV line from the administration set.

Plasma is part of which body fluid space compartments? Select all that apply: A. The ICF compartment B. The ECF compartment C. All fluid within the cells D. Interstitial fluid E. Intravascular fluid F. Fluid within joint capsules

B. The ECF compartment D. Interstitial fluid E. Intravascular fluid

2. The nurse assessing a client's peripheral IV site obtains and documents information about it. Which assessment data indicate the need for immediate nursing intervention? A.Client states, "It really hurt when the nurse put the IV in." B.The vein feels hard and cordlike above the insertion site. C.Transparent dressing was changed 5 days ago. D.Tubing for the IV was last changed 72 hours ago.

B. The vein feels hard and cordlike above the insertion site. A hard, cordlike vein suggests phlebitis at the IV site and indicates an immediate need for nursing intervention. The IV should be discontinued and restarted at another site.It is common for IVs to cause pain during insertion. An intact transparent dressing requires changing only every 7 days. Tubing for peripheral IVs should be changed every 72 to 96 hours.

A pt's blood osmalarity is 302 mOsm/L. What manifestation does the nurse expect to see in the pt? A. Increased urine output B. Thirst C. Peripheral Edema D. Nausea

B. Thirst

The pt with kypokalemia has an IV potassium supplement ordered. Which IV potassium supplement can be administered safely? A. KCl 5 mEq in 20 mL NS B. kCl 10 mEq in 100 mL NS C. KCl 15 mEq in 50 mL NS D. KCl 20 mEq in 100 mL NS

B. kCl 10 mEq in 100 mL NS

Which conditions cause the under production of bicarb? Select all that apply A. Heavy exercise B. kidney failure C. liver failure D. seizure activity E. dehydration F. Diarrhea

B. kidney failure C. liver failure E. dehydration

A newly admitted pt with CHF has a potassium level of 5.7. How does the nurse identify contributing factors for the electrolyte imbalance? Select all that apply. A. Assess the pt for hypokalemia B. obtain a list of pts home medications C. Assess the pt for hyperkalemia D. Ask about the pt's method of taking medications at home E. Evaluate the patient's appetite F. Auscultate for hypoactive bowel sounds

B. obtain a list of pts home medications C. Assess the pt for hyperkalemia D. Ask about the pt's method of taking medications at home

Which ABG results would the nurse interpret as normal? A. pH 7.28, PaCO2 24, bicarbonate 15, PaO2 95 B. pH 7.45, PaCO2 41, bicarbonate 25, PaO2 97 C. pH 7.35, PaCO2 24, bicarbonate 15, PaO2 95 D. pH 7.30, PaCO2 66, bicarbonate 38, PaO2 70

B. pH 7.45, PaCO2 41, bicarbonate 25, PaO2 97

Which ABG results would the nurse interpret as metabolic alkalosis? A. pH 7.30, PaCO2 66, bicarbonate 38, PaO2 70 B. pH 7.52, PaCO2 45, bicarbonate 36, PaO2 95 C. pH 7.55, PaCO2 24, bicarbonate 20, PaO2 95 D. pH 7.28, PaCO2 24, bicarbonate 15, PaO2 95

B. pH 7.52, PaCO2 45, bicarbonate 36, PaO2 95

1. The nurse is documenting peripheral venous catheter insertion for a client. What does the nurse include in the note? Select all that apply. A.Client's name and hospital number B.Client's response to the insertion C.Date and time inserted D.Type and size of device E.Type of dressing applied F.Vein used for insertion

B.Client's response to the insertion C.Date and time inserted D.Type and size of device E.Type of dressing applied F.Vein used for insertion The client's ability to adapt to interventions, such as IV insertion, should be noted when the intervention is performed. The date and time of the insertion are important data. IV sites need to be routinely monitored and changed at prescribed intervals per facility policy. It is important to note the device used (often the brand name is given), as well as all specifics such as needle or cannula length, gauge, and material (Teflon). It is necessary to describe the dressing applied, and the vein used should be noted.The client's name and hospital number should be on the medical record, but the nurse makes certain that the information is recorded in the correct medical record.

When cells becomes more malignant their shape tends to do what?

Becomes more smaller and rounder

Hypertrophy

Benign enlargement due to increased size of cells

VEGF medication

Bevacizumab (Avastin)

Vascular endothelial growth factor/receptor inhibitors

Bind to VEGR to prevent bindng of VEGF with it's receptors on surfaces of endothelial cells present on blood vessels.

monoclonal antibodies

Bind to target antigens (often specific cell surface membrane proteins) preventing protein from function and cell division

Epidermal growth factor/receptor inhibitors

Block epidermal growth factor from binding to surface receptor.

Why does chemotherapy pose an infection risk?

Bone marrow suppression and neutropenia

Proteasome inhibitors medication

Bortezomib (Velcade)

Hyperplasia and Hypertrophy

Both mean benign enlargement

A client has a sickle cell crisis with extreme lower extremity pain. What comfort measure does the nurse delegate to the unlicensed assistive personnel (UAP)? A. apply ice packs to the clients legs B. elevate the clients legs on pillows C. keep the lower extremities warm D. place elastic bandage wraps on the clients legs

C

A client has been treated for a deep vein thrombus and today presents to the clinic with petechiae. Lab results show a platelet count for 42,000/mm3. The nurse reviews the clients medication list to determine if the client is taking which drug? A. enoxaparin (Lovenox) B. Salicylates (aspirin) C. Unauctioned heparin D. warfarin (Coumadin)

C

A client has thrombocytopenia. What client statement indicates the client understands self-management of this condition? A. I brush and use dental floss everyday B. I chew hard candy for my dry mouth C. I usually put ice on bumps or bruises D. nonslip socks are best when I walk

C

A client is in the hospital and receiving IV antibiotics. when the nurse answers the clients call light, the client presents an appearance as shown below: A. administer epienphrine 1:1000, 0.3 mg IV push immediately B. apply oxygen by facemask at 100% and a pulse oximeter C. ensure a patent airway while calling the rapid response team D. reassure the client that these manifestations will go away

C

A nurse assesses a client on a medical-surgical unit. Which client should the nurse evaluate for a wound infection? A. client with blood cultures pending B. client who has thin, serous wound drainage C. client with a white blood cell count of 23,000/mm3 D. client whose wound has decreased in size

C

A nurse assesses a client who has a burn injury. Which statement indicates the client has a positive perspective of his or her appearance? A. I will allow my spouse to change my dressings B. I want to have surgical reconstruction C. I will bathe and dress before breakfast D. I have secured the pressure dressing as ordered

C

After assessing an older adult client with a burn wound, the nurse documents the findings as follows: vital signs-Pulse 110 bpm, BP 112/68mmHg, Resp 20, o2 rate 94% pain 3/10 laboratory results RBC 5,000,000/mm3, WBC 10,000/mm3, platelet count 200,000/mm3 Wound assessment left chest burn wound, 3cm, 2.5 cm, 0.5 cm wound bed pale, surrounding tissues with edema present. Based on the documented data, which action should the nurse take next? A. assess the clients skin for signs of adequate perfusion B. calculate intake and output ration for the last 24 hours C. prepare to obtain blood and wound cultures D. place the client in an isolation room

C

An emergency room nurse assesses a client who has been raped. With which health care team member should the nurse collaborate when planning this clients care? a. Emergency medicine physician b. Case manager c. Forensic nurse examiner d. Psychiatric crisis nurse

C

An intubated clients oxygen saturation has dropped to 88% what action by the nurse takes priory? A. determine if the tube is kinked B. ensure all connections are patent C. listen to the clients lung sounds D. suction the endotracheal tube

C

The nurse teaches burn prevention to a community group. Which statement by a member of the group should cause the nurse the greatest concern? A. I get my chimney swept every other year B. My hot water heater is set at 120 degrees C. Sometimes I wake up at night and smoke D. I use a space heater when it gets below zero

C

The provider requests the nurse start an infusion of an inotropic agent on a client. How does the nurse explain the action of these drugs to the client and spouse? a. "It constricts vessels, improving blood flow." b. "It dilates vessels, which lessens the work of the heart." c. "It increases the force of the heart's contractions." d. "It slows the heart rate down for better filling."

C

When transferring a client into a chair a nurse notices that the pressure relieving mattress overlay has deep imprints of the clients buttocks, heels, and scapulae. Which action should the nurse take next? A. turn the mattress overlay to the opposite side B. do nothing because this is an expected occurrence C. apply a different pressure-relieving device D. reinforce the overlay with extra cushions

C

a client had an acute myocardial infarction what assessment finding indicates to the nurse that a client significant complication has occurred? A. blood pressure that is 20 mm Hg below baseline B. oxygen saturation of 94% on room air C. poor peripheral pulses and cool skin D. urine output of 1.2 mL/kg/hr four 4 hours

C

a client has a pulmonary embolism and is started on O2. the student nurse asks why the clients O2 saturation has not significantly improved. what response by the nurse is best? A. breathing so rapidly interferes with oxygenation B. maybe the client has respiratory distress syndrome C. the blood clot interferes with perfusion in the lungs D. the client needs immediate intubation and mechanical ventilation

C

a client has been brought to the emergency department with a life threatening chest injury. what action by the nurse takes priority? A. apply oxygen at 100% B. assess the respiratory rate C. ensure a patient airway D. start two large-bore IV lines

C

the nurse is reviewing ECG results of a pt admitted for fluid and electrolyte imbalances. the T waves are tall and peaked. the nurse reports this finding to the health care provider and obtains an order for which serum level test? A. sodium B. glucose C. potassium D. phosphorus

C

the nurse is taking report on a pt who will be transferred from the cardiac ICU to the general med-surg unit. the reporting nurse states that S4 is heard on ausculation of the heart. this is most closely associated with which situation A. heart murmur B. pericardial friction C. ventricular hypertrophy D. normal heart sounds

C

The seven warning signs of cancer include?

C A U T I O N C- Changes in bowel or urinary habits A- A sore that does not heal U- Unusual bleeding or discharge T- Thickening or lump in breast I- Indigestion or difficulties swallowing O- Obvious changes in warts or moles N- Nagging cough or hoarseness

when working with women who are taking hormonal birth control, what health promotion measures should the nurse teach to prevent possible PE. select all that apply A. avoid drinking alcohol B. eat more omega-3 fatty acids C. exercise on a regular basis D. maintain a healthy weight E. stop smoking cigarettes

C, D, E

a client has an intra-arterial blood pressure monitoring line. the nurse notes bright red blood on the clients sheets. what action should the nurse perform first? A. assess the insertion site B. change the clients sheets C. put on a pair of gloves D. assess blood pressure

C.

Which pt is at greatest risk for developing hypocalcemia? A. 30 yr old Asian woman with breast cancer B. 45 yr old Caucasian man with hypertension and diuretic therapy C. 60 yr old African American woman with a recent ileostomy D. 70 yr old Caucasian man on long-term lithium therapy

C. 60 yr old African American woman with a recent ileostomy

A patient has a PICC placed by an IV therapy nurse at the bedside. Before using the catheter, how is its placement verified? A. The provider who ordered the procedure verifies placement. B. The line is aspirated gently and the nurse watched for blood return. C. A chest x-ray is taken, which shows the catheter tip in the lower superior vena cava. D. The line is slowly flushed with 10 mL of saline while the nurse notes the ease of flow.

C. A chest x-ray is taken, which shows the catheter tip in the lower superior vena cava.

What is the priority intervention for a patient with diabetic ketoacidosis? A. Administer bicarbonate B. Administer oxygen C. Administer insulin D. Administer potassium

C. Administer insulin

The unlicensed assistive personnel reports to the nurse that a pt being evaluated for kidney problems has produced a large amount of pale yellow urine. What does the nurse do next? A. Instruct UAP to measure the amount carefully and then discard the urine B. Instruct the UAP to save the urine in a large bottle for a 24 hour urine specimen C. Assess the patient for signs of fluid imbalance and check the specific gravity of the urine D. Compare the amount of urine output to the fluid intake for the previous 8 hours

C. Assess the patient for signs of fluid imbalance and check the specific gravity of the urine

The UAP informs the nurse that a pt with hypernatremia who was initially confused and disoriented on admission to the hospital is now trying to pull out the IV access and indwelling urinary catheter. What is the nurse's first action? A. Place bilateral soft wrist restraints B. Inform the provider of the pt's change in behavior and obtain an order for restraints C. Assess the pt D. Offer the pt oral fluids

C. Assess the pt

Which condition places a pt at risk for hypocalcemia, hyperkalemia, and hypernatremia? A. Hypothyroidism B. Diabetes Mellitus C. Chronic kidney disease D. Adrenal insufficiency

C. Chronic kidney disease

The nurse is caring for a postoperative surgical pt in the recovery time. What is the main reason for carefully monitoring the pt's urine output. A. decreasing urine output indicates poor kidney function B. Increasing urine output can indicate too much IV fluid during surgery C. Decreasing urine output may mean hemorrhage and risk for shock D. Increasing urine output may mean that kidney function is returning to normal

C. Decreasing urine output may mean hemorrhage and risk for shock

A pt in the hospital has a severely elevated magnesium level. Which intervention should the nurse complete first? A. Discontinue oral magnesium B. Administer furosemide (Lasiz) C. Discontinue parenteral magnesium D. Administer calcium to treat bradycardia

C. Discontinue parenteral magnesium

Which fluid has the highest corresponding electrolyte content? A. ICF is highest in potassium B. ECF is highest in sodium C. ECF is highest in sodium and chloride D. ICF is highest in magnesium and sodium

C. ECF is highest in sodium and chloride

The nurse is reviewing orders for several pts who are at risk for fluid volume overload. For which patient condition does the nurse question an order for diauretics? A. Pulmonary Edema B. Congestive Heart Failure C. End-stage renal failure D. Ascites

C. End-stage renal failure

Which assessment finding indicates that a pt with chronic respiratory acidosis is responding favorably to treatment? A. Nail beds pale, extremities cool B. Respiratory stridor with inspiration C. Expectorating clear, thin mucous D. Diffuse crackles auscultated bilaterally

C. Expectorating clear, thin mucous

Which pt is most likely to develop respiratory alkalosis? A. Hypoxic patient B. Patient with a body cast C. Fearful patient having a panic attack D. Morbidly obese patient

C. Fearful patient having a panic attack

17. A client is seen in the emergency department (ED) with pain, redness, and warmth of the right lower arm. The client was in the ED last week after an accident at work. On the day of the injury, the client was in the ED for 12 hours receiving IV fluids. On close examination, the nurse notes the presence of a palpable cord 1 inch (2.5 cm) in length and streak formation. How does the nurse classify this client's phlebitis? A.Grade 1 B.Grade 2 C.Grade 3 D.Grade 4

C. Grade 3 Grade 3 indicates pain at the access site with erythema and/or edema and streak formation with a 1' palpable cord.Grade 1 indicates only erythema with or without pain; the client has additional symptoms. Grade 2 indicates only pain at the access site with erythema and/or edema; the client has additional symptoms. Grade 4 indicates pain at the access site with erythema and/or edema, streak formation, a palpable venous cord longer than 1 inch (2.5 cm), and purulent drainage. No purulent drainage is present in this client, and the palpable cord is 1 inch (2.5 cm) in length.

The nurse is caring for several patients at risk for falls because of fluid and electrolyte imbalances. Which task is related to patient safety and fall prevention does the nurse delegate to the UAP? A. Assess for orthostatic hypotension B. Orient the patient to the environment C. Help the incontinent patient to toilet every 1-2 hours D. Encourage family members or significant others to stay wth the patient

C. Help the incontinent patient to toilet every 1-2 hours

A patient is talking to the nurse about sodium intake. Which statement by the patient indicates and understanding of high sodium food sources? A. I have bacon and eggs every morning for breakfast B. We never eat seafood because of the salt water C. I love chinese food, but I have it up because of the soy sauce. D. Pickled herring is a fish, and my doctor told me to eat a lot of fish.

C. I love chinese food, but I have it up because of the soy sauce.

The nurse reviews the electrocardiogram and cardiovascular status of a patient. Which findings are early changes associated with acidosis? A. Decreased heart rate with hypertension B. Hypotension and faint peripheral pulses C. Increased heart rate and increased cardiac output D. Peaked T waves and wide QRS complexes

C. Increased heart rate and increased cardiac output

13. A client is to receive an IV solution of 5% dextrose and 0.45% normal saline at 125 mL/hr. Which system provides the safest method for the nurse to accurately administer this solution? A. Controller B. Glass container C.Infusion pump D.Syringe pump

C. Infusion pump The safest method is to administer the solution with an infusion pump. Infusion pumps are used for drugs or fluids under pressure. They accurately measure the volume of fluid being infused.A controller is a stationary, pole-mounted electronic device that uses a sensor to monitor fluid flow and detect when flow has been interrupted. Because controllers rely completely on gravity to create fluid flow and do not create pressure, they do not ensure infusion but only control the drip rate. A glass container is necessary to use only with IV solutions that may cling to the plastic bag. This IV solution does not cling to plastic bags. A syringe pump does not hold sufficient volume to be practical in this situation.

A pt's ABG results show an increase in pH. Which condition is most likely to contribute to this laboratory value? A. Mechanical ventilation B. Diabetic Ketoacidosis C. Nasogastric suction D. Diarrhea

C. Nasogastric suction

A 65 yr old pt has a potassium laboratory value of 5.0 mEq/L. How does the nurse interpret this value? A. High for the pt's age B. Low for the pts age C. Normal for the pts age D. Dependent upon the medical diagnosis

C. Normal for the pts age

9. Which client does the charge nurse on a medical-surgical unit assign to the LPN/LVN? A.Cardiac client who has a diltiazem (Cardizem) IV infusion being titrated to maintain a heart rate between 60 and 80 beats/min B.Diabetic client admitted for hyperglycemia who is on an IV insulin drip and needs frequent glucose checks C.Older client admitted for confusion who has a heparin lock that needs to be flushed every 8 hours D.Postoperative client receiving blood products after excessive blood loss during surgery

C. Older client admitted for confusion who has a heparin lock that needs to be flushed every 8 hours The older client admitted for confusion with a heparin lock is the most stable and requires basic monitoring of the IV site for common complications such as phlebitis and local infection, which would be familiar to an LPN/LVN.The cardiac client with a diltiazem (Cardizem) IV infusion, the diabetic client on an IV insulin drip, and the postoperative client receiving blood products all are not stable and will require ongoing assessments and adjustments in IV therapy that should be performed by an RN.

The nurse instructs the unlicensed assistive personnel to use precautions with moving and using a lift sheet for which patient with an electrolyte imbalance? A. Young woman with diabetes and hyperkalemia B. Patient with psychiatric illness and hyponatremia C. Older woman with hypocalcemia D. Child with severe diarrhea and hypomagnesemia

C. Older woman with hypocalcemia

Which patient is the most likely candidate for a tunneled Central venous catheter? A. Patient with trauma from a motor vehicle accident. B. Patient in need of IV antibiotics for several weeks C. Patient in need of permanent parental nutrition D. Patient in need of intermittent chemotherapy.

C. Patient in need of permanent parental nutrition

Which precaution or intervention does the nurse teach a pt at continued risk for hypernatremia? A. Avoid salt substitutes B. Avoid aspirin and aspirin-containing products C. Read labels on canned or packaged foods to determine sodium content D. Increase daily intake of caffeine containing foods and beverages

C. Read labels on canned or packaged foods to determine sodium content

Which occurrence can be a result of hyperventilation? A. Hypocalcemia B. Anxiety C. Respiratory Alkalosis D. Respiratory Acidosis

C. Respiratory Alkalosis

The nurse observes tall peaked T waves on the ECG of a pt with metabolic acidosis. Before notifying the HCP, the nurse would assess the results of which laboratory test? A. Serum calcium B. Serum glucose C. Serum potassium D. Serum magnesium

C. Serum potassium

Pts with which conditions are at risk for developing hypernatremia? Select all that apply: A. Chronic constipation B. Heart failure C. Severe diarrhea D. Decreased kidney function E. Profound diaphoresis F. Cushing's syndrome

C. Severe diarrhea D. Decreased kidney function E. Profound diaphoresis F. Cushing's syndrome

A patient requires IV therapy via a peripheral line. What considerations does the nurse use when inserting the peripheral IV? A. Use either an upper or lower extremity for the insertion site. B. Start with more distal sites, such as the hand veins. C. Start with more proximal sites, such as the forearm. D. Choose the patient's non-dominant arm. E. Do not use the arm if the patient has a mastectomy on that side. F. If the vein is hard and cord-like, use an indirect approach. G. Avoid placing an IV over the palm side of the wrist.

C. Start with more proximal sites, such as the forearm.

Which nursing assessment finding indicates a worsening of respiratory acidosis? A. Decreased respiratory rate B. Decreased blood pressure C. Use of accessory respiratory muscles D. Pale nail beds

C. Use of accessory respiratory muscles

Which ABG value indicates an alkaline condition? A. PaCO2 = 66 B. Bicarbonate = 16 C. pH = 7.55 D. pH = 7.32

C. pH = 7.55

Early Signs/Symptoms HIV

CD4 + T Cell = 200-499 cells/mm3

Asymptomatic HIV Infection

CD4 + T Cell = 500 cells/mm3

HIV changes to AIDS

CD4 + T Cell = less than 200 cells/mm3

Tyrosine kinase inhibitors s/e

Cause fluid retention, electrolyte imbalance, and bone marrow suppression

Iatrogenic

Caused by us (Ex. Iatrogenic Cushing's disease)

Grading compares cancer cells w/ normal parent tissue from which it arose based on?

Cells appearance and activity

Oncogene activation

Cells are turned on under controlled conditions for cellular regulation - MONO ( under stressful conditions its turned on)

What cells have the ability to divide throughout a person lifespan?

Cells of the 1) Skin 2) Hair 3) Mucosa Membrane 4) Bone Marrow - Linings of the organs such 5) Lungs 6) Stomach 7) Intestines 8) Bladder 9) Uterus

The transformation of a normal cell to a malignant cell includes what and causes the loss of what?

Changes to the genes (DNA) of the normal cell causing a loss in cellular regulation

Titers

Checking antibody levels in the blood to see if they are adequate against a specific disease

Chemotaxis

Chemical signal calling WBC's to the site

Vesicants

Chemicals that damage tissue on direct contact - may require surgery for extreme tissue damage

Tumor lysis syndrome

Chemotherapy can cause massive destruction of cells leading the creation of uric acid which can be toxic to the kidneys leading to Acute Tubular Necrosis. You can try to prevent this with hydration and allopurinol.

Adjuvant therapy

Chemotherapy used along with surgery or radiation.

Grading of a tumor classifies what?

Classifies cellular aspect of the cancer

Lowest grading are given to those cells that are?

Closely resemble normal cells

Superior vena cava syndrome

Compression or obstruction of SVC by tumor growth or clots.

What aspect makes malignant cells difficult to control?

Contact inhibition loss due to loss of cellular regulation causing continuous cell division

Antimetobolites

Counterfeit metabolites that fool cancer cells

A client is brought to the ED after sustaining injuries in a severe car crash. the clients chest wall does not appear to be moving normally with respirations, oxygen saturation is 82%, and the client is cyanotic. what action by the nurse is the priority? A. administer oxygen and reassess B. auscultate the clients lung sounds C. facilitate a portable chest x-ray D. prepare to assist with intubation

D

A client is having a radioisotopic imaging scan. What action by the nurse is most important? A. Assess the client for shellfish allergies B. place the client on the radiation precautions C. sedate the client before the scan D. teach the client about the procedure

D

A client is in the preoperative holding area prior to surgery. The nurse notes that the client has allergies to avocados and strawberries. What action by the nurse is best? A. assess that the client has been NPO as directed B. communicate this information with dietary staff C. document the information in the clients chart D. ensure the information is relayed to the surgical team

D

A client with autoimmune idiopathic thrombocytopenic purpura (ITP) has had a splenectomy and returned to the surgical unit 2 hours ago. The nurse assesses the client and finds the abdominal dressing saturated with blood. What action is most important? A. preparing to administer a blood transfusion B. reinforcing the dressing and documenting findings C. removing the dressing and assessing the surgical site D. taking a set of vital signs and notifying the surgeon

D

A client with multiple myeloma demonstrates worsening bone density on diagnostic scans. About what drug does the nurse plan to teach this client? A. Bortezomib (Velcade) B. Dexamethasone (Decadron) C. Thalidomide (Thalomid) D. Zoledronic acid (Zometa)

D

A client with sickle cell disease (SCD) takes hydroxyurea (Droxia). the client presents to the clinic reporting an increase in fatigue. What lab result should the nurse report immediately? A. hematocrit: 25% B. hemoglobin: 9.2 mg/dL C. potassium: 3.2 mEqL D. white blood cell count: 38,000/mm3

D

A hospitalized client has a platelet count of 58,000/mm3. What action by the nurse is best? A. encourage high protein foods B. institute neutropenic precautions C. limit visitors to healthy adults D. place the client on safety precautions

D

The nurse assesses a clients oral cavity and makes the discovery shown in the photo below: What action by the nurse is most appropriate? A. encourage the client to have genetic testing B. instruct the client on high-fiber foods C. place the client in protective precautions D. teach the client about cobalamin therapy

D

The registered nurse assigns a client who has an open burn wound to a licenced practical nurse (LPN). Which instruction should the nurse provide to the LPN when assigning this client? A. administer the prescribed tetanus toxoid vaccine B. assess the clients wounds for signs of infection C. encourage the client to breath deeply every hour D. wash your hands on entering the clients room

D

While at a public park, a nurse encounters a person immediately after a bee sting. The person's lips are swollen, and wheezes are audible. Which action should the nurse take first? a. Elevate the site and notify the person's next of kin. b. Remove the stinger with tweezers and encourage rest. c. Administer diphenhydramine (Benadryl) and apply ice. d. Administer an EpiPen from the first aid kit and call 911.

D

a client is in the preoperative holding area prior to an emergency coronary artery bypass graft (CABG). the client is yelling at family members and tells the dr to just get this over with when asked to sign the consent form. what action by the nurse is best? A. ask the family members B. inform the client that this behavior is unacceptable C. stay out of the room to decrease the clients stress levels D. tell the client that anxiety is common and that you can help

D

a client is on mechanical ventilation and the clients spouse wonders why ranitidine (zantac) is needed since the client only has lung problems. what response by the nurse is best? A. it will increase the motility of the GI tract B. it will keep the GI tract functioning normally C. it will prepare the GI tract for enteral feedings D. it will prevent ulcers from the stress of mechanical ventilation

D

a client undergoing hemodynamic monitoring after a MI has a right atrial pressure of 0.5 mm Hg. what action by the nurse is most appropriate? A. level the transducer at the phlebostatic axis B. lay the client in the supine position C. prepare to administer diuretics D. prepare to administer a fluid bolus

D

the nurse is preparing to change a clients sternal dressing. what action by the nurse is most important? A. assess the vital signs B. don a mask and gown C. gather needed supplies D. perform hand hygiene

D

the nurse is reviewing diagnostic test results for a pt who is hypertensive. which lab result is an early warning sign of decreased heart compliance and prompts the decreased heart compliance and prompts the nurse to immediately notify the health care provider? A. normal B-type natriuertic peptide B. decreased hemoglobin C. elevated thyroxine (T4) D. presence of microalbuminuria

D

the nurse is reviewing the dying pts med record and sees that one tab of hyoscyamine 0.125mg was administered 2 hours ago. which assessment will the nurse perform in order to determine if the medication is effective A. assess for agitation and restlessness B. ask the pt if the nausea has decreased C. palpate the bladder to assess for urinary retention D. observe for oral secretions or wet sounding respirations

D

what is an early sign of left ventricular failure that a pt is most likely to report? A. nocturia B. weight gain C. swollen legs D. nocturnal coughing

D

a nurse prepares to discharge an older adult client home from the ED. which actions should the nurse take to prevent future ED visits? select all that apply A. provide medical supplies to the family B. consult a home health agency C. encourage participation in community activites D. screen for depression and suicide E. complete a functional assessment

D, E

Which body pH level can be fatal? A. 7.22 B. 7.11 C. 7.05 D. 6.85

D. 6.85

What interventions are appropriate for a pt with hypernatremia caused by reduced kidney sodium exretion? A. Hypotonic solutions B. o.45% sodium chloride iv infusion C. D5W IV infusion D. Administration of bumetanide E. Ensure adequate water intake F. Diuretics such as furesomede

D. Administration of bumetanide F. Diuretics such as furesomede

A pt is at risk for fluid volume excess and dependent edema. Which task does the nurse delegate to the UAP? A. Massage the legs and heels to stimulate circulation B. Evaluate the effectiveness of a pressure reducing mattress C. Assess the coccyc, elbows, and hips daily for signs of redness D. Assist the pt to change position every 2 hours

D. Assist the pt to change position every 2 hours

A young adult pt is in the early stages of being treated for severe burns. Which electrolyte imbalance does the nurse expect to assess in this pt. A. Hypernatremia B. Hypokalemia C. Hypercalcemia D. Hyperkalemia

D. Hyperkalemia

The nurse is evaluating the lab results of a pt with hyperaldosteronism. What abnormal electrolyte finding does the nurse expect to see? A. Hyponatremia B. Hyperkalemia C. Hypocalcemia D. Hypernatremia

D. Hypernatremia

Which type of electrolyte imbalance does the nurse expect to see in a patient with metabolic alkalosis? A. Hyperkalemia B. Hypophosphatemia C. Hyperchloremia D. Hypocalcemia

D. Hypocalcemia

A pt has a magnesium level of 0.8 mg/dL. Which treatment does the nurse expect to be ordered for this pt? A. Intramuscular magnesium sulfate B. Increased intake of fruits and vegetables C. Oral preparations of magnesium sulfate D. IV magnesium sulfate and discontinuation of diuretic therapy

D. IV magnesium sulfate and discontinuation of diuretic therapy

The health care provider orders magnesium sulfate MgSO4 for a patient with severe hypomagnesemia. What is the preferred route of administration for this drug? A. Oral B. Subcutaneous C. Intramuscular D. Intravenous

D. Intravenous

The nurse is caring for several pts at risk for fluid and electrolyte imbalances. Which pt problem or condition can result in a relative hypernatremia? A. Use of a salt substitute B. Presence of a feeding tube C. Drinking too much water D. Long-term NPO status

D. Long-term NPO status

An older adult patient at risk for fluid and electrolyte problems is carefully monitored by the nurse for the first indication of a fluid balance problem. What is this indication? A. Fever B. Elevated blood pressure C. Poor skin turgor D. Mental status changes

D. Mental status changes

After a 5k run on a hot summer day, a diaphoretic patient tells the volunteer nurse that she is very thirsty. A. Instruct the pt to sit down B. Apply ice to the pt's axilla area C. Tell the pt to breathe slowly and deeply D. Offer the pt bottled water to drink

D. Offer the pt bottled water to drink

The nurse is assessing skin turgor in a 65-yr-old patient. What is the correct technique to use with this patient? A. Observe the skin for a dry, scaly appearance and compare it to a previous assessment B. Pinch the skin over the back of the hand and observe for tenting; count the number of seconds for the skin to recover position C. Observe the mucous membranes and tongue for cracks, fissures, or a pasty coating D. Pinch the skin over the strenum and observe for tenting and resumption of skin to its normal position after release

D. Pinch the skin over the strenum and observe for tenting and resumption of skin to its normal position after release

The nurse is caring for several pts with electrolyte imbalances. Which intervention is included in the plan of care for a pt with hypomagnesemia? A. Implementing an oral fluid restriction of 1500 mL/day B. Implementing a renal diet C. Providing moderate environmental stimulation with music D. Placing the patient on seizure precautions

D. Placing the patient on seizure precautions

Which serum laboratory value does the nurse expect to see in a patient with hyperkalemia? A. Calcium greater than 8.0 mg/dL B. Potassium greater than 3.5 mEq/L C. Calcium greater than 11.0 mg/dL D. Potassium greater than 5.0 mEq/L

D. Potassium greater than 5.0 mEq/L

11. A severely dehydrated client requires a rapid infusion of normal saline and needs a midline IV placed. Which staff member does the emergency department (ED) charge nurse assign to complete this task? A. RN who is certified in the administration of oral and infused chemotherapy medications B. RN with 2 years of experience in the ED who is skilled at insertion of short peripheral catheters C. RN with 10 years of experience on a medical-surgical unit who has cared for many clients requiring IV infusions D. RN with certified registered nurse infusion (CRNI) certification who is assigned to the ED for the day

D. RN with certified registered nurse infusion (CRNI) certification who is assigned to the ED for the day The nurse with CRNI certification is most likely to be able to quickly insert a midline catheter for a client who is dehydrated.The chemotherapy nurse and the ED nurse have the appropriate scope of practice, but will not be as skilled in inserting a midline IV catheter. The medical-surgical nurse may be skilled at inserting short peripheral catheters, but will not be skilled in inserting midline IV catheters.

The nurse is selecting a site for peripheral IV insertion. Which patient condition influences the choice of left versus right upper extremity? A. Pneumothorax with a chest tube on the right side. B. Myocardial infarction with pain radiating down the left arm. C. Right hip fracture with immobilization and traction in place. D. Regular renal dialysis with a shunt in the left upper forearm.

D. Regular renal dialysis with a shunt in the left upper forearm.

A pt's ABG results reveal respiratory acidosis. How does the body compensate for this imbalance? A. Loss of bicarbonate B. Regular, unlabored respirations C. Hypoventilation D. Renal reabsorption of bicarbonate

D. Renal reabsorption of bicarbonate

22. A 70-year-old client with severe dehydration is ordered an infusion of an isotonic solution at 250 mL/hr through a midline IV catheter. After 2 hours, the nurse notes that the client has crackles throughout all lung fields. Which action does the nurse take first? A.Assess the midline IV insertion site. B.Have the client cough and deep-breathe. C.Notify the health care provider about the crackles. D.Slow the rate of the IV infusion.

D. Slow the rate of the IV infusion. The presence of crackles throughout the lungs is a sign of possible fluid overload. The nurse should slow the rate of infusion and further assess for indicators of volume overload and/or respiratory distress.The presence of crackles throughout the lungs is a sign of possible fluid overload. The nurse should slow the rate of infusion and further assess for indicators of volume overload and/or respiratory distress. Assessing the site and having the client cough and deep-breathe are not appropriate. Crackles do not disappear with coughing. Notifying the provider may be appropriate, but is not the initial actions for this client.

The nurse is assessing a pt with metabolic alkalosis. Which neuromuscular finding is the most ominous and warrants immediate notification of the HCP? A. Muscle cramps B. Muscle twitching C. Hyperactive deep tendon refleces D. Tetany

D. Tetany

The nurse is supervising a student nurse who is preparing an IV bag with IV administration tubing. For which action by the student nurse must the nurse intervene? A. The student touches the drip chamber. B. The sterile cap from the distal end of the set is removed. C. The distal end is attached to a needless connector. D. The student touches the tubing spike.

D. The student touches the tubing spike.

Under what circumstances does the nurse elect to use only one secondary set rather than a secondary set for each medication? A. When multiple intermittent medications are required B. To eliminate the cost of using multiple secondary sets C. When the nurse is using the back-priming method D. When the medications are compatible

D. When the medications are compatible

The nurse is caring for a patient with hypernatremia caused by fluid loss? What type of IV solution is best for treating this patient? A. Hypotonic o.225% sodium chloride B. Small volume infusions of hypertonic solution (2-3% saline) C. Isotonic sodium chloride (NaCl) D. o.9% Sodium Chloride

D. o.9% Sodium Chloride

antitumor antibodies

Damage cell DNA

Necrosis

Death of tissue

Atrophy

Decreased size - shrinking of formerly normal tissue

Hypoplasia

Defective/stunted growth of cells in an organ (smaller than normal)

The nurse is assessing a pt's urine specific gravity. The value is 1.035. How does the nurse interpret this result? A. Overhydration B. Dehydration C. Normal value for an adult D. Renal disease

Dehydration

Ploidy provides what type of information?

Description of cancer cells by chromosomes number and appearance

Radiation therapy

Destroy cancer cells with minimal damage to surrounding healthy cells

Active Immunity

Developed by the host going through its own immune response

A nurse assesses clients who are at risk for diabetes mellitus. Which client is at greatest risk? a. A 29-year-old Caucasian b. A 32-year-old African-American c. A 44-year-old Asian d. A 48-year-old American Indian

Diabetes is a particular problem among African Americans, Hispanics, and American Indians. The incidence of diabetes increases in all races and ethnic groups with age. Being both an American Indian and middle-aged places this client at highest risk.

Pathogenesis

Disease mechanism

Topoisomerase inhibitors

Disrupts enzymes essential for cell division

Which type of medication increases an older adult pt's risk for acid-base imbalance? A. Antilipidemics B. Hormonal therapy C. Diuretics D. Antidysrhythmics

Diuretics

Tumor is assessed in terms of?

Doubling time and mitotic index

Extravasation

Drug leaks into surrounding tissue causing serious complications and tissue damage

Phagocytosis

Engulfing foreign material

Necropsy

Examination of a body after death

Initiation leads to?

Excessive cell division through DNA damage that result in loss of cellular regulation by either loss of suppressor gene function or enhancement of the oncogene function

SVC syndrome s/e

Facial edema, tightness of shirt after sleeping, decreased cardiac output

Malignant cell chromosomes are normal ( True/False)

False - they are Aneuploidy meaning abnormal chromosomes. They may have more, less, or even broken chromosomes compared to normal cells

During progression tumor cells have fewer what over time?

Fewer normal cell features

Proteasome inhibitors s/e

GI distress, decrease in taste, and peripheral neuropathy

Example of Vaccinations as a part of Primary cancer prevention includes?

Gardasil to prevent HPV

After a normal cell has been initiated by a carcinogen and is a cancer cell it can become a tumor if?

Growth is enhanced

Multikinase inhibitors s/e

HTN, mucusitis, GI distress, mild neutropenia, and thrombocytopenia

Describe Benign cells.

Harmless, does not usually require attention

Examples of Viral Carcinogenesis include?

Hep B Hep C HPV Epstein-Barr virus

Peracute Onset

Immediate - seconds-minutes ex. Fracture

Hyperplasia

Increase in size due to excessive number of normal cells

Hypertrophy is?

Increase tissue size by increased size of each cell

Hyperplasia is?

Increased tissue size due to increase number of cells

Alkylating agents

Interfere with DNA or RNA synthesis

Malignant cells usually appear ________ shape and are __________

Irregular Non-tender

Initiation is?

Is the change of the gene's expression ( now overly expressed oncogenes causing excessive cell division) caused by a substance ( carcinogen) that enters the cell, get into the nucleus and mutate the DNA leading to loss of cellular regulation

Malignant Transformation is the process that?

Is the process in which normal cells becomes cancer cells

Latency period

Is the time b/w a cell's initiation and development of an overt tumor

Doubling time

Is the time it takes for the tumor to double it size

A slow growing tumor has a mitotic index of?

Less then 10 %

TNM is not useful for?

Leukemia or lymphomas

What are the most common types of tumors that goes undiagnosed for years and is usually metastsize by the time its detected?

Liver, pancreas, and cervical cancers

Besides blood, what is another way cancer is metastsize is by?

Lymphatic spread

Specific Immunity

Lymphocytes - Cell mediated immunity - Humoral immunity Antibodies & Memory Cells

sarcoma

Malignant, connective tissue involvement (osteosarcoma)

carcinoma

Malignant, epithelial tissue involvement (Squamous cell carcinoma)

Hormonal manipulation s/e

Masculinity effects on women, feminizing effects on male, acne, fluid retention, hypercalcemia, liver dysfunction, venous thromboembolism

Chmeoprevention agents may do what?

May reverse the existing gene damage or Halt the progression of the transformation process

Additional tumors from metastasis are called?

Mestatic or secondary tumors

Type 1 Hypersensitivity

Minor Type 1 - allergy responses, hives, itching, possible diarrhea Severe Type 1 - loss of blood, swelling of airways, pulmonary edema

Neutrophils

Most prevalent, phagocytic, primary cell involved in bacterial invasion

Neoplasia

New or continuous growth of cells not needed for normal development or replacement of damaged or dead tissues

Is a patient receiving teletherapy radioactive?

No; because they are receiving a external therapy

How many cells are needed to go under malignant transformation for cancer to begin?

ONE

Lesion

Observed change in diseased tissue

What is the main mechanism of carcinogenesis?

Oncogene Activation

Viruses that can cause cancer are called?

Oncoviruses

43. The nurse has explained the endotracheal suc- tioning procedure to a patient, gathered equip- ment, washed hands, and set low wall suction. Indicate the correct steps of completing the suctioning procedure in order using 1 through 9. Open the suction kit. Pour sterile saline into sterile container. Preoxygenate the patient. Discard supplies, wash hands, and document. Put on sterile gloves. Keep catheter sterile; attach to suction. Withdraw catheter, applying suc- tion and twirling catheter. Insert catheter into trachea with- out suctioning. Lubricate catheter tip in sterile saline solutio

Open the suction kit. 1 Pour sterile saline into sterile container. 2 Put on sterile gloves. 3 Keep catheter sterile; attach to suction.4 Lubricate catheter tip in sterile saline solutio5 Preoxygenate the patient. 6 Insert catheter into trachea with- out suctioning. 7 Withdraw catheter, applying suc- tion and twirling catheter. 8 Discard supplies, wash hands, and document. 9

Give an example of chronic irritation or tissue trauma that can cause cancer.

People with burn scars are at higher risk for cancer

Pathologists

Person studying disease

proteasome inhbiotrs

Prevents formation of lg complex of proteins into cells

Grading provides what information?

Prognosis and appropriate therapy

Avoidance of known or potential carcinogen in "Primary" prevent is by doing what?

Put on sunscreen Don't smoke or drink alcohol

Mixed Neutrophils/Marcophages

Pyogranulomatous

Teletherapy

Radiation delivered from a source outside the pt.

Cancer Prevention " Secondary" include?

Regular screenings Genetic screenings

Removal of " at risk " tissues of Primary cancer prevention means?

Removing warts, polyps, breast tissue

Metaplasia

Replacement of one cell type by another

Clinical Signs

Results of disease - what the animal shows us - Founded on actual observation - limping, scratching

Monoclonal medication

Rituximab (Rituxan)

Cancer stage influence?

Selection of therapy

Bradytherapy

Short/close therapy. Source comes in direct/continuous contact with tumor for specific period of time.

Cons about benign cells.

Some benign cells can become cancerous

Some chromosomes changes are associate w/ ?

Specific cancers and used for diagnosis and prognosis

TNM staging system provides what type of information for what type of cancers?

Specific prognostic values for each solid tumor type

Pathology

Study of disease

Histopathology

Study of diseased tissue microscopically

Carcinogens are?

Substances that change the activity of the cell's genes so that the cell becomes a cancer cell

The nurse has explained the tracheostomy care procedure to a patient, gathered equipment, and washed hands. Indicate the correct steps of completing the tracheostomy care procedure in order using 1 through 10. Remove old dressing and excess secretions. Wash hands, dispose of equip- ment, and document. Suction tracheostomy tube if necessary. Put on sterile gloves. Reinsert inner cannula into outer cannula. Open tracheostomy kit and pour peroxide into one side of con- tainer and saline into another. Clean stoma site and plate. Rinse inner cannula in sterile sa- line. Remove inner cannula; place it in peroxide solution use brush to clean. Change tracheostomy ties if needed and place new tracheos- tomy dressing.

Suction tracheostomy tube if necessary. 1 Remove old dressing and excess secretions. 2 Open tracheostomy kit and pour peroxide into one side of con- tainer and saline into another. 3 Put on sterile gloves. 4 Remove inner cannula; place it in peroxide solution use brush to clean.5 Rinse inner cannula in sterile sa- line. 6 Reinsert inner cannula into outer cannula. 7 Clean stoma site and plate. 8 Change tracheostomy ties if needed and place new tracheos- tomy dressing. 9 Wash hands, dispose of equip- ment, and document. 10

Examples of Ultraviolet (UV) includes

Sun, tanning beds

Angiogensis inhibitors

Targets mammalian target of rapamycin (MTOR)

Angiogensis medication

Temsirolimus (Torisel)

Example of Hypertrophy is?

The amount of heart muscle cells are fixed at birth, but the heart will continue to grow in size as the person grows, but each heart muscle cell will increase in their size rather then their amount

TNM is used to describe what?

The anatomic extent of cancers

Progression is defined as ?

The continued change of a cancer , making it more malignant over time

Promotion is defined as?

The enhanced growth of an initiated cell by substances known as promoters

Staging Determines what?

The exact location of the cancer ( how many tissue it invades) and degree of metastsis

Prognosis

The expected outcome - poor - fair - good - excellent

Differential Diagnosis

The list of possible etiologies (causes) we come up with after learning the signalment, history, and clinical signs

Anaphylaxis

The most acute and severe reaction is

Mitotic index

The percentage of actively dividing cells w/I a tumor

Primary tumor is identified by?

The tissue from which it arose (parent tissue) Ex: Lung cancer/Breast cancer

Chemoprevention as a part of "Primary" prevention of cancer includes?

The use of drugs, chemicals, nutrients, hormones and other substances to disrupt the important steps of cancer development

Define "Primary" cancer prevention.

The use of strategies to prevent the actual occurrences of cancer

Promoters are?

They are many normal hormones and body proteins like insulin and estrogen that make cell division more frequent

In neoplasia are new cells cancerous or benign?

They may be benign or cancerous

Describe Benign cells growth.

They normal cells growing at the wrong place or at the wrong time

What is one way tumor cells can enter the blood vessel?

They release enzymes that creates large pores in the blood vessels allowing tumor cells to enter and circulate

Malignant Transformation occurs?

Through the loss of cellular regulation leading to initiation, promotion, progression, and metastasis

Grading and staging have been develop to what?

To standardize cancer diagnosis, prognosis, and treatment

What is the single most preventable source of carcinogen?

Tobacco

Hyperactivity

Too much immune response

Blood Borne Metastasis

Tumor cells release into the blood and spread to distant organs and tissues

TNM stands for?

Tumor, Node, Metastsis

Chemotherapy

Tx of cancer with chemicals to increase survival time or cure

Metastasis occurs at a slow or fast rate

Usually fast

Idiopathic

We don't know

Metastasis occurs when?

When cancer cells move from the primary location by breaking off from the original group and establishing remote colonies in other places Ex: lung cancer invades the esophagus

Define Co-carcinogens.

When taking two carcinogens at a time, they enhance each other carcinogenic activity

Primary tumor is the ?

Where the initial start of the tumor took place. Its the original tumor

Acute Onset

Within minutes-hours- ex. food poisoning, allergic response

Is a patient receiving bradytherapy radioactive?

Yes; because the therapy is internal

Which procedure has a risk for complication of pneumothorax a. Thoracentesis b. Bronchoscopy c. PFT d. Ventilation-perfusion scan

a

Drug therapy with first-generation antihistamines to treat sinusitis is used with caution in older adults because of which possible side effect? SELECT ALL THAT APPLY a. reduced clearance b. hypotension c. confusion d. dry mouth e. constipation f. decreased risk of confusion

a c d e

Which of the following may be warning signs of lung cancer? Select all that apply. a. Dyspnea b. Dark yellow-colored sputum c. Persistent cough or change in cough d. Abdominal pain and frequent stools e. Use of accessory muscles for breathing f. Labored or painful breathing

a. Dyspnea c. Persistent cough or change in cough e. Use of accessory muscles for breathing f. Labored or painful breathing

Which drugs are essential for slowing the progression of the disease in a patient with pulmonary fibrosis? a. Immunosuppressants b. Opioids c. Antibiotics d. Bronchodilators

a. Immunosuppressants

Immunity

ability of the body to protect itself multi-factorial

Natural Immunity

acquired through exposure to the actual pathogen

Radiation dose

amount of radiation absorbed by tissue

Radiation exposure

amount of radiation delivered to a tissue

Malignant cells

are abnormal, serve no useful function, and are harmful to the normal body tissues.

external factors, including environmental exposure

are responsible for about 80% of cancer in north america

30% of cancers diagnosed in north america

are tobacco use related

hematologic cancers

arise from blood cell forming tissues

solid tumors

arise from specific tissues

colonoscopy

at age 50 years and then q 10 years

Which aspect of PFTs would be considered a normal result in the older adult? a. Incrased forced vital capacity b. Decline in forced expiratory volume in 1 second c. Decrease in diffusion capacity of carbon monoxide d. Increased functional residual capacity

b

Which condition causes a pt to have the GREATEST risk for ventilator associated pneumonia? a. history of alcohol use b. presence of feeding tube c. weight loss d. IV therapy with normal saline

b

What is the most serious complication of cystic fibrosis? a. Pancreatic insufficiency b. Constant presence of thick, sticky mucous c. Intestinal obstruction d. Cirrhosis of the liver

b. Constant presence of thick, sticky mucous

18. Nursing interventions to prevent infection in patients with humidified oxygen include which actions? a. Use sterile normal saline to provide mois- ture. b. Drain condensation into the humidifier. c. Drain condensation from the water trap. d. Maintain a sterile closed system at all times.

c c. Drain condensation from the water trap.

The nurse is instructing a patient regarding complications of COPD. Which statement by the patient indicates the need for additional teaching? a. "I have to be careful because I am susceptible to respiratory infections." b. "I could develop heart failure, which could be fatal if untreated." c. "My COPD is serious, but it can be reversed if I follow my doctor's order." d. "The lack of oxygen could cause my heart to beat in an irregular pattern."

c. "My COPD is serious, but it can be reversed if I follow my doctor's order."

hormonal manipulation

change in usual hormonal responses because some hormones make hormone-selective tumors rapidly grow

environmental carcinogens are

chemical, physical, or viral agents that cause cancer.

CINV

chemotherapy induced nausea and vomiting

malignant- abnormal chromosomes- aneuploidy

chromosomes are lost, gained, or broken; cells can have more or less than 23 chromosomes; may have broken and rearranged chromosomes.

What is a pulse oximeter used to meausre? a. Oxygen perfusion in the extremities b. Pulse and perfusion in the extremities c. Generalized tissue perfusion d. Hemoglobin saturation

d

A child attending day camp has asthma, and her parent sent with her all of medicine in a small carry bag. The child has an asthma attack that is severe enough to warrant a rescue drug. Which medication from the child's bag is best to use for the acute symptoms? a. Omalizumab b. Fluticasone c. Salmeterol d. albuterol

d. albuterol

21. What is the best description of the nurse's role in the delivery of oxygen therapy? a. Receiving the therapy report from the respiratory therapist b. Evaluating the response to oxygen therapy c. Contacting respiratory therapy for the de- vices d. Being familiar with the devices and tech- niques used in order to provide proper care

dd. Being familiar with the devices and tech- niques used in order to provide proper care

45. While the nursing student changes a patient's tracheostomy dressing, the nurse observes the student using a pair of scissors to cut a 4 x 4 gauze pad to make a split dressing that will fit around the tracheostomy tube. What is the nurse's best action? a. Give the student positive reinforcement for use of materials and technique. b. Report the student to the instructor for remediation of the skill. c. Change the dressing immediately after the student has left the room. d. Direct the student in the correct use of materials and explain the rational

dd. Direct the student in the correct use of materials and explain the rational

47. A patient with a tracheostomy is being dis- charged to home. In patient teaching, what does the nurse instruct the patient to do? a. Use sterile technique when suctioning. b. Instill tap water into the artificial airway. c. Clean the tracheostomy tube with soap and water. d. Increase the humidity in the home.

dd. Increase the humidity in the home.

48. A patient with a permanent tracheostomy is interested in developing an exercise regimen. Which activity does the nurse advise the pa- tient to avoid? a. Aerobics b. Tennis c. Golf d. Swimming

dd. Swimming

31. A patient with a tracheostomy without a tube in place develops increased coughing, inability to expectorate secretions, and difficulty breath- ing. What are these assessment findings related to? a. Overinflation of the pilot balloon b. Tracheoesophageal fistula c. Cuff leak and rupture d. Tracheal stenosis

dd. Tracheal stenosis

Lymphopenia

decreased lymphocytes

Neutropenia

decreased neutrophils

Antibodies

good guys

Monocytosis

increased monocytes

Neutrophilia

increased neutrophils

Multikinase inhibitors (MKI)

inhibit activity of specific kinases in cancer cells and tumor blood vessels

first step in carcinogenesis

intitiation

two types of radiation

ionizing and uv

initiation

irreversible event that can lead to cancer development. a cancer cell is not a threat if it cannot divide, it cannot form a tumor.

Vasodilation

leads to heat and redness

Pathognomonic lesion

lesion specific to only one disease condition. Tells you without a doubt that this is what the disease is

s/e of radiation

local skin changes, hair loss, taste alteration, fatigue, bone marrow suppression, xerostomia, risk for fx to bone exposed to radiation

malignant- anaplasia

lose the specific appearance of their parent cells. becomes smaller and rounded. many types of cancer cells look alike and this make it harder to diagnose type of cancer.

Most common causes of SVC syndrome

lymphoma, thyoma, lung cancer, breast cancer

oncogene activiation

main mechanism of carcinogenesis regardless of the specific cause, directed early embryonic development, should be controlled forever at 8 days post conception, damaged suppressor genes prevent control of the expression of oncogenes.

prostate specific antigen

men older than 50

benign- have normal chromosomes

most have 23 chromosomes, normal for humans

Diapedesis

movement of WBC's squeezing through vessels and crawling to the site of need

Neoplastic cells develop from what type of cells?

normal cells

Non-Infectious

not caused by an orgnaism

malignant- large nuclear cytoplasmic ratio

nucleus of a cancer cell is larger than that of a normal cell and the cancer cell is small. Large nucleus/small cytoplasm.

malignant- migration- metastasize

occurs because cancer cells do not bind tightly together and have many enzymes on their cell surfaces. can slip through blood vessels and other body sites. major cause of death.

Pathogen

organism causing disease

Chronic Onset

over days or longer

Acidosis

pH < 7.35

Alkalosis

pH > 7.45

immune function

protects the body from foreign invaders and non self cells. these include cancer cells. cell mediated immunity is the part of the immune system that protects against cancer. natural killer and helper t cells provide immune surveillance.

cyclins

proteins that promote cells to enter and complete cell division; activated and allow a cell to leave G0 state and enter the cycle. then they allow the cycle to take place.

benign- orderly growth, with normal growth patterns

rate of growth is normal but not appropriate time or place, hyperplastic expansion but no invasion.

malignant- rapid or continuous cell division

reenter the cell cycle for mitosis almost continuously, do not respond to apopototic signals and are "immortal"

lymphatic spread

related to the number structure and locatoin of lymh nodes and vessels, primary sites that are rich in lymphatics have more arly metastatic spread than areas with few lymphatics.

IgE

responds most to allergy and parasites

secondary prevention

routine screening, gene therapy in the future.

secondary prevention

screening strategies to detect cancer early, at time when cure or control is more likely

Microscopic or Histologic

seen through use of the microscope

Photodynamic therapy

selective destruction of cancer cells via chemical reactions triggered by light

malignant- specific functions are lost partially or completely in cancer cells.

serve no useful purpose

epidermal growth factor (EGF) s/e

skin reaction, cardiac effects

cancers are divided into two major categories:

solid and hematologic.

Signalment

species, breed, age and sex of a patient (include whether spayed or neutered

G0

state of not actively reproducing new cells. not dividing

tumor angiogenesis factor TAF

triggers capillaries and other blood vessels in the area to grow new brances into the tumor. ensure the tumors continued nourishment and growth.

TNM

tumor, node, metastasis; describes the anatomic extent of cancers, not for leukemia or lymphomas. useful for prognosis and comparison of treatment results

primary prevention

use of strategies to prevent the actual occurrence of cancer. most effective with a known cause

viral carcinogenesis

viruses affect body cells and break the dna strands, viruses can then insert their own genetic material into the human dna.

oncoviruses

viruses that cause cancer

Gross Lesion

visible to the naked eye ex. Canine Demodex

mammography

yearly for 40 years and older

breast exam

yearly for women older than 40

A client calls the clinic to report exposure to poison ivy and an itchy rash that is not helped with over the counter antihistamines. What response by the nurse is most appropriate? A. antihistamines do not help poison ivy B. there are different antihistamines to try C. you should be seen in the clinic right away D. you will need to take some IV steroids

A

A client has a platelet count of 9000/mm3. the nurse finds the client confused and mumbling. What action takes priority? A. calling the rapid response team B. delegating taking a set of vital signs C. instituting bleeding precautions D. placing the client on bedrest

A

A client has been admitted after sustaining a humerus fracture that occurred when picking up the family cat. What test results would the nurse correlate to this condition? A. bence-jones protein in urine B. epstein-barr virus: positive C. hemoglobin: 18 mg/dL D. red blood cell count: 8.2/mm3

A

A client has frequent hospitalizations for leukemia and is worried about functioning as a parent to four small children. What action by the nurse would be most helpful? A. assist the client to make sick day plans for household responsibilities B. determine if there are family members or friends who can help the client C. help the client inform friends and family that they will have to help out D. refer the client to a social worker in order to investigate respite child care

A

A client hospitalized with sickle cell crisis frequently asks for opioid plan medications, often shortly after receiving a dose. The nurses on the unit believe the client is drug seeking when the client requests pain medication, what action by the nurse is best? A. give the client pain medication if it is time for another dose B. Instruct the client not to request pain medication too early C. request the provider leave a prescription for a placebo D. tell the client it is too early to have more pain medication

A

A client suffered an episode of anaphylaxis and has been stabilized in the ICU. when assessing the clients lungs, the nurse hears the following sounds. What medication does the nurse prepare to administer? (click the audio button to hear the audio) A. albuterol (proventil) via nebulizer B. diphenhydramine (benadryl) IM C. epinephrine 1:10,000 5 mg IV push D. Methylpredniosolone (solu-Medrol) IV push

A

A clietn receiving rivaroxaban (Xarelto) and asks the nurse to explain how it works. what response by the nurse is best? A. it inhibits thrombin B. it inhibits fibrinogen C. it things your blood D. it works against vitamin K

A

A hospital responds to a local mass casualty event. Which action should the nurse supervisor take to prevent staff post-traumatic stress disorder during a mass casualty event? a. Provide water and healthy snacks for energy throughout the event. b. Schedule 16-hour shifts to allow for greater rest between shifts. c. Encourage counseling upon deactivation of the emergency response plan. d. Assign staff to different roles and units within the medical facility.

A

A nurse administers topical gentamicin sulfate (Garamycin) to a clients burn injury. Which laboratory value should the nurse monitor while the client is prescribed this therpy? A. creatinine B. red blood cells C. sodium D. magnesium

A

A nurse assess a client who has open lesions. Which action should the nurse take first? A. put on gloves B. ask the client about his or her occupation C. assess the client pain D. obtain vital signs

A

A nurse assess an older adult client with the skin disorder shown below: How should the nurse document this finding? A. petechiae B. ecchymoses C. actinic lentigo D. senile angiomas

A

A nurse assesses a client who has a chronic skin disorder. Which finding indicates the client is effectively coping with the disorder? A. clean hair and nails B. poor eye contact C. disheveled appearance D. drapes a scarf over the face

A

A nurse assesses a client who has a chronic wound. The client states, I do not clean the wound and change the dressing every day because it costs too much for supplies. How should the nurse respond? A. you can use tap water instead of sterile saline to clean your wound B. if you don't clean the wound properly, you could end up in the hospital C. sterile procedure is necessary to keep this wound from getting infected D. good hand hygiene is the only thing that really matters with wound care

A

A nurse assesses a client who has a lesion on the skin that is suspicious for skin cancer as shown below: Which diagnostic test should the nurse anticipate being ordered for this client? A. punch skin biopsy B. viral cultures C. woods lamp examination D. diascopy

A

A nurse assesses a client who has two skin lesions on his chest. Each lesion is the size of a nickel, flat, and darker in color than the clients skin. How should the nurse document these lesions? A. two 2cm hyperpigmented patches B. two 1 inch erythematous plaques C. two 2mm pigmented papules D. two 1 inch moles

A

A nurse assesses a client who is admitted with inflamed soft-tissue folds around the nail plates. Which question should the nurse ask to elicit useful information about the possible condition? A. what do you do for a living? B. are your nails professionally manicured? C. do you have diabetes mellitus? D. have you had a recent fungal infection?

A

A nurse assesses an older client who is scratching and rubbing white ridges on the skin between the fingers and on the wrists. Which action should the nurse take? A. place the client in a single room B. administer an antihistamine C. assess the clients airway D. apply gloves to minimize friction

A

A nurse cares for a client who has a deep wound that is being treated with a wet-to-damp dressing. Which intervention should the nurse include in this clients plan of care? A. change the dressing every 6 hours B. assess the wound bed once a day C. change the dressing when it is saturated D. contact the provider when the dressing leaks

A

A nurse cares for a client with burn injuries. Which intervention should the nurse implement to appropriately reduce the clients pain? A. administer the prescribed intravenous morphine sulfate B. apply ice to skin around the burn wound for 20 minutes C. adminster prescribed intramuscular ketorolac (Toradol) D. decrease tactile stimulation near the burn injuries

A

A nurse caring for a client with sickle cell disease (SCD) reviews the clients laboratory work. Which finding should the nurse report to the provider? A. creatinine: 2.9 mg/dL B. hematocrit: 30% C. Sodium: 147 mEq/L D. WBC count 12,000/mm3

A

A nurse delegates hydrotherapy to an unlicensed assistive personnel (UAP). which statement should the nurse include when delegating this activity? A. keep the water temperature constant when showering the client B. assess the wound beds during hydrotherapy C. apply a topical enzyme agent after bathing the client D. use sterile saline to irrigate and clean the clients wounds

A

A nurse has educated a client on an epinephrine auto-injector (EpiPen). what statement by the client indicates additional instruction is needed? A. I don't need to go to the hospital after using it B. I must carry two EpiPens with me at all times C. I will write the expiration date on my calendar D. this can be injected right through my clothes

A

Emergency personnel discovered a patient lying outside in the cool evening air for an unknown length of time. The patient is in a hypothermic state and the metabolic needs of the tissues are decreased. What other assessment finding does the nurse expect to see? a. Blood pressure and heart rate lower than normal b. Heart rate and respiratory rate higher than normal c. Normal vital signs due to compensatory mechanisms d. Gradually improved vital signs with enteral nutrition

A

The EMTs arrive at the ED with an unresponsive client with an oxygen mask in place. What will the nurse do first? A. assess that the client is breathing adequately B. insert a large bore IV line C. please the client on a cardiac monitor D. assess for the best neurologic response

A

The nurse assess a client who has a severe burn injury. Which statement indicates the client understands the psychosocial impact of a severe burn injury? A. It is normal to feel some depression B. I will go back to work immediately C. I will not feel anger about my situation D. once I get home, things will be normal

A

The nurse gets the hand-off report on four clients. Which client should the nurse assess first? a. Client with a blood pressure change of 128/74 to 110/88 mm Hg b. Client with oxygen saturation unchanged at 94% c. Client with a pulse change of 100 to 88 beats/min d. Client with urine output of 40 mL/hr for the last 2 hours

A

The nurse is caring for a client with leukemia who has he priority problem of fatigue. What action by the client best indicates that an important goal for this problem has been met? A. doing activities of daily living using rest periods B. helping plan a daily activity schedule C. requesting a sleeping pill at night D. telling visitors to leave when fatigued

A

a client is in shock and the nurse prepares to administer insulin for a blood glucose reading of 208 mg/dL the spouse asks why the client needs insulin as the client is not a diabetic. what response by the nurse is best? A. high glucose is common in shock and needs to be treated B. some of the medications we are giving are to raise blood sugar C. the IV solution has lots of glucose, which raises blood sugar D. the stress of this illness has made your spouse a diabetic

A

a client is in the clinic a month after having a MI the client reports sleeping well since moving into the guest bedroom. what response by the nurse is best? A. do you have any concerns about sexuality? B. Im glad to hear you are sleeping well now C. sleep near your spouse in case of emergency D. why would you move into the guest room?

A

a client is on a dopamine infusion via a peripheral line. what action by the nurse takes priority for safety? A. assess the IV site hourly B. monitor the pedal pulses C. monitor the clients vital signs D. obtain consent for a central line

A

a client is receiving norepinephrine (levophed) for shock. what assessment finding best indicates a therapeutic effect from this drug? A. alert and orientated, answering questions B. client denial of chest pain or chest pressure C. IV site without redness or swelling D. urine output of 30 mL/hr for 2 hours

A

a family in the ED is overwhelmed at the loss of several family members due to a shooting incident in the community. which intervention should the nurse complete first? A. provide a calm location for the family to cope and discuss needs B. call the hospital Chaplin to stay with the family and pray for the deceased C. do not allow visiting of the victims until the bodies are prepared D. provide privacy for law enforcement to interview the family

A

a nurse is caring for a client on mechanical ventilation and finds the client agitated and thrashing about what action by the nurse is most appropriate? A. assess the cause of agitation B. reassure the client that he or she is safe C. restrain the clients hands D. sedate the client immediately

A

a nurse is caring for a client who is intubated and has an intra-aortic balloon pump. the client is restless and agitated. what action should the nurse perform first for comfort? A. allow family members to remain at the bedside B. ask the family if the client would like a fan in the room C. keep the TV tuned to the clients favorite channel D. speak loudly to the client in case of hearing problems

A

a nurse teaches a community health class about water safety. which statement by a participant indicates that additional teaching is needed? A. I can go swimming all by myself because I am a certified lifeguard B. i cannot leave my toddler alone in the bathtub for even a min C. i will appoint one adult to supervise the pool at all times during a party D. i will make sure that there is a phone near my pool incase of emergency

A

a nurse wants to become involved in community disaster preparedness and is interested in helping set up and staff and first aid stations or community acute care centers in the event of disaster. which organization is the best fit for this nurses interests? A. the medical reserve corps B. the national guard C. the health department D. a disaster medical assistance team

A

a nurse wants to become part of a disaster medical assistance team (DMAT) but is concerned about maintaning licensure in several different states. which statement best addresses these concerns? A. deployed DMAT providers are federal employees, so their licenses are good in all 50 states B. the gov't has a program for quick licensure activation wherever you are deployed C. during a time of crisis, licensure issues would not be the gov'ts priority concern D. if you are deployed, you will be issued a temp license in the state in which you are working

A

a nursing student is caring for a client who had a myocardial infarction. the student is confused because the client states nothing is wrong and yet listens attentively while the student provides education on lifestyle changes and healthy menu choices. what response by the faculty member is best? A. continue to educate the client on possible healthy changes B. emphasize complications that can occur with noncompliance C. tell the client that denial is normal and will soon go away D. you need to make sure the client understands this illness

A

a provider prescribes a rewarming bath for a client who presents with partial thickness frostbite. which action should the nurse take prior to starting this treatment? A. administer IV morphine B. wrap the limb with compression dressing C. massage the frostbitten areas D. assess the limb for compartment syndrome

A

a pt is prescribed niacin (niaspan) to lower LDL-C and very low density lipoprotein (VLDL) why are lower doses prescribed to the pt? A. to reduce side effects of flushing and feeling warm B. to prevent muscle myopathies C. to prevent elevation of blood pressure D. to prevent undesirable hypokalemia

A

a pts chart notes that the examiner has heard S1 and S2 on auscultation of the heart. what does this documentation refer to? 1st and 2nd heart sounds B. pericardial friction rub C. murmur D. gallop

A

a pts cholesterol screening shows a low density lipoprotein cholesterol (LDL-C) value greater than 190 mg/dL. what is the nurses best interpretation of these results? A. all pts with LDL-C equal to or greater than 190. should be evaluated for secondary causes of hyperlipidemia and treated with statin therapy B. any pt with low LDL-C value should be routinely followed with every 6 month lipid profile values monitoring to see trends in this value C. this pt should be taught to exercise 6-7 days/week to help lower LDL-C D. repeat total cholesterol and LDL-C cholesterol testing during the next routine exam

A

an ED nurse cares for a middle aged mountain climber who is confused and exhibits bizarre behaviors after administering oxygen, which priority intervention should the nurse implement? A. administer dexamethasone (Decadron) B. complete a monumental state examination C. prepare the client for computed tomography of the brain D. request a psychiatric consult

A

an Ed nurse assess a client admitted after a lightning strike. which assessment should the nurse complete first A. electrocardiogram (ECG) B. wound inspection C. creatinine kinase D. computed tomography (CT) of head

A

an older adult is on cardiac monitoring after a myocardial infarction. the client shows frequent dysrhythmias. what action by the nurse is most appropriate? A. assess for any hemodynamic effects of the rhythm B. prepare to administer antidysrhythmic medication C. notify the provider or call the rapid response team D. turn the alarms off on the cardiac monitor

A

in assessing a pt the nurse finds that the point of maximal impulse appears in more than one intercostal space and has shifted lateral to the midclavicular line. how does the nurse interpret this data? A. left ventricular hypertrophy B. superior vena cava obstruction C. pulmonary hypertension D. constrictive pericarditis

A

the advanced practice nurse is assessing the vascular status of a pts lower extremities using the ankle brachial index. what is the correct technique for this assessment method? A a BP cuff is applied to the lower extremities and the systolic pressure is measured by doppler ultrasound at both the dorsalis pedis and posterior tibial pulses B. the dorsalis pedis and posterior tibal pulses are manuall palpated and compared bilaterally for strength and equality and compared to the standard index C. a BP cuff is applied to the lower extremities to observe for an exagerated decrease in systolic pressure by more than 10 mmHg during inspiration D. BP on nthe legs is measured with the pt supine, then the pt stands for several mins and BP is measured in arms

A

the health care provider tells the nurse that a client is to be started on a platelet inhibitor. About what drug does the nurse plan to teach the client? A. Clopidogrel (Plavix) B. Enoxaparin (Lovenox) C. Reteplase (Retavase) D. Warfarin (Coumadin)

A

the hospital administration arranges for critical incident stress debriefing for the staff after a mass casualty incident. which statement by the debriefing team leader is most appropriate for this situation? A. you are free to express your feelings; whatever is said here stays here B. lets evaluate what went wrong and develop policies for future incidents C. this session is only for nursing and medical staff, not for ancillary personnel D. lets pass around the written policy compliance from for everyone

A

the nurse is performing a cardiac assessment on an older adult. what is a common assessment finding for this pt? A S4 heart sound B. leg edema C. pericardial friction rub D. change in point of maximum impulse location

A

the nurse is providing health teaching for a pt at risk for heart disease. which factor is the most modifiable, controllable risk factor A. obesity B. diabetes C. ethnic background D. family history of cardiovascular disease

A

the nurse is reviewing preliminary ECG results of a pt admitted for mental status changes. the nurse alerts the health care provider about ST elevation or depression in the pt because it is an indication of which condition? A. myocardial injury or ischemia B. ventricular irritability C. subarachnoid hemorrhage D. prinzmetal's angina

A

the nurse is talking to a pt who has been trying to quit smoking. which statement by the pt indicates an understanding of cigarette usage as it relates to reducing cardiovascular risks A. i need to be completely cigarette-free for at least 3 years B. i don't smoke as much as i used to, i'm down to one pack/day C. i started smoking a while ago but ill quit in a couple of years D. i only smoke to relax when i drink or go out with friends

A

the nursing student is assisting in the care of a pt with advanced right sided heart failure. in addition to bringing a stethoscope what additional piece of equipment does the student bring in order to assess this pt? A. tape measure B. glasgow coma scale C. portable doppler D. bladder ultrasound scanner

A

the terminally ill pt is nearing death. his wife expresses concern that he has no appetite and eats very little. what is the nurse's best responce to this concern? A. teach the wife about risk of aspiration and explain that loss of appetite is normal when a pt nears death B. encourage the wife to feed the pt as much as he will take to maintain nutrition C. request the health care provider order a dietary nutrition consult to include foods that the pt perfers D. keep fluids and finger foods at the bedside for easy access whenever the pt is hungry or thirsty

A

what does the P wave in a ECG represent A. atrial depolarization B. atrial repolarazation C. ventricular depolarization D. ventricular repolarizaation

A

which statement about the peripheral vascular system is true A. veins are equipped with valves that direct blood flow to the heart and prevent backflow B. the velocity of blood flow depends on the diameter of the vessel lumen C. blood flow decreases and blood tends to clot as the viscosity decreases D. the parasympathetic nervous system has the largest effect on blood flow to organs

A

while on a camping trip, a nurse cares for an adult client who had a drowning incident in a lake and is experiencing agonal breathing with a palpable pulse. Which action should the nurse take first? A. deliver rescue breaths B. wrap the client in dry blankets C. assess for signs of bleeding D. check for a carotid pulse

A

A nurse delegates care for a client who has open skin lesions. Which statements should the nurse include when delegating this clients hygiene care to an unlicensed assistive personnel (UAP)? select all that apply A. wash your hands before touching the client B. wear gloves when bathing the client C. assess skin for breakdown during the bath D. apply lotion to lesions while the skin is wet E. use a damp cloth to scrub the lesions

A, B

A nurse manages wound care for clients on a medical-surgical unit. Which client wounds are paired with the appropriate treatments? select all that apply A. client with a left heel ulcer with slight necrosis whirlpool treatments B. client with an eschar-covered sacral ulcer surgical dbridement C. client with a sunburn and erythema soaking in warm water for 20 minutes D. client with urticaria wet-to-dry dressing changes every 6 hours E. client with a sacral ulcer with purulent drainage transparent film dressing

A, B

A nurse teaches a client to perform total skin self-examinations on a monthly basis. Which statements should the nurse include in this clients teaching? select all that apply. A. A look for asymmetry of shape and irregular borders B. Assess for color variation within each lesion C. examine the distribution of lesions over a section of the body D. monitor for edema or swelling of tissues E. focus your assessment on skin areas that itch

A, B

The nurse caring frequently for older adults in the hospital is aware of risk factors that place them at a higher risk for shock. For what factors would the nurse assess? (Select all that apply.) a. Altered mobility/immobility b. Decreased thirst response c. Diminished immune response d. Malnutrition e. Overhydration

A, B, C, D

the nurse caring for mechanically ventilated clients uses best practices to prevent ventilator-associated pneumonia. what actions are included in this practice? A. adherence to proper hand hygiene B. administering anti-ulcer medication C. elevating the head of the bed D. providing oral care per protocol E. suctioning the client on a regular schedule

A, B, C, D

the nurse is caring for mechanically ventilated clients knows that older adults are at higher risk for weaning failure. what age related changes contribute to this? select all that apply A. chest wall stiffness B. decreased muscle strength . inability to cooperate D. less lung elasticity E. poor vision and hearing

A, B, D

A nurse plans care for a client with burn injuries. Which interventions should the nurse implement to prevent infection in the client? Select all that apply. A. ask all family members and visitors to perform hand hygiene before touching the client B. carefully monitor burn wounds when providing each dressing change C. clean equipment with alcohol between uses with each client on the unit D. allow family members to only bring the client plants from the hospital gift shop E. use aseptic technique and wear gloves when performing wound care

A, B, E

A student nurse learns that the spleen has several functions. What functions do they include? Select all that apply. A. breaks down hemoglobin B. destroys old or defective RBCs C. forms vitamin K for clotting D. stores extra iron in ferritin E. stores platelets not circulating

A, B, E

a nurse teaches a client who has sever allergies to prevent bug bites. which statements should the nurse include in this clients training? select all that apply. A. consult an exterminator to control bugs in and around your home B. do not swat at insects or wasps C. wear sandals whenever you go outside D. keep your prescribed epinephrine auto injector in a bedside drawer E. use screens in your windows and doors to prevent flying insects from entering

A, B, E

an ED nurse is preparing to transfer a client to the trauma ICU which info should the nurse include in the nurse to nurse hand-off report? select all that apply A. mechanism of injury B. diagnostic test results C. immunizations D. list of home medications E. isolation precautions

A, B, E

A nurse cares for an older client with burn injuries. Which age-related changes are paired appropriately with their complications from the burn injuries? Select all that apply? A. slower healing time increased risk for loss of function from contracture formation B. reduced inflammatory response deep partial thickness wound with minimal exposure C. reduced thoracic compliance increased risk for atelectasis D. high incidence of cardiac impairments increased risk for acute kidney injury E. thinner skin may not exhibit a fever when infection is present

A, C, D

A nurse plans care for a client who has a wound that is not healing. Which focused assessments should the nurse complete to develop the clients plan of care? select all that apply. A. Height B. Allergies C. Alcohol use D. Prealbumin laboratory results E. Liver enzyme laboratory results

A, C, D

the nurse caring for hospitalized clients includes which actions on their care plans to reduce the possibility of the clients developing shock? select all that apply A. assessing and identifying clients at risk B. monitoring the daily white blood cell count C. performing proper hand hygiene D. removing invasive lines as soon as possible E. using aseptic technique during procedures

A, C, D, E

a nurse is providing health education at a community center. which instructions should the nurse include in teaching about prevention of lightning injuries during a storm? select all that apply. A. seek shelter inside a building or vehicle B. hide under a tall tree C. do not take a bath or shower D. turn off the TV E. remove all body piercings F. put down golf clubs or gardening tools

A, C, D, F

emergency medical services (EMS) brings a large number of clients to the ED following a mass casualty incident. the nurse identifies the clients with which injuries with yellow tags? select all that apply A. partial thickness burns covering both legs B. open fractures of both legs with absent pedal pulses C. neck injury and numbness of both legs D. small pieces of shrapnel embedded in both eyes E. head injury and difficult to arouse F. bruising and pain in the right lower abdomen

A, C, D, F

a nurse is caring for a client who has coronary artery bypass grafting yesterday. what actions does the nurse delegate to the unlicensed assistive personnel (UAP)? select all that apply A. assist the client to the chair for meals and to the bathroom B. encourage the client to use the spirometer every 4 hours C. ensure the client wears TED house or sequential compression devices D. have the client rate pain on a 0-10 scale and report to the nurse E. take and record a full set of vital signs per hospital protocol

A, C, E

the nurse is assessing a pt with right sided heart failure which assessment findings does the nurse expect to see in this pt. select all that apply? A. dependent edema B. weight loss C. jugular venous distension D. hypotension E. hypotension F. hepatomegaly F. angina

A, C, E

a client is in the early stages of shock and is restless. what comfort measures does the nurse delegate to the nursing student? select all that apply A. bringing the client warm blankets B. giving the client hot tea to drink C. massaging the clients painful legs D. reorienting the client as needed E. sitting with the client for reassurance

A, D, E

a nurse is teaching a wilderness survival class. which statements should the nurse include about the prevention of hypothermia and frostbite? select all that apply A. wear synthetic clothing instead of cotton to keep your skin dry B. drink plenty of fluids brandy can be used to keep your body warm C. remove your hat when exercising to prevent the loss of heat D. wear sunglasses to protect skin and eyes from harmful rays E. know your physical limits. come in out of the cold when limits are reached

A, D, E

an emergency department nurse plans care for a client who is admitted with heat stroke. which interventions should the nurse include in this clients plan of care? select all that apply A. administer o2 via mask or nasal cannula B. administer ibuprofen, an antipyretic med C. apply cooling techniques until core body temp is less than 101 F D. infuse 0.9% sodium chloride via a large bore IV cannula E. obtain baseline serum electrolytes and cardiac enzymes

A, D, E

A nurse plans care for a client who is immobile. Which interventions should the nurse include in this clients plan of care to prevent pressure sores? select all that apply. A. place a small pillow between bony surfaces B. elevate the head of the bed to 45 degrees C. limit fluids and proteins in the diet D. use a life sheet to assist with re-positioning E. re-position the client who is in a chair every 2 hours F. keep the clients heels off the bed surfaces G use a rubber ring to decrease sacral pressure when up in the chair

A, D, F

the complex care provided during an emergency requires interdisciplinary collaboration. which interdisciplinary team members are paired with the correct responsibilities? select all that apply A. psychatric crisis nurse interacts with clients and families when sudden illness, serious injury or death of a loved one may cause a crisis B. forensic nurse examiner performs rapid assessments to ensure clients with the highest acuity receive the quickest evaluation, treatment and prioritization of resources C. triage nurse provides basic life support interventions such as o2, basic wound care, splinting, spinal immobilization and monitoring of vital signs D. EMT obtains client histories, collects evidence, and offers counseling and follow up care for victims of rape, child abuse, and domestic violence E. paramedic provides prehospital advanced life support, including cardiac monitoring, advanced airway management, and medication administration

A, E

atherosclerousis affects which larger arteries? select all that apply A. renal B. femoral C. coronary D. brachial cephalic E. aorta F. carotid

A,B,C,E,F

which statements about BP are accurate? select all that apply A. pulse pressure is the difference between the systolic and diastolic pressures B. the right ventricle of the heart generates the greatest amount of blood pressure C. diastolic blood pressure is primarily determined by the amount of peripheral vasoconstriction D. to maintain adequate blood flow through the coronary arteries, mean arterial pressure (MAP) must be at least 60 E. diastolic blood pressure is the highest pressure during contraction of the ventricles F. systolic blood pressure is the amount of pressure/force generated by the left ventricle to distribute blood into the aorta with each contraction of the heart

A,C,D,F

which actions are the responsibilities of the monitor technician? select all that apply A. watch the bank of monitors on a unit B. notify the health care provider of any changes C. print routine ECG strips D. apply battery operated transmitter leads to patients E. interpret the rhythms F. report patient rhythm and significant changes to the nurse

A,C,E,F

a pt is admitted with a vascular problem. based on the pathophysiology of systemic arterial pressure, the systemic arterial pressure is a product of what factures? select all that apply A. cardiac output B. norepinephrine C. preload D. total peripheral vascular resistance E. diastolic blood pressure F. afterload

A,D

Changes from normal pH can have what effects on which body functions? Select all that apply: A. Alter fluid and electrolyte balance B. Increase effectiveness of drugs C. Reduce function of hormones D. Increase function of enzymes E. Increase excitability of the heart muscle F. Cause increased activity of the GI tract

A. Alter fluid and electrolyte balance C. Reduce function of hormones E. Increase excitability of the heart muscle F. Cause increased activity of the GI tract

24. The nurse who is starting the shift finds a client with an IV that is leaking all over the bed linens. What does the nurse do initially? A. Assess the insertion site. B.Check connections. C.Check the infusion rate. D.Discontinue the IV and start another.

A. Assess the insertion site. The initial response by the nurse is to assess the insertion site. The purpose of this action is to check for patency, which is the priority. IV assessments typically begin at the insertion site and move "up" the line from the insertion site to the tubing, to the tubing's connection to the bag.Checking the IV connection is important, but is not the priority in this situation. Checking the infusion rate is not the priority. Discontinuing the IV to start another may be required, but it may be possible to "save" the IV, and the problem may be positional or involve a loose connection.

Which system should be assessed first for a patient at risk for acidosis? A. Cardiovascular system B. Neuromuscular system C. CNS D. Respiratory system

A. Cardiovascular system

The nurse is interpreting the ABG results of a patient with acute respiratory insufficiency. As the PaCO2 level increases, which result would the nurse expect? A. Decreased pH B. Decreased Bicarb C. Increased PaO2 D. Increased pH

A. Decreased pH

A pt who has a decreased amount of hydrogen ions and a decreased amount of carbon dioxide in the body will have what response? A. Decreased rate and depth of respirations B. Decreased renal absorption of hydrogen ions C. Increased rate and depth of respirations D. Decreased renal excretions of bicarbonate

A. Decreased rate and depth of respirations

The nurse monitors the effectiveness of magnesium sulfate by assessing which factor every hour? A. Deep tendon reflexes B. Vital signs C. Serum laboratory values D. Urine output

A. Deep tendon reflexes

The pt with mild fluid volume overload has been instructed by the provider to follow dietary sodium restriction. What would the nurse teach this patient about sodium restriction? A. Do not add salt to ordinary table foods B. Restrict sodium intake to 2gm per day C. Restrict sodium intake to 4mg per day D. Do not add salt when cooking or eating

A. Do not add salt to ordinary table foods

The nurse is giving discharge instructions to the pt with advanced heart failure who is at continued risk for fluid volume overload. For which physical change does the nurse instruct the patient to call the health care provider. A. Greater than 3 lbs gained in a week or greater than 1-2 lbs gained in a 24 hour period B. Greater than 5 lbs gained in a week or greater than 1-2 lbs gained in a 24 hour period C. Greater than 15 lbs gained in a month or greater than 5 lbs in a week D. Greater than 20lbs gained in a month or greater than 5 lbs gained in a week

A. Greater than 3 lbs gained in a week or greater than 1-2 lbs gained in a 24 hour period

A pt with hyponatremia would have which GI findings upon assessment? A. Hyperactive bowel sounds on auscultation B. Hard, dark brown stools C. Hypoactive bowel sounds on auscultation D. Frequent watery bowl movements E. Abdominal cramping F. Nausea

A. Hyperactive bowel sounds on auscultation D. Frequent watery bowl movements E. Abdominal cramping F. Nausea

The nurse is taking care of a trauma patient who was in a motor vehicle accident. The patient has a history of hypertension, which is managed with spironolactone. This pt is at risk for developing which electrolyte balance? A. Hyperkalemia B. Hypernatremia C. Hypokalemia D. Hypocalcemia

A. Hyperkalemia

A pt with a recent history of anterior neck injury reports muscle twitching and spasms with tingling in the lips, nose, and ears. The nurse suspects these symptoms may be caused by which condition? A. Hypocalcemia B. Hypokalemia C. Hyponatremia D. Hypomagnesemia

A. Hypocalcemia

On assessment, the pt has respiratory muscle weakness resulting in shallow respirations. Which electrolyte abnormality would the nurse expect. A. Hypokalemia B. Hyperkalemia C. Hypocalcemia D. Hypercalcemia

A. Hypokalemia

Which assessment findings are related to mild hypercalcemia? Select all that apply: A. Increased heart rate B. Parasthesia C. Decreased deep tendon reflexes D. Hypoactive bowel sounds E. Shortened QT interval F. Profound muscle weakness

A. Increased heart rate C. Decreased deep tendon reflexes D. Hypoactive bowel sounds E. Shortened QT interval

Which findings indicate that a patient may have hypervolemia? Select all that apply: A. Increased, bounding pulse B. Jugular venous distention C. Presence of crackles D. Excessive thirst E. Elevated blood pressure F. Orthostatic Hypotension

A. Increased, bounding pulse B. Jugular venous distention C. Presence of crackles E. Elevated blood pressure

The nurse is preparing to give a patient IV drug therapy. What information does the nurse need before administering the drug? A. Indications, contraindications, and precautions for IV therapy. B. Appropriate dilution, pH, and osmolarity of solution C. Rate of infusion and dosage of drugs D. Compatibility with other IV medications E. Percentage of adverse events for the drug F. Specifics of monitoring because of immediate effect

A. Indications, contraindications, and precautions for IV therapy. B. Appropriate dilution, pH, and osmolarity of solution C. Rate of infusion and dosage of drugs D. Compatibility with other IV medications F. Specifics of monitoring because of immediate effect

A pt with COPD has just developed respiratory distress. Pulse Ox 88% on 2 L Nasal cannula oxygen, dyspnea at rest, and resps are 32 per minute. The pt reports SOB. Which statements apply to the clinical situation? Select all that apply: A. Interference in alveolar-capillary diffusion results in carbon dioxide retention. B. The nurse should instruct the pt to use pursed-lip breathing. C. Position the pt with the head of the bed at less than 20 degrees. D. Interference in alveolar-capillary diffusion results in acidemia. E. The nurse should explain to the pt that rapid breathing will relieve the SOB. F. Use of an as-needed bronchodilator may relieve the shortness of breath (SOB).

A. Interference in alveolar-capillary diffusion results in carbon dioxide retention. B. The nurse should instruct the pt to use pursed-lip breathing. D. Interference in alveolar-capillary diffusion results in acidemia. F. Use of an as-needed bronchodilator may relieve the shortness of breath (SOB).

Which statement best explains how antidiuretic hormone ADH affects urine output? A. It increases permeability to water in the tubules, causing a decrease in urine output? B. It increases urine output as a result of water being absorbed by the tubules. C. Urine output is reduced as the posterior pituitary decreases ADH production D. Increased urine output results from increased osmolarity and fluid in the ECF

A. It increases permeability to water in the tubules, causing a decrease in urine output?

What interventions are included in the plan of care for a pt with metabolic ketoacidosis? Select all that apply: A. Monitor ABG levels for decreasing pH level B. Maintain patent IV access C. Administer fluids as prescribed D. Monitor for irritability and muscle tetany E. Monitor loss of bicarbonate through the GI tract such as diarrhea F. Administer IV 50% dectrose as needed

A. Monitor ABG levels for decreasing pH level B. Maintain patent IV access C. Administer fluids as prescribed E. Monitor loss of bicarbonate through the GI tract such as diarrhea

The nurse is assessing a pt with a mild increase in sodium level. What early manifestation does the nurse observe in this patient? A. Muscle twitching and irregular muscle contractions B. Inability of muscles and nerves to respond to stimulus C. Muscle weakness occuring bilaterally with no specific pattern D. Reduced or absent bilateral deep tendon reflexes

A. Muscle twitching and irregular muscle contractions

Which are typical nursing assessment findings for a pt with hypocalcemia? Select all that apply: A. Positive Chvostek's sign B. Hypertension C. Diarrhea D. Prolonged ST interval E. Elevated T wave F. Positive Trousseau's sign

A. Positive Chvostek's sign C. Diarrhea D. Prolonged ST interval F. Positive Trousseau's sign

Which pts are at risk for developing hyponatremia? Select all that apply: A. Postoperative patient who has been NPO for 24 hours with no IV fluid infusiong B. Pts with decreased fluid intake for three days C. Pts receiving excessive IV fluids with 5% dextrose D. Diabetic pt with blood glucose of 250 mg/dL E. Patient with overactive adrenal glands F. Tennis player in 100 F (37.7) weather who has been drinking water

A. Postoperative patient who has been NPO for 24 hours with no IV fluid infusiong C. IV fluids with 5% dextrose D. Diabetic pt with blood glucose of 250 mg/dL F. Tennis player in 100 F (37.7) weather who has been drinking water

Which pt is at risk for excess insensible water loss? A. Pt with continuous GI suctioning B. Pt with slow, deep respirations C. Pt receiving oxygen therapy D. Patient with hypothermia

A. Pt with continuous GI suctioning

Which pt is most likely to have a decrease in bicarbonate? A. Pt with pancreatitis B. Pt with hypoventilation C. Pt who is vomiting D. Pt with emphysema

A. Pt with pancreatitis

14. The nurse is starting a peripheral IV catheter on a recently admitted client. What actions does the nurse perform before insertion of the line? Select all that apply. A.Apply povidone-iodine to clean skin, dry for 2 minutes. B.Clean the skin around the site. C.Prepare the skin with 70% alcohol or chlorhexidine. D.Shave the hair around the area of insertion. E.Wear clean gloves and touch the site only with fingertips after applying antiseptics.

A.Apply povidone-iodine to clean skin, dry for 2 minutes. B.Clean the skin around the site. C.Prepare the skin with 70% alcohol or chlorhexidine. Povidone-iodine (Betadine) is applied to the selected insertion site before insertion. The solution is allowed to dry, which takes about 2 minutes. The insertion site should be cleansed before the antiseptic skin preparations are completed. After soap and water cleansing, prepping with 70% alcohol or chlorhexidine is done.Clipping, rather than shaving, hair around the selected IV site is done. Shaving is abrasive and makes the skin more vulnerable to infection (i.e., microbial invasion). The insertion site should not be palpated again after it has been prepped; this mistake is frequently made with IV starts.

A nurse teaches a client who is diagnosed with diabetes mellitus. Which statement should the nurse include in this client's plan of care to delay the onset of microvascular and macrovascular complications? a. "Maintain tight glycemic control and prevent hyperglycemia." b. "Restrict your fluid intake to no more than 2 liters a day." c. "Prevent hypoglycemia by eating a bedtime snack." d. "Limit your intake of protein to prevent ketoacidosis."

ANS: A Hyperglycemia is a critical factor in the pathogenesis of long-term diabetic complications. Maintaining tight glycemic control will help delay the onset of complications. Restricting fluid intake is not part of the treatment plan for clients with diabetes. Preventing hypoglycemia and ketosis, although important, are not as important as maintaining daily glycemic control.

A nurse prepares to administer insulin to a client at 1800. The client's medication administration record contains the following information: • Insulin glargine: 12 units daily at 1800 • Regular insulin: 6 units QID at 0600, 1200, 1800, 2400 Based on the client's medication administration record, which action should the nurse take? a. Draw up and inject the insulin glargine first, and then draw up and inject the regular insulin. b. Draw up and inject the insulin glargine first, wait 20 minutes, and then draw up and inject the regular insulin. c. First draw up the dose of regular insulin, then draw up the dose of insulin glargine in the same syringe, mix, and inject the two insulins together. d. First draw up the dose of insulin glargine, then draw up the dose of regular insulin in the same syringe, mix, and inject the two insulins together.

ANS: A Insulin glargine must not be diluted or mixed with any other insulin or solution. Mixing results in an unpredictable alteration in the onset of action and time to peak action. The correct instruction is to draw up and inject first the glargine and then the regular insulin right afterward.

A nurse cares for a client with diabetes mellitus who is visually impaired. The client asks, "Can I ask my niece to prefill my syringes and then store them for later use when I need them?" How should the nurse respond? a. "Yes. Prefilled syringes can be stored for 3 weeks in the refrigerator in a vertical position with the needle pointing up." b. "Yes. Syringes can be filled with insulin and stored for a month in a location that is protected from light." c. "Insulin reacts with plastic, so prefilled syringes are okay, but you will need to use glass syringes." d. "No. Insulin syringes cannot be prefilled and stored for any length of time outside of the container."

ANS: A Insulin is relatively stable when stored in a cool, dry place away from light. When refrigerated, prefilled plastic syringes are stable for up to 3 weeks. They should be stored in the refrigerator in the vertical position with the needle pointing up to prevent suspended insulin particles from clogging the needle.

A client receiving muromonab-CD3 (Orthoclone OKT3) asks the nurse how the drug works. What response by the nurse is best? a. "It increases the elimination of T lymphocytes from circulation." b. "It inhibits cytokine production in most lymphocytes." c. "It prevents DNA synthesis, stopping cell division in activated lymphocytes." d. "It prevents the activation of the lymphocytes responsible for rejection."

ANS: A Muromonab-CD3 (Orthoclone OKT3) is a monoclonal antibody that works to increase the elimination of T lymphocytes from circulation. The corticosteroids broadly inhibit cytokine production in most leukocytes, resulting in generalized immunosuppression. The main action of all antiproliferatives (such as azathioprine [Imuran]) is to inhibit something essential to DNA synthesis, which prevents cell division in activated lymphocytes. Calcineurin inhibitors such as cyclosporine (Sandimmune) stop the production and secretion of interleukin-2, which then prevents the activation of lymphocytes involved in transplant rejection.

A nurse is providing community education on the seven warning signs of cancer. Which signs are included? (Select all that apply.) a. A sore that does not heal b. Changes in menstrual patterns c. Indigestion or trouble swallowing d. Near-daily abdominal pain e. Obvious change in a mole

ANS: A, B, C, E The seven warning signs for cancer can be remembered with the acronym CAUTION: changes in bowel or bladder habits, a sore that does not heal, unusual bleeding or discharge, thickening or lump in the breast or elsewhere, indigestion or difficulty swallowing, obvious change in a wart or mole, and nagging cough or hoarseness. Abdominal pain is not a warning sign.

A nurse cares for a client who has diabetes mellitus. The nurse administers 6 units of regular insulin and 10 units of NPH insulin at 0700. At which time should the nurse assess the client for potential problems related to the NPH insulin? a. 0800 b. 1600 c. 2000 d. 2300

ANS: B Neutral protamine Hagedorn (NPH) is an intermediate-acting insulin with an onset of 1.5 hours, peak of 4 to 12 hours, and duration of action of 22 hours. Checking the client at 0800 would be too soon. Checking the client at 2000 and 2300 would be too late. The nurse should check the client at 1600.

The student nurse learns that the most important function of inflammation and immunity is which purpose? a. Destroying bacteria before damage occurs b. Preventing any entry of foreign material c. Providing protection against invading organisms d. Regulating the process of self-tolerance

ANS: C The purpose of inflammation and immunity is to provide protection to the body against invading organisms, whether they are bacterial, viral, protozoal, or fungal. These systems eliminate, destroy, or neutralize the offending agents. The cells of the immune system are the only cells that can distinguish self from non-self. This function is generalized and incorporates destroying bacteria, preventing entry of foreign invaders, and regulating self-tolerance.

The nursing instructor explains the difference between normal cells and benign tumor cells. What information does the instructor provide about these cells? a. Benign tumors grow through invasion of other tissue. b. Benign tumors have lost their cellular regulation from contact inhibition. c. Growing in the wrong place or time is typical of benign tumors. d. The loss of characteristics of the parent cells is called anaplasia.

ANS: C Benign tumors are basically normal cells growing in the wrong place or at the wrong time. Benign cells grow through hyperplasia, not invasion. Benign tumor cells retain contact inhibition. Anaplasia is a characteristic of cancer cells.

A nurse has taught a client about dietary changes that can reduce the chances of developing cancer. What statement by the client indicates the nurse needs to provide additional teaching? a. "Foods high in vitamin A and vitamin C are important." b. "I'll have to cut down on the amount of bacon I eat." c. "I'm so glad I don't have to give up my juicy steaks." d. "Vegetables, fruit, and high-fiber grains are important."

ANS: C To decrease the risk of developing cancer, one should cut down on the consumption of red meats and animal fat. The other statements are correct.

After teaching a young adult client who is newly diagnosed with type 1 diabetes mellitus, the nurse assesses the client's understanding. Which statement made by the client indicates a correct understanding of the need for eye examinations? a. "At my age, I should continue seeing the ophthalmologist as I usually do." b. "I will see the eye doctor when I have a vision problem and yearly after age 40." c. "My vision will change quickly. I should see the ophthalmologist twice a year." d. "Diabetes can cause blindness, so I should see the ophthalmologist yearly."

ANS: D Diabetic retinopathy is a leading cause of blindness in North America. All clients with diabetes, regardless of age, should be examined by an ophthalmologist (rather than an optometrist or optician) at diagnosis and at least yearly thereafter.

An older adult has a mild temperature, night sweats, and productive cough. The client's tuberculin test comes back negative. What action by the nurse is best? a. Recommend a pneumonia vaccination. b. Teach the client about viral infections. c. Tell the client to rest and drink plenty of fluids. d. Treat the client as if he or she has tuberculosis (TB).

ANS: D Due to an age-related decrease in circulating T lymphocytes, the older adult may have a falsely negative TB test. With signs and symptoms of TB, the nurse treats the client as if he or she does have TB. A pneumonia vaccination is not warranted at this time. TB is not a viral infection. The client should rest and drink plenty of fluids, but this is not the best answer as it does not address the possibility that the client's TB test could be a false negative.

A client is in the oncology clinic for a first visit since being diagnosed with cancer. The nurse reads in the client's chart that the cancer classification is TISN0M0. What does the nurse conclude about this client's cancer? a. The primary site of the cancer cannot be determined. b. Regional lymph nodes could not be assessed. c. There are multiple lymph nodes involved already. d. There are no distant metastases noted in the report.

ANS: D TIS stands for carcinoma in situ; N0 stands for no regional lymph node metastasis; and M0 stands for no distant metastasis.

A client is having a bone marrow biopsy today. What action by the nurse takes priority? A. administer pain medication first B. ensure valid consent is on the chart C. have the client shower in the morning D. premediate the client with sedatives

B

A client with Sjgrens syndrome reports dry skin, eyes, mouth, and vagina. What nonpharmacologic comfort measure does the nurse suggest? A. frequent eyedrops B. home humidifier C. strong moisturizer D. tear duct plugs

B

A nurse evaluates the following data in a clients chart: Admission note-66 year old male with a health history of a cerebral vascular accident and left sided paralysis. Lab results-WBC count 8000/mm3, prealbumin: 15.2 mg/dL, Albumin: 4.2 mg/dL, lymphocyte count: 2000/mm3. Wound care note-sacral ulcer 4cm, 2cm, 1.5 cm Based on this information, which action should the nurse take? A. Perform a neuromuscular assessment B. request a dietary consult C. Initiate contact precautions D. assess the clients vital signs

B

After educating a caregiver of a home care client, a nurse assesses the caregiver's understanding. Which statement indicates that the caregiver needs additional education? a."I can help him shift his position every hour when he sits in the chair." b."If his tailbone is red and tender in the morning, I will massage it with baby oil." c."Applying lotion to his arms and legs every evening will decrease dryness." d."Drinking a nutritional supplement between meals will help maintain his weight."

B

After teaching a client who has psoriasis, a nurse assesses the clients understanding. Which statement indicates the client needs additional teaching? A. at the next family reunion, I'm going to ask my relatives if they have psoriasis B. I have to make sure I keep my lesions covered, so I do not spread this to others C. I expect that these patches will get smaller when I lie out in the sun D. I should continue to use the cortisone ointment as the patches shrink and dry out

B

a client has hemodynamic monitoring after a myocardial infarction. what safety precaution does the nurse implement for this client? A. document pulmonary artery wedge pressure (PAWP) readings and assess their trends C. ensure the balloon does not remain wedged C. keep the client on strict NPO status D. maintain the client in a semi-fowlers position

B

a client has intra-arterial blood pressure monitoring after a MI. the nurse notes the clients heart rate has increased from 88 to 110 beats/min and the blood pressure dropped from 120/82 to 100/60 mm Hg what action by the nurse is most appropriate? A. allow the client to rest quietly B. assess the client for bleeding C. document the findings in the chart D. medicate the client for pain

B

a client has presented to the emergency department with an acute MI what action by the nurse is best to meet the joint commissions core measures outcomes? A. obtain an electrocardiogram (ECG) now and in the morning B. give the client an aspirin C. notify the rapid response team D. prepare to administer thrombolytics

B

a client is being discharged home after a large myocardial infarction and subsequent coronary artery bypass grafting surgery. the clients sternal wound has not yet healed. what statement by the client most indicates a higher risk of developing sepsis after discharge? A. all my friends and neighbors are planning a party for me B. I hope I can get my water turned back on when I get home C. I am going to have my daughter scoop the cat litter bod D. my grandkids are so excited to have me coming home!

B

a client is being discharged soon on warfarin (coumadin) what menu selection for dinner indicates the client needs more education regarding this medication? A. hamburger and french fries B. large chefs salad and muffin C. no selection; spouse brings pizza D. tuna salad sandwich and chips

B

a client is hospitalized with a second episode of PE recent genetic testing reveals the client has an alteraction in the gene CYP2C19. what action by the nurse is best? A. instruct the client to eliminate all vitamin K from the diet B. prepare preoperative teaching for an inferior vena cava (IVC) filter C. refer the client to a chronic illness support group D. teach the client to use a soft bristled toothbrush

B

a client is in the hospital after suffering a myocardial infarction and has bathroom privileges. the nurse assists the client to the bathroom and notes the clients O2 saturation to be 95% pulse 88 bpm, and respiratory rate 16 breaths/min after returning to bed. what action by the nurse is best? A. administer oxygen at 2 L/min B. allow continued bathroom privileges C. obtain a bedside commode D. suggest the client use a bedpan

B

a client is on IV heparin to treat a pulmonary embolism. the clients most recent partial thromboplastin time (PTT) was 25 seconds. what order should the nurse anticipate? A. decrease the heparin rate B. increase the heparin rate C. no change to the heparin rate D. stop heparin; start warfarin (coumadin)

B

a client presents to the ED after prolonged exposure to the cold. the client is difficult to arouse and speech is incoherent which action should the nurse take first? A. reposition the client into a prone position B. administer warmed IV fluids to the client C. wrap the clients extremities in warm blankets D. initiate extracorporeal rewarming via hemodialysis

B

a client received tissue plasminogen activator (t-PA) after a myocardial infarction and now is on an intravenous infusion of heparin. the clients spouse asks why the client needs this medication. what response by the nurse is best? A. the t-PA didnt dissolve the entire coronary clot B. the heparin keeps that artery from getting blocked again C. heparin keeps the blood as thin as possible for a longer time D. the heparin prevents a stroke from occurring as the t-PA wears off

B

a client with coronary artery disease (CAD) ask the nurse about taking fish oil supplements. what response by the nurse is best? A. fish oil is contraindicated with most drugs for CAD B. the best source is fish, but pills have benefits too C. there is no evidence to support fish oil use with CAD D. you can reverse CAD totally with diet and supplements

B

a dying pt is receiving morphine for severe pain. the health care provider informs the nurse that the pt is at risk for acute renal failure. what assessment will the nurse perform in order to determine if the kidney is failing to excrete the morphine metabolites? A. assess the pt for adequate pain relief B. observe for signs of confusion or delirium C. auscultate the lungs for crackles or wheezes D. observe the color, clarity, and amount of urine

B

a family member calls the nurse into the pts room and says i think mom just died what should the nurse do first? A. notify the nurseing supervisor to have the body moved to the morgue B. ascertain that the pt does not rouse to verbal or tactile stimuli C. make sure that the health care provider has completed and signed the death certificate D. provide privacy for the family and significant others with the deceased

B

a home health care nurse is visiting an older client who lives alone after being discharged from the hospital after a coronary artery bypass graft. what finding in the home most causes the nurse to consider additional referrals? A. dirty carpets in need of vacuuming B. expired food in the refrigerator C. old medications in the kitchen D. several cats present in the home

B

a hospice pt is deteriorating and the family is concerened about the restlessness and agitation. which intervention should the nurse perform? A. notify the primary health care provider and request orders for transfer to the hospital B. assess for pain, provide analgestics and make the pt as comfortable as possible C. initiate IV hydration to provide the pt with necessary fluids D. encourage the family to assist the pt to eat in order to gain energy

B

a nurse answers a call light and finds a client anxious, SOB, reporting chest pain and having a BP of 88/52 mm Hg on the cardiac monitor. what action by the nurse takes priorty? A. assess the clients lung sounds B. notify the rapid response team C. provide reassurance to the client D. take a full set of vital signs

B

a nurse is caring for a client after surgery who is restless and apprehensive. the UAP reports the vital signs and the nurse sees they are only slightly different from previous readings. what action does the nurse delegate next to the UAP? A. assess the client for pain or discomfort B. measure urine output from the catheter C. reposition the client to the unaffected side D. stay with the client and reassure him or her

B

a nurse is caring for a client after surgery. the clients respiratory rate has increased from 12 to 18 breaths/min and the pulse rate increased from 86 to 98 bpm since they were last assessed 4 hours ago. what action by the nurse is best? A. ask if the client needs pain meds B. assess the clients tissue perfusion further C. document the findings in the clients chart D. increase the rate of the clients IV infusion

B

a nurse is caring for four clients on IV heparin therapy. which lab value possibly indicates that a serious side effect has occurred? A. hemoglobin: 14.2 g/dL B. platelet count: 82,000/L C. RBC count 4.8/mm3 D. WBC count 8.7/mm3

B

a nurse is caring for four clients which client should the nurse assess first? A. client with an acute MI, pulse 102 bpm B. client who is 1 hour post angioplasty, has tongue swelling and anxiety C. client who is post coronary artery bypass, chest tube drained 100 mL/hr D. client who is post coronary artery bypass, potassium 4.2 mEq/L

B

a nurse is caring for several clients at risk for shock. which lab value requires the nurse to communicate with the health care provider? A. creatinine: 0.9 mg/dL B. lactate: 6 mmol/L C. sodium: 150 mEq/L D. WBC count 11,000/mm3

B

a nurse is evaluating levels and functions of trauma centers. which function is appropriately paired with the level of the trauma center? A. level 1 located within remote areas and provides advanced life support within resource capabilities B. level 2 located within community hospitals and provides care to most injured clients C. level 3 located in rural communities and provides only basic care to clients D. level 4 located in large teaching hospitals and provides a full continuum of trauma care for all clients

B

a nurse plans care for a client admitted with a snakebite to the right leg. with whom should the nurse collaborate? A. the facility's neurologist B. the poison control center C. the physical therapy department D. a herpetologist (snake specialist)

B

a nurse works at a community center for older adults. what self-management measure can the nurse teach the clients to prevent shock? A. do not get dehydrated in warm weather B. drink fluids on a regular schedule C. seek attention for any lacerations D. take meds as prescribed

B

a provider prescribes diazepam (valium) to a client who was bitten by a black widow spider. the client asks what is this medication for? how should the nurse respond? A. this medication is an antivenom for this type of bite B. it will relieve your muscle rigidity and spasms C. it prevents respiratory difficulty from excessive secretions D. this med will prevent respiratory failure

B

a pt gets a new prescription for pravigard for treatment of high cholesterol because this is a combo drug. the nurse alerts the physician when the pt discloses an allergy to which drug? A. sulfa B. aspirin C. some calcium channel blockers D. some diuretics

B

the nurse is caring for a client with a chest tube after a coronary artery bypass graft. the drainage slows significantly what action by the nurse is most important? A. increase the setting on the suction B. notify the provider immediately C. re-position the chest tube D. take the tubing apart to assess for clots

B

the nurse is caring for clients on the medical-surgical unit. What action by the nurse will help prevent a client from having a type 2 hypersensitivity reaction? A. administering steroids for severe serum sickness B. correctly identifying the client prior to a blood transfusion C. keeping the client free of the offending agent D. providing a latex-free environment for the client

B

the nurse is interviewing a pt with a history of high BP and heart problems. which statement by the pts causes the nurse to suspect the pt may have heart failure? A. i noticed a very fine red rash on my chest B. i had to take off my wedding ring last week C. ive had fever quite frequently D. i have pain in my shoulder when i cough

B

the nurse is reviewing the lab results for a pt who was admitted with dyspnea. which diagnostic test best differentiates between heart failure and lung dysfunction? A. arterial blood gas B. B-type natriuretic peptide C. hemoglobin D. serum electrolytes

B

the nurse is working in a women's health clinic is reviewing the risk factors for several pts for stroke and MI which pt has the hightes risk for MI? A. 49 year old on estrogen replacement therapy B. 55 year old with unstable angina C. 23 year old with diabetes that is currently not well controlled D. 60 year old with well controlled hypertension

B

the primary pacemaker of the heart, the sinoatrial (SA) node is functional if a pts pulse is at what refular rate? A. fewer than 60 bpm B. 60-100 bpm C. 80-100 bpm D. more than 100 bpm

B

the pt has smoked half a pack of cigarettes per day for 2 years. how many pack years as this pt smoked A. 1/2 pack/year B. 1 pack/year C. 1.5 pack/year D. 2 pack/year

B

under what circumstances should the nurse contact the pts health care proxy? A. a pt has a sudden and unexpected episode of dizziness B. pt is discovered at 4am in a comatose state C. pt refuses to eat unless he gets a beer with dinner D. pt needs catherization for a urine specimen

B

what does stimulation of the sympathetic nervous system produce A. delayed electrical impulse causing hypotension B. increased the heart rate C. virtually no effect on the ventricles of the heart D. slowed atrioventricular (AV) conduction time that results in a slow heart rate

B

what is the correct technique for assessing a pt with arterial insufficiency in the right lower leg? A. use the doppler to find the dorsalis pedis and posterior tibial pulses on the right leg B. palpate the peripheral arteries in a head to toe approach with a side to side comparison C. check all the pulse points in the right leg in dependent and supine positions D. palpate the major arteries, such as radial and femoral and observe for pallor

B

which description best defines the cardiovascular concept of afterload A. degree of myocardial fiber stretch at end of diastole and just before heart contracts B. amount of resistance the ventricles must overcome to eject blood through the semilunar valves and into the peripheral blood vessels C. pressure that the ventricle must overcome to open the tricuspid valve D. force of contraction independent of preload

B

which patient has a disorder that would be considered among the leading causes of death in the US? A. history of alchol abuse B. alzheimer's disease C. positive for HIV D. pancreatitis

B

which term describing the difference between systolic and diastolic values is an indirect measure of cardiac output? A. paradoxical BP B. pulse pressure C. ankle brachial index D. normal BP

B

A nurse prepares to admit a client who has herpes zoster. Which actions should the nurse take? select all that apply A. prepare a room for reverse isolation B. assess staff for a history of vaccination for chickenpox C. check the admission orders for analgesia D. choose a roommate who also is immune suppressed E. ensure that gloves are available in the room

B, C, E

A nurse works in a gerontology clinic. What age related changes cause the nurse to alter standard assessment techniques from those used for younger adults? Select all that apply. A. dentition deteriorates with more cavities B. nail beds may be thickened or discolored C. progressive loss of hair occurs with age D. sclerae begin to turn yellow or pale E. skin becomes dry as the client ages

B, C, E

A nursing student learns that many drugs can impair the immune system. Which drugs does this include? Select all that apply. A. acetaminophen (Tylenol) B. Amphotericin B (Fungizone) C. Ibuprofen (Motrin) D. Metformin (Glucophage) E. Nitrofurantonin (Macrobid)

B, C, E

a client is 1 day post op after a coronary artery bypass graft. what nonpharmacologic comfort measures does the nurse include when caring for this client? select all that apply A. administer pain medication before ambulating B. assist the client into a position of comfort in bed C. encourage high-protein diet selections D. provide complementary therapies such as music E. remind the client to splint the incision when coughing

B, D, E

A nurse assesses a client who presents with an increase in psoriatic lesions. Which questions should the nurse ask to identify a possible trigger for worsening of this clients psoriatic lesions? select all that apply A. have you eaten a large amount of chocolate lately? B. have you been under a lot of stress lately? C. have you recently used a public shower? D. have you been out of the country recently? E. have you recently had any other health problems F. have you changed any medications recently?

B, E, F

the nurse is conducting an initial cardiovascular assessment on a middle ages pt. what techniques does the nurse employ in the assessment? select all that apply. A. take blood pressure on the dominate arm B. palpate pulses at all major sites C. palpate for temperature differences in the lower extremites D. perform bilateral but separate palpation on the carotid arteries E. ausculate for bruites in the radial and brachial arteries F. check for orthostatic hypotension

B,C,D,F

the nurse is caring for a pt at risk for heart problems. what are normal findings for the cardiovascular assessment of this pt select all that apply A. presence of a thrill B. splitting of S2; decreases with expiration C. jugular venous distension to level of mandible D. point of maximal impulse (PMI) in fifth intercostal space at midclavicular line E. paradoxical chest movement with inspiration and expiration F. accentuated or intensified S1 after exercise

B,D,F

the nurse is performing BP screening at a community center. which pts are referred for evaluation of their BP? select all that apply A. diabetic pt with a BP of 118/78 B. pt with heart disease with a BP of 134/90 C. pt with no known health problems with a BP of 125/86 D. diabetic pt with BP of 180/80 E. pt with no known halth problems who has a BP of 106/70 F. pt with muscle cramping who is prescribed a statin drug

B,D,F

the terminally ill pt who is near death has loud wet reqpirations that are disturbing to the family. which interventions by the nurse are appropriate at this time? select all that apply A. ausculate lung sounds and obtain a chest x-ray B. place a small towel under the pts mouth C. use oropharyngeal suctioning to remove secretions D. administer an ordered anticholinergic drug to dry the secretions E. assist the pt to cough and deep breath to mobilize secretions F. reposition the pt onto one side to reduce gurgling

B,D,F

An older adult pt needs an oral potassium solution but is refusing it because it has a strong and unpleasant taste. What is the BEST strategy the nurse can use to administer the drug? A. Tell the pt that failure to take the drug could result in serious heart problems? B. Ask the pt's preference of juice and mix the drug with a small amount C. Mix the solution into food on the pt's meal tray and encourage the pt to eat everything D. Offer the drug to the pt several times and then document the pt's refusal

B. Ask the pt's preference of juice and mix the drug with a small amount

A patient has been admitted to the ED with diabetic ketoacidosis. On intake assessment, the pt cannot recall the medications she takes. What first action does the nurse take? A. Instruct the pt to compare a hospital list of medications to her home medications. B. Ask the pt's significant other to bring the pt's medications from home. C. Request that the patient complete a meal recall for the past 24 hours. D. Teach the patient about the importance of keeping a list of current medications in her purse.

B. Ask the pt's significant other to bring the pt's medications from home.

15. The nurse is admitting clients to the same-day surgery unit. Which insertion site for routine peripheral venous catheters does the nurse choose most often? A. Back of the hand for an older adult B. Cephalic vein of the forearm C. Lower arm on the side of a radical mastectomy D. Subclavian vein

B. Cephalic vein of the forearm The cephalic vein of the forearm is the insertion site chosen most often. For same-day surgery, the cephalic or basilic vein allows insertion of a larger IV catheter while allowing movement of the arm without impairing intravenous flow.Peripheral venous catheters should never be inserted into the back of the hand in an older adult because the veins are brittle. Peripheral venous catheters should never be inserted into the lower arm on the same side as a radical mastectomy because they interfere with limited circulation. Catheters are typically inserted into the subclavian vein by the health care provider, not by the nurse.

The nurse is caring for a psychiatric patient who is continuously drinking water. The nurse monitors for which complication related to potential hyponatremia? A. Proteinuria/prerenal failure B. Change in mental status/increased intracranial pressure C. Pitting edema/circulatory failure D. Possible occult blood/GI bleeding in stool

B. Change in mental status/increased intracranial pressure

18. A client who is receiving intravenous antibiotic treatments every 6 hours has an intermittent IV set that was opened and begun 20 hours ago. What action does the nurse take? A. Change the set immediately. B.Change the set in about 4 hours. C.Change the set in the next 12 to 24 hours. D.Nothing; the set is for long-term use.

B. Change the set in about 4 hours. Because both ends of the set are being manipulated with each dose, standards of practice dictate that the set should be changed every 24 hours, so the set should be changed in about 4 hours.It is not necessary to change out the set immediately, but it must be changed before the next 12 to 24 hours.

5. The nurse is administering a drug to a client through an implanted port. Before giving the medication, what does the nurse do to ensure safety? A.Administer 5 mL of a heparinized solution. B.Check for blood return. C.Flush the port with 10 mL of normal saline. D.Palpate the port for stability.

B. Check for blood return. To ensure safety, before a drug is given through an implanted port, the nurse must first check for blood return. If no blood return is observed, the drug should be held until patency is reestablished.If no blood return is observed, the drug should be held until patency is reestablished. Ports are flushed with heparin or saline after, rather than before, use. The port is palpated for stability, but this action alone does not ensure the client's safety.

The nurse is caring for a pt who takes potassium and digoxin. For what reason does the nurse monitor both laboratory results? A. Digoxin increases potassium loss through the kidneys B. Digoxin toxicity can result if hypokalemia is present C. Digoxin may cause potassium levels to rise to toxic levels D. Hypokalemia causes the cardiac muscle to be less sensitive to digoxin

B. Digoxin toxicity can result if hypokalemia is present

A pt with low potassium requires an IV potassium infusion. The pharmacy sends a 250-mL IV bag of dextrose in water with 40 mEq of potassium. The label is marked "to infuse over 1 hour". What is the nurses best action? A. obtain a pump and administer the solution B. Double check the providers order and call the pharmacy C. Hold the infusion because there is an error in labeling D. Recalculate the rate so that it is safe for the pt

B. Double check the providers order and call the pharmacy

20. The nurse is to administer a unit of whole blood to a postoperative client. What does the nurse do to ensure the safety of the blood transfusion? A. Asks the client to both say and spell his or her full name before starting the blood transfusion B. Ensures that another qualified health care professional checks the unit before administering Checks the blood identification numbers with the laboratory technician at the blood bank at the time it is dispensed Makes certain that an IV solution of 0.9% normal saline is infusing into the client before starting the unit

B. Ensures that another qualified health care professional checks the unit before administering To ensure safety, blood must be checked by two qualified health care professionals, usually two registered nurses.Administering an incorrectly matched unit of blood creates great consequences for the client and is considered to be a sentinel event. It requires a great amount of follow-up and often changing of policies to improve safety. The Joint Commission requires that the client provide two identifiers, but they are the name and date of birth or some other identifying data, depending on the facility; saying and spelling the name is only one identifier. Although a check is provided at the blood bank, this is not the one that is done before administration to the client. Clients do need to have normal saline running with blood, but this is not considered to be part of the safety check before administration of blood and blood products.

23. The nurse is revising an agency's recommended central line catheter-related bloodstream infection prevention (CR-BSI) bundle. Which actions decrease the client's risk for this complication? Select all that apply. A. During insertion, draping the area around the site with a sterile barrier B. Immediately removing the client's venous access device (VAD) when it is no longer needed C.Making certain that observers of the insertion are instructed to look away during the procedure D.Thorough hand hygiene (i.e., no quick scrub) before insertion E.Using chlorhexidine for skin disinfection

B. Immediately removing the client's venous access device (VAD) when it is no longer needed D. Thorough hand hygiene (i.e., no quick scrub) before insertion E. Using chlorhexidine for skin disinfection As soon as the VAD is deemed unnecessary, it needs to be removed to reduce the risk for infection. Thorough handwashing is a key factor in insertion and maintenance of a central line device. Quick handwashing is not sufficient. Chlorhexidine is recommended for skin disinfection because it has been shown to have the best outcomes in infection prevention.During the insertion, the whole body (head to toe) of the client is draped with a sterile barrier. Draping only the area around the site will increase risk for infection. Looking away will not reduce the risk for infection. Reducing the number of people in the room and having everyone wear a mask will help reduce the risk for infection.

The nurse is caring for a pt with hypovolemia secondary to severe diarrhea and vomiting. In evaluating the respiratory system for this pt, what does the nurse expect to find on assessment? A. No changes, because the respiratory system is not involved B. Increased respiratory rate, because the body perceives hypovolemia as hypoxia C. Hypoventilation, because the respiratory system is trying to compensate for low pH D. Normal respiratory rate, but a decreased oxygen saturation

B. Increased respiratory rate, because the body perceives hypovolemia as hypoxia

A patient is in the hospital for his first chemotherapy treatment for lung cancer. Which IV access methods are appropriate for this patient? A. Peripheral IV access B. Peripherally inserted central catheter (PICC) C. Dialysis catheter D. Tunneled central venous catheter E. Implanted port

B. Peripherally inserted central catheter (PICC) D. Tunneled central venous catheter E. Implanted port

A patient requires a non-tunneled percutaneous central catheter. What is the nurse's role in this procedure? A. Inset the catheter using sterile technique B. Place the patient in Trendelenburg position. C. Read the chest x-ray to validate placement. D. Select and prepare the insertion site.

B. Place the patient in Trendelenburg position.

In addition to magnesium levels, which other lab values should the nurse be sure to monitor when a patient has hypomagnesemia? A. Sodium and potassium B. Potassium and calcium C. Calcium and sodium D. Chloride and sodium

B. Potassium and calcium

Which pt showing symptoms of an acid-base imbalance must the nurse see first? A. Pt who exhibits increased heart rate of 110/min and increased cardiac output B. Pt showing activity weakness and lethargy C. Pt who has a reduced attention span D. Pt who has asymptomatic bradycardia with a heart rate of 57/min

B. Pt showing activity weakness and lethargy

Which assessment about compensation for acid-base imbalance is accurate? A. The respiratory system is less sensitive to acid-base changes B. The respiratory system can begin compensation within seconds to minutes C. The renal system is less powerful than the respiratory system. D. The renal system is more sensitive to acid-base changes.

B. The respiratory system can begin compensation within seconds to minutes

The nurse is caring for a pt with severe hypocalcemia. What safety measure does the nurse put in place for this patient. Select all that apply: A. Encourage the pt to use a cane when ambulation B. Turn on a bed alarm when the pt is in bed C. Obtain an order for zolpidem (Ambien) to ensure the pt sleeps at night D. Place the pt on a low bed E. Ensure the top side rails are up when the patient is in bed F. Raise all four side rails.

B. Turn on a bed alarm when the pt is in bed D. Place the pt on a low bed E. Ensure the top side rails are up when the patient is in bed

A nurse assesses clients on a medical-surgical unit. Which client is at greatest risk for pressure ulcer development? A. a 44 year old prescribed IV antibiotics for pneumonia B. A 26 year old who is bedridden with a fractured leg C. a 65 year old with hemi-paralysis and incontinence D. a 78 year old requiring assistance to ambulate with a walker

C

A nurse cares for a client who has facial burns. The client asks, will I ever look the same? How should the nurse respond? A. with reconstructive surgery, you can look the same B. we can remove the scars with the use of a pressure dressing C. you will not look exactly the same but cosmetic surgery will help D. you shouldnt start worrying about your appearance right now

C

A nurse cares for a client who is prescribed vancomycin (Vancocin) 500 mg IV every 6 hours for a methicillin-resistant Staphylococcus aureus (MRSA) infection. Which action should the nurse take? A. administer it over 30 minutes using an IV pump B. Give the client diphenhydramine (Benadryl) before the drug C. assess the IV site at least every 2 hours for thrombophlebitis D. ensure that the client has increased oral intake during therapy

C

A nurse cares for a client with a burn injury who presents with drooling and difficulty swallowing. Which action should the nurse take first? A. assess the level of consciousness and pupillary reactions B. Ascertain the time food or liquid was last consumed C. auscultate breath sounds over the trachea and bronchi D. measure abdominal girth and auscultate bowel sounds

C

A nurse cares for an older adult client who has a chronic skin disorder. The client states, I have not been to church in several weeks because of the discoloration of my skin. How should the nurse respond? A. I will consult the chaplain to provide you with spiritual support B. you do not need to go to church; god is everywhere C. tell me more about your concerns related to your skin D. religious people are nonjudgmental and will accept you

C

A nurse is caring for a client who is about to receive a bone marrow transplant. To best help the client cope with the long recovery period, what action by the nurse is best? A. arrange a visitation schedule among friends and family B. explain that this process is difficult but must be endured C. help the client find things to hope for each day of recovery D. provide plenty of diversionary activities for this time

C

A nurse is caring for a young male client with lymphoma who is to begin treatment. What teaching topic is a priority? A. genetic testing B. infection prevention C. sperm banking D. treatment options

C

A nurse is caring for four clients. After reviewing todays lab results, which client should the nurse see first? A. client with an international normalized ratio of 2.8 B. client with a platelet count of 128,000/mm3 C. client with a prothrombin time (PT) of 28 seconds D. client with a red blood cell count of 5.1 million/L

C

A nurse prepares to administer intravenous cimetidine (Tagamet) to a client who has a new burn injury. The client asks, why am I taking this medication? How should the nurse respond? A. tagamet stimulates intestinal movement so you can eat more B. it improves fluid retention, which helps prevent hypovolemic shock C. it helps prevent stomach ulcers, which are common after burns D. Tagamet protects the kidney from damage caused by dehydration

C

A nurse reviews the following data in the chart of a client with burn injuries: Admission notes: 36 year old female with bilateral leg burns, NKDA, health history of asthma and seasonal allergies wound assessment: bilateral leg burns present with a white and leather like appearance. no blisters or bleeding present. client rates pain 2/10. Based on the data provided, how should the nurse categorize this clients injuries? A. partial-thickness deep B. partial-thickness superficial C. full thickness D. superficial

C

A nurse reviews the lab results for a client who was burned 24 hours ago. Which lab result should the nurse report to the health care provider immediately? A. arterial pH: 7.32 B. Hematocrit: 52% C. serum potassium: 6.5mEq/L D. serum sodium: 131 mEq/L

C

A nurse teaches a client being treated for a full-thickness burn. Which statement should the nurse include in this clients discharge teaching? A. you should change the batteries in your smoke detector once a year B. join a program that assists burn clients to reintegration into the community C. I will demonstrate how to change your wound dressing for you and your family D. let me tell you about the many options available to you for reconstructive surgery

C

A nurse uses the rule of nines to assess a client with burn injuries to the entire back region and left arm. How should the nurse document the percentage of the clients body that sustained burns? A. 9% B. 18% C. 27% D. 36%

C

a client in shock has been started on dopamine. what assessment finding requires the nurse to communicate with the provider immediately? A. blood pressure of 98/68 mm Hg B. pedal pulses 1+/4+ bilaterally C. report of chest heaviness D. urine output of 32 mL/hr

C

a client in the cardiac stepdown unit reports severe crushing chest pain accompanied by nausea and vomiting what action by the nurse takes priority? A. administer an asprin B. call for an electrocardiogram (ECG) C. maintain airway patency D. notify the provider

C

a client is on a ventilator and is sedated. what care may the nurse delegate to the unlicensed assistive personnel (UAP)? A. assess the client for sedation needs B. get family permission for residents C. provide frequent oral care per protocol D. use nonverbal pain assessment tools

C

a client is receiving an infusion of tissue plasminogen activator (t-PA). the nurse assesses the client to be disoriented to person, place, and time. what action by the nurse is best? A. assess the clients pupillary responses B. request a neurologic consolation C. stop the infusion and call the provider D. take and document a full set of vital signs

C

a client presents to the ED with an acute MI at 1500 (3pm) the facility has 24 hour catheterization laboratory abilities to meet the joint commissions core measures set, by what time should the client have a percutaneous coronary intervention performed? A. 1530 (3:30 pm) B. 1600 (4:00 pm) C. 1630 (6:30 pm) D. 1700 (5:00 pm)

C

a nurse assesses a client recently bitten by a coral snake. which assessment should the nurse complete first? A. unilateral peripheral swelling B. clotting times C. cardiopulmonary status D. electrocardiogram rhythm

C

a nurse cares for clients during community wide disaster drill. once of the clients asks, why are the indiciduals with black tags not receiving any care? how should the nurse respond? A. to do the greatest good for the greatest number of people it is necessary to sacrifice some B. not everyone will survive a disaster, so it is best to identify those people early and move on C. in a disaster, extensive resources are not used for one person at the expense of many others D. with black tags, volunteers can identify those who are dying and can give them comfort care

C

a nurse caring for a client notes the following assessments: white blood cell count 3800/mm3, blood glucose level 198 mg/dL, and temp 96.2 F. what action by the nurse takes priority? A. document the findings in the clients chart B. give the client warmed blankets for comfort C. notify the health care provider immediately D. prepare to administer insulin per sliding scale

C

a nurse is assisting the health care provider who is intubating a client. the provider has been attempting to intubate for 40 seconds. What action by the nurse takes priority? A. ensure the client has adequate sedation B. find another provider to intubate C. interrupt the procedure to give O2 D. monitor the clients O2 saturation

C

a nurse is caring for a client whos wife died in a recent mass casualty accident. the client says, I cant believe that my wife is gone and I am left to raise my children all by myself. how should the nurse respond? A. please accept my sympathies for your loss B. i can call the hospital chaplain if you wish C. you sound anxious about being a single parent D. at least your children still have you in their lives

C

a nurse is triaging clients in the ED which client should be considered urgent? A. 20 year old female with chest stab wound and tachycardia B. 45 year old homeless man with skin rash and sore throat C. 75 year old with cough and temp of 102 F D. 50 year old with new onset confusion and slurred speech

C

a nurse is triaging clients in the emergency department. which client should the nurse classify as nonurgent? A. 44 year old with chest pain and diaphoresis B. 50 year old with chest trauma and absent breath sounds C. 62 year old with simple fracture of the left arm D. 79 year old with temp of 104F

C

a provider prescribes crotalidae polyvalent immune fab (CroFab) for a client who is admitted after being bitten by a pit viper snake. which assessment should the nurse complete prior to administering this medication? A. assess temperature and for signs of fever B. check the clients creatinine kinase level C. ask about allergies to pineapple or papaya D. inspect the skin for signs of urticaria (hives)

C

a pt comes to the clinic stating, my right foot turns a darkish red color when i sit too long, and when i put my foot up it turns pale. which condition does the nurse suspect? A. central cyanosis B. peripheral cyanosis C. arterial insufficiency D. venous insufficiency

C

a pt in the telemetry unit who has continuous ECG monitoring is scheduled for a test in the radiology department who is responsible for determining when monitoring can be suspended? A. telemetry technician B. charge nurse C. health care provider D. primary nurse

C

a pts bilateral radial pulses are occasionally weak and irregular. which assessment techniques does the nurse use first to investigate this finding? A. check the color and capillary refill in the upper extremities B. check the peripheral pulses in the lower extremities C. take the apical pulse for 1 min, noting any irregularity in heart rhythm D. check the cardiac monitor for irregularities in rhythm

C

a student nurse is caring for a client who will be receiving sodium nitroprusside (Nipride) via IV infusion what action by the student causes the registered nurse to intervene? A. assessing the IV site before giving the drug B. obtaining a programmable (smart) IV pump C. removing the IV bag from the brown plastic cover D. taking and recording a baseline set of vital signs

C

after teaching a client how to prevent altitude related illnesses, a nurse assesses the client understanding. which statement indicates the client needs additional teaching? A. if my climbing partner cant think straight we should descend to a lower altitude B. i will ask my provider about medication to help prevent acute mountain sickness C. my partner and I will plan to sleep at a higher elevation to acclimate more quickly D. i will drink plenty of fluids to stay hydrated while on the mountain

C

after teaching a client who expressed concern about a rash located beneath her breast, a nurse assesses the clients understanding. Which statement indicates the client has a good understanding of this condition? A. this rash is probably due to fluid overload B. I need to wash this daily with antibacterial soap C. I can use powder to keep this area dry D. I will schedule a mammogram as soon as I can

C

an ED case manager is consulted for a client who is homeless which intervention should the case manager provide? A. communicate client needs and restrictions to support staff B. prescribe low cost antibiotics to treat community-acquired infection C. provide referrals to subsidized community based health clinics D. offer counseling for substance abuse and mental health disorders

C

an Ed charge nurse notes an increase in sick calls and bickering among the staff after a week with multiple trauma incidents. which action should the nurse take? A. organize a pizza party for each shift B. remind the staff of the facility's sick-leave policy C. arrange for critical incident stress debriefing D. talk individually with staff members

C

an emergency room nurse is triaging victims of a multi-casualty event. which client should receive care first? A. 30 year old distraught mother holding her crying child B. 65 year old conscious male with head lac C. 26 year old male who has pale, cool, clammy skin D. a 48 year old with simple fracture of lower leg

C

based on the physiologic force that propels blood forward in the veins, which pt has the greatest risk for venous stasis? A. older adult pt with hypertension who rides a bike daily B. middle aged construction worker taking warfarin C. bedridden pt in the end stage of alzheimer's disease D. teenage pt with a broken leg who sits and plays video games

C

because cardiac dysrhythmias are abnormal rhythms of the heart's electrical system, the heart is unable to perform what function? A. it cannot oxygenate the blood throughout the body B. it cannot remove carbon dioxide from the body C. it cannot effectively effectively pump oxygenated blood throughout the body D. it cannot effectively conduct impulses with increased activity

C

in assessing a pt who has come to the clinic for a physical exam, the nurse notes that the pt has decreased skin temp what is this finding most indicative of A. anemia B. heart failure C. arterial insufficiency D. stroke

C

the night shift nurse is listening to report and hears that a pt has paroxysmal nocturnal dyspena. what does the nurse plan to do next? A. instruct the pt to sleep in a side lying position and then check on the pt every 2 hours to help with switching sides B. make the pt comfortable in a bedside recliner with several pillows to keep the pt more upright throughout the night C. check on the pt several hours after bedtime and assist the pt to sit upright and dangle the feet when dyspnea occurs D. check the pt frequently because the pt has insomnia due to a fear of suffociation

C

the nurse is performing an assessment on a cardiac pt. in order to determine if the pt has a pulse deficit, what does the nurse do? A. take the pts BP and subtract the diastolic from the systolic pressure B. take the pts pulse in a supine position and then in a standing position C. assess the apical and radial pulses for a full minute and calculate differences in rate D. take the radial pulse, have the pt rest for 15 mins and then retake the pulse

C

the nurse is taking a history on a pt recently diagnosed with heart failure. the pt admits to "sometimes having trouble catching my breath" but is unable to provide more specific details. what question does the nurse ask to gather more data about the pts symptoms? A. do you have any medical problems, such as high blood pressure? B. what did your dr tell you about your diagnosis? C. what was your most strenuous activity in the past week? D. how do you feel about being told that you have heart failure?

C

the nurse is taking vitals and reviewing the ECG of a pt who is training for a marathon. the heart rate is 45 bpm and the ECG shows sinus bradycardia. how does the nurse interpret this data? A. a rapid filling rate that lengthens diastolic filling time and leads to decreased cardiac output B. the bodys attempt to compensate for a decreased stroke volume by decreasing the heart rate C. an adequate stroke volume that is associated with cardiac conditioning D. a common finding in the health adult that would be considered normal

C

the nurse reads in a pts chart that a carotid bruit was heard during the last 2 annual checkups. today on ausculation the bruit is absent. how does the nurse evaluate this data? A. the problem has resolved spontaneously B. there may have been an anomaly in previous findings C. the occlusion of the vessel may have professed past 90% D. the antiplatelet therapy is working

C

the terminally ill patient has an advance directive living will, which indicates that no heroic measures such as CPR and intubation should be performed. She also has a DNR order in her chart written by the health care provider. as the patient nears death, her daughter tells the nurse that she wants everything possible done to save her mothers life. What is the nurses best action? A. call a code and bring the crash cart to the patient's bedside B. inform the health care provider of this change in the plan of care C. respect the patient's wishes and ask the chaplain to stay with the daughter D. inform the daughter that further interventions are not warranted

C

to qualify for medicare hospice benifits, a crierion for admission is that the pts prognosis must be limited to what amount of time? A. 2 weeks or less B. 3 months or less C. 6 months or less D. 1 year or less

C

when the nurse assess the dying pt inadequate perfusion is suspected because the pts lower extremities are cold, mottled, and cyanotic. which intervention should the nurse perform? A. place the lower extremities in a dependent position B. give warm oral or IV fluids C. cover the pt with a warm blanket D. gently rub the extremities to stimulate circulation

C

which category of cardiovascular drugs increases heart rate and contractility A. diuretics B. beta blockers C. catecholamines D. benzodiazepines

C

which definition best describes left sided heart failure? A. increased volume and pressure develop and result in peripheral edema B. it can occur when cardiac output remains normal or above normal C. there is decreased tissue perfusion from poor cardiac output and pulmonary congestion from increased pressure in the pulmonary vessels D. it is the percentage of blood ejected from the heart during systole

C

which pt and family have the best understanding of home hospice? A. family believes that the dying pt receives care at home if there are no funds for hospitalization B. family expects that the pt will resist hospice, therefore, an involuntary order is requested C. the dying pt and family want to focus on facilizing quality of life the pt and family expect an RN to provide around the clock nursing care

C

which pt statement best represents the symptom that is most distressing and feared by terminally ill pts A. i get really nervous when i cant catch my breath B. my family will be so upset if i can't recognize them C. i'm hoping my dr prescribes alot of pain meds D. when i get nauseated, i wont be able to eat or drink

C

while caring for a pt of the orthodox jewish faith who is dying, what cultural concept should the nurse keep in mind A. traditionally, jewish cultures are male dominated B. expression of greif is open, especially among women C. a person who is extremely ill and dying should not be left alone D. family members are likely to avoid visiting the terminally ill family member

C

while triaging clients in a crowded emergency department a nurse assesses a client who presents with symptoms of TB which action should the nurse take first? A. apply o2 via nasal cannula B. administer IV 0.9% saline solution C. transfer the client to a negative pressure room D. obtain a sputum culture and sensitivity

C

19. A client who used to work as a nurse asks, "Why is the hospital using a 'fancy new IV' without a needle? That seems expensive." How does the nurse respond? A. "OSHA, a government agency, requires us to use this new type of IV." B. "These systems are designed to save time, not money." C."They minimize health care workers' exposure to contaminated needles." D. "They minimize clients' exposure to contaminated needles."

C. "They minimize health care workers' exposure to contaminated needles." The nurse informs the client that needleless IVs were designed to protect health care personnel from exposure to contaminated needles.The Occupational Safety and Health Administration (OSHA) requires the use of devices with engineered safety mechanisms only. It does not mandate that they be needleless. Saving time and money is not the purpose of the needleless IV, and it was not designed to protect clients from exposure to contaminated needles.

12. A 22-year-old client is seen in the emergency department (ED) with acute right lower quadrant abdominal pain, nausea, and rebound tenderness. It appears that surgery is imminent. What gauge catheter does the ED nurse choose when starting this client's intravenous solution? A.24 B.22 C.18 D.14

C. An 18-gauge catheter is the size of choice for clients who will undergo surgery. If they need to receive fluids rapidly, or if they need to receive more viscous fluids (such as blood or blood products), a lumen of this size would accommodate those needs.Neither a 24-gauge nor a 22-gauge catheter is an appropriate size (too small) for clients who will undergo surgery. If it becomes necessary to administer fluids to the client rapidly, another IV would be needed with a larger needle—18, for example. Administering through the smallest gauge necessary is usually best practice, unless the client may be going into hypovolemic status (shock). A 14-gauge catheter is an extremely large-gauge needle that is very damaging to the vein.

The nurse must insert a short peripheral IV catheter. In order to decrease the risk of deep vein thrombosis or phlebitis, which vein does the nurse choose for the infusion site? A. Wrist B. Foot C. Forearm D. Antecubital

C. Forearm

The pt with hyperkalemia is prescribed patiromer. Which statement most accurately describes the function of this drug? A. It works in the kidneys to increase excretion of potassium. B. The drug prevents the kidneys from absorbing potassium. C. It binds with potassium in the GI tract and decreases its absorption. D. The drug increases motility in the GI tract, eliminating potassium in diarrhea stools.

C. It binds with potassium in the GI tract and decreases its absorption.

A patient requires an infusion of packed red blood cells (PRBCs) . Which factor allows the nurse to infuse the PRBCs through the patient's PICC? A. Length of the PICC allows infusion within 6 hours B. The nurse is unable to obtain an infusion pump C. Lumen size of the PICC is 4 Fr or larger. D. PRBCs can be warmed before infusion.

C. Lumen size of the PICC is 4 Fr or larger.

A pt has taken antacids for the past 3 days to relieve "heartburn". What alteration in acid-base balance would the nurse expect for this pt? A. Respiratory alkalosis B. Metabolic acidosis C. Metabolic alkalosis D. Respiratory acidosis

C. Metabolic alkalosis

25. The nurse is inserting a peripheral intravenous (IV) catheter. Which client statement is of greatest concern during this procedure? A."I hate having IVs started." B."It hurts when you are inserting the line." C."My hand tingles when you poke me." D."My IV lines never last very long."

C. My hand tingles when you poke me." The client's statement about a tingling feeling indicates possible nerve puncture and is of greatest concern to the nurse. To avoid further nerve damage, the nurse should stop immediately, remove the IV catheter, and choose a new site.Statements such as, "I hate having IVs started," "It hurts when you are inserting the line," and "My IVs never last very long," are addressed with teaching about the importance of proper protection of the site.

4. When flushing a client's central line with normal saline, the nurse feels resistance. Which action does the nurse take first? A.Decrease the pressure being used to flush the line. B.Obtain a 10-mL syringe and reattempt flushing the line. C.Stop flushing and try to aspirate blood from the line. D.Use "push-pull" pressure applied to the syringe while flushing the line.

C. Stop flushing and try to aspirate blood from the line. The nurse's first step is to stop flushing and try to aspirate blood from the line. If resistance is felt when flushing any IV line, the nurse should stop and further assess the line. Aspiration of blood would indicate that the central line is intact and is not obstructed by thrombus.Decreasing the pressure to flush the line is not appropriate. Continuing or reattempting to flush the line, or using a push-pull action on the syringe, might result in thrombus or injection of particulate matter into the client's circulation.

The nurse is assessing the pt with a risk for hypocalcemia. What is the correct technique to test for Chvostek's sign? A. Pt reflexes arms against the chest and examiner attempts to pull the arms away from the chest B. Place a blood pressure cuff around the upper arm and inflate the cuff to greater than the pt's systolic pressure C. Tap the pt's face just below and in front of the ear to trigger facial twitching of one side of the mouth, nose, and check. D. Lightly tap the pt's patellar and achilles tendons with a reflex hammer and measure the movement

C. Tap the pt's face just below and in front of the ear to trigger facial twitching of one side of the mouth, nose, and check.

Oncogenes that are over expressed can cause what?

Can cause a cell to develop into a tumor

A client is receiving plasmapheresis as treatment for goodpastures syndrome. When planning care, the nurse places highest priority on interventions for which client problem? A. reduced physical activity related to the diseases effects on the lungs B. inadequate family coping related to the clients hospitalization C. inadequate knowledge related to the plasmapheresis process D. potential for infection related to the site for organism invasion

D

A nurse assesses bilateral wheezes in a client with burn injuries inside the mouth. Four hours later the wheezing is no longer heard. Which action should the nurse take? A. document the findings and reassess in 1 hour B. loosen any constrictive dressings on the chest C. raise the head of the bed to a semi-fowlers position D. gather appropriate equipment and prepare for an emergency airway

D

A nurse cares for a client who has burn injuries. The clients wife asks, when will his high risk for infection decrease? How should the nurse respond? A. when the antibiotic therapy is complete B. as soon as his albumin levels return to normal C. once we complete the fluid resuscitation process D. when all of his burn wounds have closed

D

A nurse cares for clients who have various skin infections. Which infection is paired with the correct pharmacologic treatment? A. viral infection clindamycin (cleocin) B. bacterial infection acyclovir (Zyvox) C. yeast infection Linezolid (Zyvox) D. fungal infection ketoconazole (Nizoral)

D

A nurse is caring for a client who has a pressure ulcer on the right ankle. Which action should the nurse take first? A. draw blood for albumin, prealbumin, and total protein B. prepare for and assist with obtaining a wound culture C. place the client in bed and instruct the client to elevate the foot D. assess the right leg for pulses, skin color, and temperature

D

A nurse is preparing to hang a blood transfusion. Which action is most important? A. documenting the transfusion B. placing the client on NPO status C. placing the client in isolation D. putting on a pair of gloves

D

A nurse teaches a client who has very dry skin. Which statement should the nurse include in this clients education? A. use lots of moisturizer several times a day to minimize dryness B. take a cold shower instead of soaking in the bathtub C. use antimicrobial soap to avoid infection of cracked skin D. after you bathe, put lotion on before your skin is totally dry

D

A nursing student is caring for a client with leukemia. The student asks why the client is still at risk for infection when the clients white blood cell count is high. What responce by the registered nurse is best? A. if the WBCs are high, there already is an infection present B. the client is in a blast crisis and has too many WBCs C. there must be a mistake; the WBCs should be very low D. those WBCs are abnormal and don't provide protection

D

What is the total time required for ventricular depolarization and repolarization as represented on the ECG? A. PR interval B. QRS complex C. ST segment D. QT interval

D

a client who is hospitalized with burns after losing the family home in fire becomes angry and screams at a nurse when dinner is served late. how should the nurse respond? A. do you need something for pain right now? please stop yelling, i brought dinner as soon as i could C. i suggest that you get control of yourself D. you seem upset, i have time to talk if youd like

D

a nurse assess a client admitted with a brown recluse spider bite. which priority assessment should the nurse perform to identify complications of this bite? A. ask the client about purritus at the bite site B. inspect the bite site for a bluish purple vesicle C. assess the extremity for redness and swelling D. monitor the clients temp every 4 hours

D

a nurse assesses a client who has multiple areas of ecchymosis on both arms. Which question should the nurse ask first? A. are you using lotion on your skin? B. do you have a family history of this? C. do your arms itch? D. what medications are you taking?

D

a nurse is assessing a client who had a MI upon ausculating heart sounds, the nurse hears the following sound. what actions by the nurse is most appropriate? (click the media button to hear the audio clip) A. assess for further chest pain B. call the rapid response team C. have the client sit upright D. listen to the clients lung sounds

D

a nurse is caring for a client on mechanical ventilation. when double checking the ventilator settings with the respiratory therapist, what should the nurse ensure as a priority? A. the client is able to initiate spontaneous breaths B. the inspired oxygen has adequate humidification C. the upper peak airway pressure limit alarm is off D. the upper peak airway pressure limit alarm is on

D

a nurse is field triaging clients after an industrial accident. which client condition should the nurse triage with a red tag? A. dislocated right hip and open fracture of the right lower leg B. large contusion to the forehead and a bloody nose C. closed fracture of the right clavicle and arm numbness D. multiple fractured ribs and SOB

D

a pt is admitted for heart failure and has edema, neck vein distension, and ascites. what is the most reliable way to monitor fluid gain or loss in this pt? A. check for pitting edema in the dependent body parts B. auscultate the lungs for crackles or wheezing C. assess skin turgor and the condition of mucous membranes D. weigh the patient daily at the same time with the same scale

D

the dying pt reports shortness of breath and has an o2 saturation of 90% he refuses o2 therapy but requests that the nurse obtain a fan to increase the circulation of air. based on the concept of comfort, what should the nurse do first A. explain that the use of a fan will not increase the o2 level B. try a nonpharmcologic intervention such as position change or distraction C. call the health care provider and report the refusal of o2 D. offer morphine and advise the pt that a fan will be provided

D

the nurse is assessing a pt with suspected CVD when assessing the precordium which assessment technique does the nurse begin with? A. percussion B. palpation C. auscultation D. inspection

D

the nurse is caring for several patients in the telemetry unit who are being remotely watched by a monitor technician. what is the nurses primary responsibility in the monitoring process of these patients? A. watching the bank of monitors on the unit B. printing ECG rhythm strips routinely and as needed C. interpreting rhythms D. assessment and management of patients

D

the nurse is conducting dietary teaching with a pt. which statement by the pt indicates an understanding of fat sources and the need to limit saturated fats? A. coconut oil has a rich flavor and is a good cooking oil B. sunflower oil is high in saturated fats so I should avoid it C. meat and eggs mostly contain unsaturated fats D. canola oil has monounsaturated fat and is recommended

D

the nurse is notified by the telemetry monitor technician about a pts heart rate. which method does the nurse use to confirm the technicians report? A. count QRS complexes in a 6 second strip and multiply by 10 B. analyze an ECG rhythm strip by using an ECG caliper C. run an ECG rhythm strip and use the memory method D. assess the pts heart rate directly by taking an apical pulse

D

the nurse is performing a dietary assessment on a 45 ear old business executive at risk for CVD. which assessment method used by the nurse is the most reliable and accurate A. ask the pt to identify foods he or she eats that contain sodium, sugar, cholesterol, fiber and fat B. ask the pts spouse, who does the cooking and shopping to identify the types of foods that are consumed C. ask the pts how cultural beliefs and economic status influence the choice of food items D. ask the pt to recall the intake of food, fluids and alcohol during a typical 24 hour period

D

which action is an example of active euthanasia for a dying patient? A. discontinuing the mechanical ventilator B. terminating the IV fluids C. suspending telemetry heart monitoring D. giving a large dose of IV morphine

D

which exercise regimen for an older adult meets the recommended guidelines for physical fitness to promote heart health A. 6 hour bike ride every saturday B. golfing for 4 hours two times/week C. running for 15 mins 3x per week brisk walk 30 mins/day

D

while listening to a pts heart sounds, the nurse detects a murmur. what does the nurse understand about the cause of murmurs? A. a murmur is caused by the closing of the aortic and pulmonic valves B. a murmur is caused when the blood flows from the atrium to a noncompliant ventricle C. a murmur is caused by anemia, hypertension, or ventricular hypertrophy D. a murmur is caused when there is turbulent blood flow through normal or abnormal valves

D

A nurse assesses an older adults skin. Which findings require immediate referral? select all that apply A. excessive moisture under axilla B. increased hair thinning C. increased presence of fungal toenails D. lesion with various colors E. spider veins on legs F. asymmetric 6-mm dark lesion on forehead

D, F

an ED charge nurse prepares to receive clients from a mass casualty within the community. what is the role of this nurse during the event? A. ask ED staff to discharge clients from the med surg units in order to make room for critically injured victims B. call additional med surg and critical care nursing staff to come to the hospital and assist when victims are brought in C. inform the incident commander at the mass casualty scene about how many victims may be handled by the ED D. direct med surg and critical care nurses to assist with clients currently in the ED while emergency staff prepare to receive the mass casualty victims

D.

Which serum laboratory value does the nurse expect to see in the pt with hypokalmia? A. Calcium less than 8.0 mg/dL B. Potassium less than 5.0 mEq/L C. Calcium less than 11.0 mg/dL D. Potassium less than 3.5 mEq/L

D. Potassium less than 3.5 mEq/L

Which intervention does the nurse implement for a pt with hypocalcemia? A. Encourage activity by the pt as tolerated, including weight-lifting B. Encourage socialization and active participation in stimulating activities C. Keep a tracheostomy tray at the bedside for emergency use D. Provide adequate intake of vitamin D and calcium-rich foods

D. Provide adequate intake of vitamin D and calcium-rich foods

Which pt is most likely to develop metabolic alkalosis as a result of base excess? A. Pt taking thiazide diuretics B. Pt who is having NG suction C. Pt with severe vomiting D. Pt who had massive blood transfusion

D. Pt who had massive blood transfusion

A pt being treated for pneumonia reports pain that increases on inspiration. The nurses suspects which complication has occurred? a. pleuritic chest pain b. pulmonary emboli c. pleural effusion d. meningitis

a

A pt has been diagnosed with airway obstruction during sleep. The nurse will likely include pt education about which device for home use? a. continuous positive airway pressure (CPAP) to deliver a positive airway pressure b. O2 via face mask to prevent hypoxia c. neck brace to support the head to facilitate breathing d. nebulizer treatments with bronchodilators

a

A pt has been diagnosed with sleep apnea. Which assessment findings indicate that the pt is having complications associated with sleep apnea? a. side effects of hypoxemia, hypercapnia and sleep deprivation b. decrease in arterial CO2 levels and sleep deprivation c. respiratory alkalosis with retention of CO2 d. irritability, obesity, and enlarged tonsils/adenoids

a

A pt has had neck dissection surgery with a reconstructive flap over the carotid artery. Which intervention is appropriate for the flap care? a. evaluate the flap every hour for the first 72 hours b. monitor the flap by gently placing a doppler on the flap c. position the pt so that the flap is in the dependent position d. apply a wet-to-dry dressing to the flap

a

A pt is admitted to the hospital with cough, purulent sputum production, temperature of 100.3F and reports SOB. Which intervention does the nurse provide first? a. set up O2 equipment and administer O2 b. instruct the pt about the importance of keeping the O2 delivery device on. c. monitor the effectiveness of O2 therapy (pulse ox, ABGs) as appropriate d. monitor the pt's anxiety related to the need for O2 delivery

a

A pt is admitted to the hospital with pneumonia. What does the nurse expect the chest x-ray to reveal? a. patchy areas of increased density b. tension pneumothorax c. thick secretions causing airway obstruction d. large hyper-inflated airways

a

A pt with a history of frequent and recurrent episodes of tonsillitis now reports a severe sore throat with pain that radiates behind the ear and difficulty swallowing. The nurse suspects the pt may have peritonsillar abscess. On physical assessment, which deviated structure supports the nurse's supposition? a. uvula b. trachea c. tongue d. mucous membranes

a

A pt with an active nosebleed (epistaxis) is admitted to the ED. Which intervention does the nurse use FIRST? a. have the pt sit upright with the head forward b. insert nasal packing c. apply direct lateral pressure to the nose d. place a nasal catheter

a

After a bronchoscopy procedure, the pt coughs up sputum which contains blood. What is the best nursing action at this time? a. Assess vital signs and respiratory status and notify the provider of the findings b. Monitor the pt for 42 hrs to see if blood continues in the sputum c. Send the sputum othe lab for cytology for possible lung cancer d. Reassure the pt that this is a normal response after a bronchoscopy

a

After being treated in the ED for posterior nosebleed, the pt is admitted to the hospital. The nasal packing is in place and vital signs are stable. The pt has an IV of NS at 125 ml/hr. What is the PRIORITY for nursing care? a. airway management b. managing potential dehydration c. managing potential decreased cardiac output d. monitoring for potential infection

a

The nurse is caring for a pt who had a nasoseptoplasty. Which action is the best to assign to the LPN? a. administer a stool softener to ease bowel movements b. assess the pt's airway and breathing after general anesthesia c. evaluate the pr's emotional rxn to the facial edema and bruising d. take vital signs every 4 hrs as ordered by the physician

a

The nurse is conducting an in-service for the hospital staff about practices that help prevent pneumonia among at-risk pts. Which nursing intervention is encouraged as standard practice? a. administering vaccines to pts at risk b. implementing isolation for debilitated pts c. restricting foods from home in immune suppressed pts d. decontaminating respiratory therapy equipment weekly

a

The nurse is providing post-op nursing care for a pt with surgical correction of a deviated septum. Which intervention is part of the standard care for this pt? a. apply ice to the nasal area and eyes to decrease swelling and pain b. encourage deep coughing to prevent atelectasis and clear secretions c. administer NSAIDs/Tylenol q4-6 hrs for pain d. apply moist heat and humidity to the nasal area for comfort and circulation

a

The nurse is providing pre-operative teaching for a pt who will have a malignant sinus tumor surgically removed. Which KEY teaching point would the nurse be sure to include? a. pts often have changes in the sensations of taste and smell b. problems of speech rarely occur with this type of surgery c. often pts gain weight and need dietary consults for weight loss d. after the surgery, you will need to have irrigations with an alcohol based solution

a

The nurse is reviewing the arterial blood gas results ofr a 25 y/o trauma pt who has new onset of shortness of breath and demonstrates shallow and irregular respirations. The arterial blood gas restuls are: pH, 7.26; PCO2, 47%; PO2, 89%, HCO3, 24. What imbalance does the nurse suspect this pt has? a. Respiratory acidosis b. Resiratory alkalosis c. Metabolic acidosis d. Metabolic alkalosis

a

The nurse is taking a history on a pt who reports sleeping in a reclinder chair at night because lying on the bed causes shortness of breath. How is this documented? a. Orthopnea b. Paroxysmal nocturnal dyspnea c. Orthostatic nocturnal dyspnea d. Tachypnea

a

The nurse reviews the complete blood count results for the patient who has chronic obstructive pulmonary disease (COPD) and lives in a high mountain area. What lab results does th enurse expect to see for this patient? a. Increased red blood cells b. Decreased neutrophils c. Decreased eosinophils d. Increased lymphocytes

a

The nurse teaches a pt about the imapct of cigarette smoking on the lower respiratory tract. Which statement by the pt indicates an understanding of the information? a. Using nicotine replacement therapy will increase my chances of success b. If I stop smoking the damage to my lungs will be reversed c. Cigarette smoke affects my ability to cough out secreations from the lungs d. Smoking makes the large and small airways get bigger

a

What does the nurse include in the teaching session for a pt who is scheduled to have a partial laryngectomy? a. supraglottic mehod of swallowing b. presence of a trach tube and NG tube for feeding due to post-op swelling c. not being able to eat solid foods d. permanence of the tracheostomy, referred to as a laryngectomy stoma

a

When blood passes through the lungs, what happens to oxygen? a. It diffuses from the alveoli into the red blood cells b. It diffuses from the red blood cells into the alveoli c. It decreases concentration in the blood. d. It increases concentration in the alveoli.

a

Which statement by the pt indicates understanding about radiation therapy for neck cancer? a. my voice will initially be hoarse, but should improve over time b. there are no side effects other than a hoarse voice c. dry mouth after radiation therapy is temporary and short term d. my throat is not directly affected by radiation

a

Which surgical procedure of the neck area poses no risk postoperatively for aspiration? a. total laryngectomy b. transoral cordectomy c. hemilaryngectomy d. partial laryngectomy

a

A pt comes to the walk-in clinic reporting seasonal nasal congestion, sneezing, rhinorrhea, and itchy, watery eyes. The nurse identifies that the pt most likely has rhinosinusitis and should also be assessed for sinusitis. Which manifestations does the nurse assess in a pt with rhinosinusitis? SELECT ALL THAT APPLY a. pain over the cheek radiating to the teeth b. tenderness to percussion over the sinuses c. generalized musculoskeletal achiness d. generalized facial pain when bending over e. referred pain to the temple or back of the head f. generalized swelling of the face and neck

a b d e

The pt with laryngeal cancer that is being treated with radiation is experiencing hoarseness. What teaching points must the nurse stress with this pt? SELECT ALL THAT APPLY a. your voice should improve within 4-6 weeks after the radiation therapy is completed b. typically the hoarseness becomes worse during the radiation therapy c. gargle 4-6x/day with an alcohol based mouthwash d. rest your voice and use alternative communication methods during radiation therapy e. wash your neck 3X/day with a strong antiseptic soap f. the speech therapist can offer options for alternative communication methods

a b d f

Which are warning signs of head and neck cancer? SELECT ALL THAT APPLY a. difficulty swallowing b. change in fit of dentures c. intermittent bilateral ear pain d. weight gain e. numbness in the mouth, lips or face f. lump in the mouth, neck or throat

a b e f

Packing has been removed from a pt with epistaxis. Which discharge instructions would the nurse be sure to teach the pt and his family? SELECT ALL THAT APPLY a. use saline spray to add moisture and prevent rebleeding b. use lots of petroleum jelly to coat the inside and outside of the nasal passages for comfort c. avoid vigorous nose blowing d. do not take aspirin containing products/NSAIDS e. no strenuous lifting for at least 1 month f. consume only small meal for 2 weeks

a c d e

Which factors can contribute to acute rhinosinusitis? SELECT ALL THAT APPLY a. viruses b. coughing c. irritants d. bacteria e. facial trauma f. antibiotic therapy

a c d e

A pt with facial trauma has undergone surgical intervention to wire the jaw shut. In performing discharge teaching with this pt, which topics does the nurse cover? SELECT ALL THAT APPLY a. oral care b. activity c. use of wire cutters d. communication e. aspiration prevention f. dental liquid diet

a c d e f

Following radiation therapy for head and neck cancer, the nurse instructs the pt about which potential side effects? SELECT ALL THAT APPLY a. skin redness and tenderness b. numbness of the mouth, lips, or face c. difficulty swallowing d. hoarseness e. dry mouth f. impaired taste

a c d e f

To reduce the spread of colds, which points must the nurse include when teaching pts? SELECT ALL THAT APPLY a. stay home from work, school, or other places where people gather b. seek medical attn at the first site of an oncoming cold c. cover both mouth and nose when coughing/sneezing d. dispose of used tissues properly e. thorough hand washing is essential f. avoid crowds of people

a c d e f

An adult pt is diagnosed with rhinosinusitis. What does the nurse instruct the pt to do? SELECT ALL THAT APPLY a. get plenty of rest, at least 8-10 hours/day b. keep fluid intake between 1000 and 1200 ml/day c. use a humidifier to help relieve congestion d. use nasal saline irrigation to safely relieve symptoms e. try sleeping with the head of your bed flat for better drainage f. limit exposure to any allergic causes

a c d f

Which are examples of a pandemic influenza? SELECT ALL THAT APPLY a. H1N1 (swine flu) b. seasonal flu c. Spanish influenza d. H5N1 (bird flu) e. viral influenza f. H7N9 (avian flu)

a c d f

A pt with rapid onset of severe headache, muscle aches, fever, chills, fatigue and weakness comes to the emergent care unit. On further assessment, he tells the nurse that additional symptoms include sore throat, cough and sneezing. What instructions should be given to the patient for his cough? SELECT ALL THAT APPLY a. be sure to wash your hands carefully whenever you cough/sneeze b. don't try to stop your sneezing because it will get worse c. cover your mouth with a tissue whenever you cough or sneeze d. be sure to perform oral hygiene at least 4x/day e. if you don't have a tissue, cough into your upper sleeve, not your hand f. be sure to dispose of used tissues immediately

a c e f

After several weeks of "not feeling well", a pt is seen in the HCP office for possible TB. If TB is present, which assessment finding does the nurse expect to observe? SELECT ALL THAT APPLY a. fatigue b. weight gain c. night sweats d. chest soreness e. low grade fever f. shortness of breath

a c e f

Which side effects would a pt with obstructive sleep apnea report? SELECT ALL THAT APPLY a. excessive daytime sleepiness b. excessive daytime hyperactivity c. inability to concentrate d. excessive production of sputum e. irritability f. heavy snoring

a c e f

Which med-surg concepts take priority when the nurse is caring for a patient with a non-infectious upper respiratory problem? SELECT ALL THAT APPLY a. gas exchange b. acid-base balance c. fluid and electrolyte balance d. cellular regulation e. tissue integrity f. elimination

a d

A pt is receiving enteral feedings and an NG tube is in place. In order to prevent aspiration, which precautions are used? SELECT ALL THAT APPLY a. no bolus feedings are given at night b. hold the feeding if the residual volume exceeds 20 ml c. vary the time of feedings according to pt's preference d. elevate the head of the bed during and after feedings e. evaluate the pt's tolerance of the feedings f. allow the pt to indicate when he/she is ready for the next bite

a d e f

A pt is seen in the HCP office and is diagnosed with community acquired pneumonia (CAP). What are the MOST common symptoms associate with CAP? SELECT ALL THAT APPLY a. dyspnea b. abdominal pain c. back pain d. chest discomfort e. increased sputum production f. fever

a d e f

Which statements about the Middle East respiratory syndrome (MERS) are accurate? SELECT ALL THAT APPLY a. MERS is caused by a virus that causes many respiratory illnesses including the common cold. b. The patient with MERS displays only respiratory symptoms such as cough and shortness of breath c. diagnostic tests for MERS include blood, urine and sputum for culture and sensitivity d. interventions for MERS can include IV fluids, mechanical ventilation and dialysis e. a pt being treated for MERS should be maintained on airborne, contact and reverse isolation f. "convalescent serum" may be given if the pt and convalescent person are the same blood type

a d f

Which diagnostic tests are most likely to be done for an older pt suspected of having pneumonia? SELECT ALL THAT APPLY a. sputum gram stain b. pulmonary function test c. fluorescein bronchoscopy d. peak flowmeter measurement e. chest x-ray f. CBC

a e f

15. Increased risk for oxygen toxicity is related to which factors? (Select all that apply.) a. Continuous delivery of oxygen at greater than 50% concentration b. Delivery of a high concentration of oxygen over 24 to 48 hours c. The severity and extent of lung disease d. Neglecting to monitor the patient's status and reducing oxygen concentration as soon as possible e. Excluding measures such as continuous positive airway pressure (CPAP) or posi- tive end-expiratory pressure (PEEP)

a, b, c, d, e a. Continuous delivery of oxygen at greater than 50% concentration b. Delivery of a high concentration of oxygen over 24 to 48 hours c. The severity and extent of lung disease d. Neglecting to monitor the patient's status and reducing oxygen concentration as soon as possible e. Excluding measures such as continuous positive airway pressure (CPAP) or posi- tive end-expiratory pressure (PEEP)

39. A patient with a tracheostomy or endotracheal tube has inline suctioning. Which nursing interventions apply to proper suctioning tech- nique? (Select all that apply.) a. Oxygenate the patient before suctioning. b. Instruct the patient that he or she is going to be suctioned. c. Suctioning time is the same for a tracheos- tomy and an endotracheal tube. d. The suction line is unlocked after suction- ing is completed. e. The suction tubing is locked after suction- ing is completed.

a, b, c, e a. Oxygenate the patient before suctioning. b. Instruct the patient that he or she is going to be suctioned. c. Suctioning time is the same for a tracheos- tomy and an endotracheal tube. . e. The suction tubing is locked after suction- ing is completed.

50. A patient with a tracheostomy who receives unnecessary suctioning can experience which complications? (Select all that apply.) a. Bronchospasm b. Mucosal damage c. Impaired gag reflex d. Bronchodilation e. Bleeding

a, b, e a. Bronchospasm b. Mucosal damage e. Bleeding

A patient has a recent tracheostomy. What necessary equipment does the nurse ensure is kept at the bedside? (Select all that apply.) a. Ambu bag b. Pair of wire cutters c. Oxygen tubing d. Suction equipment e. Tracheostomy tube with obturator

a, c, d, e a. Ambu bag c. Oxygen tubing d. Suction equipment e. Tracheostomy tube with obturator

19. Which factors are considered hazards associat- ed with oxygen therapy? (Select all that apply.) a. Increased combustion b. Oxygen narcosis c. Oxygen toxicity d. Absorption atelectasis e. Oxygen-induced hypoventila

a, c, d, e a. Increased combustion c. Oxygen toxicity d. Absorption atelectasis e. Oxygen-induced hypoventila

What are possible complications that can oc- cur with suctioning from an artificial airway? (Select all that apply.) a. Infection b. Coughing c. Hypoxia d. Tissue (mucosa) trauma e. Vagal stimulation f. Bronchospasm g. Cardiac dysrhythmias

a, c, d, e, f, g a. Infection c. Hypoxia d. Tissue (mucosa) trauma e. Vagal stimulation f. Bronchospasm g. Cardiac dysrhythmias

1. At what times is oxygen therapy needed for a patient? (Select all that apply.) a. To treat hypoxia b. To treat hypothermia c. To treat hypoxemia d. When the normal 35% oxygen level in the air is inadequate e. When the normal 21% oxygen level in the air is inadequate

a, c, e a. To treat hypoxia c. To treat hypoxemia e. When the normal 21% oxygen level in the air is inadequate

5. What are the hazards of administering oxygen therapy? (Select all that apply.) a. Oxygen supports and enhances combus- tion. b. Oxygen itself can burn. c. Each electrical outlet in the room must be covered if not in use. d. All electrical equipment in the room must be grounded. e. Solutions with high concentrations of alco- hol or oil cannot be used in the room.

a, d, e a. Oxygen supports and enhances combus- tion. d. All electrical equipment in the room must be grounded. e. Solutions with high concentrations of alco- hol or oil cannot be used in the room.

The nurse is performing a respiratory assessment on an older adult patient. Which questions are appropriate to ask when using Gordon's Functional Health Pattern Assessment approach? SATA a. How has your general health been? b. Do you now or have you ever smoked? c. Have you had any colds this past year? d. Do you have sufficient energy to do what you like to do? e. When was the last time you were hospitalized?

a,b,c,d

What observations does the nurse make when performing a general assessment of a pt's lungs and thorax? SATA a. Symmetry of chest movement b. Rate, rhythm, and depth of respirations c. Use of accessory muscles for breathing d. Comparison of anteroposterior diameter with lateral diameter e. Measurement of the length of chest cavity f. Assessment of chest expansion and respiratory excursion

a,b,c,d,f

The nurse is assessing a middle aged patient who reports a decreased tolerance for exercise and that she must work harder to breathe. Which questions assist the nurse in determining what these changes are related to? SATA a. Do you have anemia? b. When did you first notice these symptoms? c. Do you have or have you ever smoked cigarettes? d. How often do you exercise? e. Are you coughing up any colored sputum?

a,b,c,e

The nurse is rpovidng care for a pt who woudl like to quit smoking. Which important teaching points must be included when teacing this pt? SATA a. Talk with your hcp about nicotine replacement therapies b. Ask for help from family and friends who have quit smoking c. Smoking while using a nicotine patch is acceptable as long as you are gradually decreasing how much you smoke d. Remove all ashtrays, cigarettes, pipes, cigars, and lighters from your home to decrease the temptation to smoke e. If you are used to having a cigarette after eating, get up from the table as soon as you are finished eating f. Avoid starting an exercise program at the same time you quit smoking because making two big changes at the sametime is setting yourself up for failure

a,b,d,e

For what reasons would a pt have a bronchoscopy? SATA a. Obtain samples for cultures b. Diagnose pulmonary disease c. View upper airway structures d. Adminster medications e. Obtain samples for biopsy

a,b,e

What conditions shift th curve to the right, meaning hemoglobin will dissociate oxygen? SATA a. Increased carbon dioxide concentration b. Decreased tissue concentration of glucose breakdown products c. Increased tissue pH (alkalosis) d. Decreased tissue temperature e. Decreased tissue pH (acidosis)

a,b,e

A pt has undergon a percutaneous lung biopsy. After the procedure, what tests may be ordered to confirm that there is no pneumothorax? SATA a. Computed tomography b. Pulmonary function test c. magnetic resonance imaging d. Digital chest radiography e. Chest x-ray

a,e

The nurse has completed a community presentation about lung cancer. Which statement from a participant demonstrates and understanding of the information presented? a. "The primary prevention for reducing the risk of lung cancer is to stop smoking and avoid secondhand smoke." b. "The overall 5 year survival rate fro all patients with lung cancer is 85%." c. "The death rate for lung cancer is less than prostrate, breast, and colon cancer combined." d. "Cures are most likely for patients who undergo treatment for stage III disease."

a. "The primary prevention for reducing the risk of lung cancer is to stop smoking and avoid secondhand smoke."

The patient with COPD is undergoing pulmonary rehabilitation by walking. What does the nurse teach this patient about when to increase his walking time? a. "You should increase your walking time when your rest periods decrease." b. "you should increase your walking time when your heart rate remains less than 80/min." c. "You should increase your walking time when you do not need to use an inhaler."

a. "You should increase your walking time when your rest periods decrease."

Which are key elements for a personal asthma action plan? Select all that apply. a. A schedule for prescribed daily controller drug and directions for prescribed reliever drug b. A list of possible triggers for each asthma attack c. Patient-specific daily asthma control assessment questions. d. Directions for adjusting the daily controller drug schedule e. Emergency actions to take when asthma is not responding to controller and reliever drugs. f. When to contact the HCP (in addition to regularly scheduled visits)

a. A schedule for prescribed daily controller drug and directions for prescribed reliever drug c. Patient-specific daily asthma control assessment questions. d. Directions for adjusting the daily controller drug schedule e. Emergency actions to take when asthma is not responding to controller and reliever drugs. f. When to contact the HCP (in addition to regularly scheduled visits)

What are the goals for drug therapy in the treatment of asthma? Select all that apply. a. Drugs are used to stop an attack once it has started. b. Weekly drugs are used to reduce the asthma response. c. Combination drugs are avoided in the treatment of asthma. d. Some patients only require drug therapy during an asthma episode. e. Drugs are used to change airway responsiveness. f. Some drugs are used to decrease inflammation.

a. Drugs are used to stop an attack once it has started. d. Some patients only require drug therapy during an asthma episode. e. Drugs are used to change airway responsiveness. f. Some drugs are used to decrease inflammation.

A patient with a history of asthma enters the emergency department with severe dyspnea, accessory muscle involvement, neck vein distention, and severe inspiratory/expiratory wheezing. The nurse is prepared to assist the physician with which procedure if the patient does not respond to initial interventions. a. Emergency intubation b. Emergency needle thoracentesis c. Emergency chest tube insertion d. Emergency pleurodesis

a. Emergency intubation

A patient with asthma is repeatedly non-compliant with the medication regimen, which has resulted in the patient being hospitalized for a severe asthma attack. Which interventions does the nurse suggest to help the patient manage asthma on a daily basis? Select all that apply. a. Encourage active participation in the plan of care. b. Help the patient develop a flexible plan of care. c. Have the pharmacist establish a plan of care. d. Teach the patient about asthma and the treatment plan. e. Assess symptom severity using a peak flow meter 1-2 times a week. f. Educate the patient about implementation of his or her personal asthma action plan.

a. Encourage active participation in the plan of care. d. Teach the patient about asthma and the treatment plan. f. Educate the patient about implementation of his or her personal asthma action plan.

The nurse is taking a history from a patient with chronic cystic fibrosis. Which symptoms would the nurse expect? Select all that apply. a. Frequent respiratory infections b. Occasional respiratory congestion c. Decreased exercise tolerance d. ABGs that show respiratory alkalosis e. Increased sputum production f. Decreased carbon dioxide levels on ABGs

a. Frequent respiratory infections c. Decreased exercise tolerance e. Increased sputum production

The patient is receiving high-frequency chest wall oscillation (HFCWO). What are the actions of this therapy? Select all that apply. a. It dislodges mucous from the bronchial walls. b. It increases mobilization of mucous. c. It causes bronchodilation of the airways. d. It moves mucous upward toward the central airways. e. It decreases inflammation within the lung tissues. f. It thins secretions, making them easier to clear from the lungs.

a. It dislodges mucous from the bronchial walls. b. It increases mobilization of mucous. d. It moves mucous upward toward the central airways. f. It thins secretions, making them easier to clear from the lungs.

The nurse is caring for a patient who has cystic fibrosis. Which assessment findings indicate the need for exacerbation therapy? Select all that apply. a. New onset crackles b. Increased activity tolerance c. Increased frequency of coughing d. Increased chest congestion e. Increased SaO2. f. At least a 10% decrease in FEV1.

a. New onset crackles c. Increased frequency of coughing d. Increased chest congestion f. At least a 10% decrease in FEV1.

A patient has returned several times to the clinic for treatment of respiratory problems. Which action does the nurse perform first? a. Obtain a history of the patient's previous respiratory problems and response to therapy. b. Ask the patient to describe his compliance to the prescribed therapies. c. Obtain a request for diagnostic testing, including TB and HIV. d. Listen to the patient's lungs, obtain a pulse oximetry reading, and count the respiratory rate.

a. Obtain a history of the patient's previous respiratory problems and response to therapy.

In assisting a patient with COPD to relieve dyspnea, which sitting positions are beneficial to the patient for breathing? Select all that apply. a. On edge of chair, leaning forward with arms folded and resting on a small table. b. In a low semi-reclining position with the shoulders back and knees apart. c. Forward in a chair with feet spread apart and elbows placed on the knees. d. Head slightly flexed, with feet spread apart, and shoulders relaxed. e. Low semi-Fowler's with knees elevated. f. Side lying to facilitate diaphragm movement.

a. On edge of chair, leaning forward with arms folded and resting on a small table. c. Forward in a chair with feet spread apart and elbows placed on the knees. d. Head slightly flexed, with feet spread apart, and shoulders relaxed.

Which are the main purposes of asthma treatment? Select all that apply. a. Prevent asthma episodes b. Avoid secondhand smoke c. Improve airflow d. Relieve symptoms e. Improve exercise tolerance f. Control asthma episodes

a. Prevent asthma episodes c. Improve airflow d. Relieve symptoms f. Control asthma episodes

The nurse is caring for an older adult patient with a chronic respiratory disorder. Which interventions are best to use in caring for this patient? Select all that apply. a. Provide rest periods between activities such as bathing, meals, and ambulation. b. Place the patient in a supine position after meals to allow for rest. c. Schedule drug administration around routine activities to increase adherence to drug therapy. d. Arrange chairs in strategic locations to allow the patient to walk and rest. e. Teach the patient to avoid getting the pneumococcal vaccine. f. Encourage the patient to have an annual flu vaccination.

a. Provide rest periods between activities such as bathing, meals, and ambulation. c. Schedule drug administration around routine activities to increase adherence to drug therapy. d. Arrange chairs in strategic locations to allow the patient to walk and rest. f. Encourage the patient to have an annual flu vaccination.

A patient with a history of bronchitis for greater than 20 years is hospitalized. With this patient's history, what is a potential complication? a. Right-sided heart failure b. Left-sided heart failure c. Renal disease. d. Stroke

a. Right-sided heart failure

Patients with asthma are taught self-care activities and treatment modalities according to the "step method." Which symptoms and medication routines relate to Step 3? a. Symptoms occur daily; daily use of inhaled corticosteroid and a long-acting beta agonist. b. Symptoms occur more than once per week; daily use of anti-inflammatory inhaler. c. Symptoms occur less than once per week; use of rescue inhalers once per week. d. Frequent exacerbations with limited physical activity; increased use of rescue inhalers.

a. Symptoms occur daily; daily use of inhaled corticosteroid and a long-acting beta agonist

The nurse is caring for a patient with a chest tube. What is the correct nursing intervention for this patient? a. The patient is encouraged to cough and do deep-breathing exercises frequently b. "Stripping" of the chest tubes is done routinely to prevent obstruction by blood clots c. Water level in the suction chamber need not be monitored, just the collection chamber d. Drainage containers are positioned upright or on the bed next to the patient.

a. The patient is encouraged to cough and do deep-breathing exercises frequently

Which of the following are characteristics of chronic pulmonary emphysema? Select all that apply. a. Decreased surface area of alveoli b. chronic thickening of bronchial walls c. high arterial oxygen level d. hypercapnia e. ABGs show chronic respiratory acidosis f. increased eosinophils

a. decreased surface area of alveoli d. hypercapnia e. ABGs show chronic respiratory acidosis

29. A patient is receiving preoperative teaching for a partial laryngectomy and will have a trache- ostomy postoperatively. How does the nurse define a tracheostomy to the patient? a. Opening in the trachea that enables breathing b. Temporary procedure that will be reversed at a later date c. Technique using positive pressure to im- prove gas exchange d. Procedure that holds open the upper air- ways

aa. Opening in the trachea that enables breathing

A patient has a temporary tracheostomy fol- lowing surgery to the neck area to remove a benign tumor. Which nursing intervention is performed to prevent obstruction of the tra- cheostomy tube? a. Provide tracheal suctioning when there are noisy respirations. b. Provide oxygenation to maintain pulse ox- imeter readings. c. Inflate the cuff to maximum pressure and check it once per shift. d. Suction regularly and PRN with a Yankau- er suction.

aa. Provide tracheal suctioning when there are noisy respirations.

46. The nurse is caring for a patient with a tra- cheostomy who has recently been transferred from the ICU, but he has had no unusual oc- currences related to the tracheostomy or his oxygenation status. What does the routine care for this patient include? a. Thorough respiratory assessment at least every 2 hours b. Maintaining the cuff pressure between 50 and 100 mm Hg c. Suctioning as needed; maximum suction time of 20 seconds d. Changing the tracheostomy dressing once a day

aa. Thorough respiratory assessment at least every 2 hours

20. A patient is receiving warmed and humidified oxygen. In discarding the moisture formed by condensation, why does the nurse minimize the time that the tubing is disconnected? a. To prevent the patient from desaturating b. To reduce the patient's risk of infection c. To minimize the disturbance to the patient d. To facilitate overall time management

aa. To prevent the patient from desaturating

A parent calls the emergency department about her child who reports a severe sore throat and refuses to drink fluids or to take liquid pain meds. What is the MOST important question for the nurse to ask in order to determine the urgency of seeking immediate medical attention? a. does the child seem to be refusing fluids and meds because of the sore throat? b. is the child drooling or do you hear stridor, a raspy rough sound when the child breathes? c. when did the symptoms start and how long have you been encouraging fluids? d. is the throat red or do you see any white patches in the back of the throat?

b

A pt demonstrates labored, shallow respirations and a respiratory rate of 32/min with a pulse oximetry reading of 85%. What is the priority nursing intervention? a. Notify respiratory therapy to give the pt a breating treatment b. Start oxygen via nasal cannula a 2L/min c. Obtain an order for a stat arterial blood gas (ABG) d. Encourage coughing and deep breating exercises

b

A pt diagnosed with TB has been receiving treatment for 3 months and has clinically shown improvement. The family asks the nurse if the pt is still infectious. What is the nurse's BEST reply? a. the pt is still infectious until the entire treatment is completed b. the pt is likely not infectious but needs to continue treatment for at least 6 mos c. the pt is infectious until there is a negative chest xray d. the pt may/may not be infectious, a purified protein derivative test (PPD) must be done

b

A pt has been transferred from the ICU to the med-surg unit after a laryngectomy. What does the nurse suggest to encourage the pt to participate in self-care? a. changing the trach collar b. suctioning the mouth with an oral suction device c. checking the stoma with a flashlight d. observing the color of the reconstructive flap

b

A pt has had an inner maxillary fixation for a mandibular fracture. Which piece of equipment should be kept at the bedside at ALL times? a. waterpik b. wire cutters c. pair of hemostats d. emesis basin

b

A pt having respiratory difficulty has a pH of 7.48. What is the nurse's best interpretation of this value? a. Acidosis b. Alkalosis c. Chronic respiratory illness d. Shortness of breath

b

A pt is admitted for a posterior nosebleed. Posterior packing is in place and the pt is on O2 therapy, ABX, and opioid analgesics. What is the PRIORITY assessment? a. tolerance of packing or tubes b. gag and cough reflexes c. mouth breathing d. skin breakdown around the nares

b

A pt is admitted to the hospital with a diagnosis of TB. While providing medication teaching, the pt asks the nurse why she must give the drugs by directly observed therapy (DOT). What is the nurse's BEST response? a. DOT can be done by having any person other than the pt observe that the drugs are swallowed b. DOT is to assure that the drug regimen is followed and drug-resistant TB organisms do not occur c. DOT was developed because too many pts do not take their drugs as prescribed d. DOT is used only with homeless people who cannot be trusted to take the drugs as prescribed

b

A pt reports smoking a pack of cigarettes a day for 9 years. He then quit for 2 years and then smoked 2 packs a day for the last 30 years. What are the pack-years for this patiet? a. 39 years b. 69 years c. 19.5 years d. 41 years

b

A pt treated for pneumonia is being prepared for discharge by the nurse. The pt is capable of performing self care and is anxious to return to his job at the construction site. Which discharge instructions does the nurse give to this pt? a. you are not contagious to others, so you can return to work as soon as you like b. you will continue to feel tired and will fatigue easily for the next several weeks c. try to drink 4 quarts of water/day, especially if you are physically active d. you should be able to return to work full time in 2 weeks when your energy level returns

b

A pt with HIV is admitted to the hospital with a temp of 99.6F and reports of bloody sputum, night sweats, feeling of tiredness and SOB. What are theses assessment findings consistent with? a. pneumocystis jiroveci pneumonia (PJP) b. TB c. superinfection as a result of low CD4 count d. severe bronchitis

b

An older adult with dehydration has altered mental status and inspissated (thickly crusted) oral and nasopharyngeal secretions. What PRIORITY instruction would the nurse give the UAP when providing care for this pt? a. bathe the pt 2X/day b. provide comprehensive oral care every 2 hrs c. ambulate the pt in the hall every 4 hrs d. check vital signs, including temp, every 6 hrs

b

Before a bronchoscopy procedure, the pt received benzocaine spray as a topical anesthetic to numb the oropharynx. The nurse is assessing the pt after the procdure. Which finding suggests that the pt is developing methemoglobinemia? a. The pt has a decreased hematocrit level b. The pt does not respond to supplemental oxygen c. The blood sample is a bright cheery red color d. The pt. experiences sedation and amnesia

b

In which situation would the oxygen dissociation curve shift to the left? a. Decreased pH (acidosis) b. Increased pH (alkalosis) c. Increased body temperature d. Increased body carbon dioxide concentration

b

The nurse has identified the problem of ineffective airway clearance with bronchospasms for a pt with pneumonia. The pt has no previous history of chronic respiratory disorders. The nurse obtains an order for which nursing intervention? a. increased liters of humidified O2 via facemask b. scheduled and prn aerosol nebulizer bronchodilator treatments c. handheld bronchodilator inhaler as needed d. corticosteroid via inhaler or IV to reduce the inflammation

b

The nurse hears fine crackles during a lung assessment of the patient who is in the inital postoperative period. Which nursing intervention helps relieve this respiratory problem? a. Monitor the pt w/ a pulse oximeter b. Encourage coughing and deep-breathing c. Obtain an order for a chest x-ray d. Obtain an order for high-flow oxygen

b

The nurse is assessing a pt with significant and obvious facial trauma after being struck repeatedly in the face. Which finding is the PRIORITY and requires immediate intervention? a. asymmetry of the mandible b. restlessness with high-pitched respirations c. nonparallel extra-ocular movements d. pain upon palpation over nasal bridge

b

The nurse is caring for an older adult who uses a wheelchair and spends over half of each day in bed. Which intervention is important in promoting pulmonary hygiene related to age and decraesed mbiligy? a. Obtain an order for prn oxygen via nasal cannula b. Encourage the pt to turn, cough, and deep breathe c. Reassure the pt. that immobility is temporary d. Monitor the respiratory rate and check pulse oximetry readings

b

The nurse is giving discharge instructions to a pt diagnosed with a viral influenza. Which statement by the pt indicates the need for future teaching? a. I should try to rest, increase my fluid intake, and get a humidifier for the house b. I will wait for my test results; then I can get a prescription for ABX c. OTC analgesics like Tylenol or ibuprofen, can be used for pain d. I should gargle several times a day with warm salt water and use throat lozenges

b

The nurse is making home visits to an older adult recovering from a hip fracture and identifies the problem of risk for respiratory infection. Which condition represents a factor of normal aging that would contribute to this INCREASED risk? a. inability to force a cough b. decreased strength of respiratory muscles c. increased elastic recoil of alveoli d. increased macrophages in alveoli

b

The nurse is reviewing ABG results from an 86 y/o pt. Which results would be considered normal findings for a pt. of this age? a. Normal pH, normal PaO2, normal PaCO2 b. Normal pH, decreased PaO2, normal PaCO2 c. Decreased pH, decreased PaO2, normal PaCO2 d. Decreased pH, decreased PaO2, decreased PaCO2

b

The nurse is teaching a pt about post rhinoplasty care. Which pt statement indicates an understanding of the instruction? a. i will have a very large dressing on my nose b. I will have bruising around my eyes, nose and face c. there will swelling that will cause a loss of sense of smell d. my nose will be 3 times its normal size for 3 weeks

b

The nurse makes observations about several respiratory pts' abilities to perform ADLs in order to quantify the level of dyspnea. Which pt is considered to have class V dyspnea? a. Experiences subective shortness of breath when walking up a flight of stairs b. LImited to bed or chair and experiences shortness of breath at rest c. Can independently shower and dress by cannot keep pace with similarly aged people d. Experiences shortness of breath during aerobic exercise such as jogging

b

The pt with a nasal fracture has clear fluid draining from the nose that dries on a piece of filter paper and leaves a yellow "halo" ring on the dried edge of the fluid. What is the nurse's FIRST action? a. document the finding b. notify the HCP c. send a sample to the lab d. place the pt in a supine position

b

The respiratory therapist consults with and reports to the nurse that a pt is producing frothy pink sputum. What does the nurse suspect is occuring with this pt? a. Pneumothorax b. Pulmonary edema c. Pulmonary infection d. Pulmonary infarction

b

Upon performing a lung sound assessment of the anterior chest, the nurse hears moderately loud sounds on inspiration that are equal in length with expiration. In what area is the lung sound considered normal? a. trachea b. primary bronchi c. lung fields d. larynx

b

What condition INCREASES the risk for the pt to develop community acquired pneumonia? a. pt has received the pneumococcal vaccination b. pt uses tobacco and alcohol often and regularly c. pt lives alone and eats alone d. pt received influenza shot in Nov rather than Sept

b

What is the characteristc of normal lung sounds that should be heard throughout the lung fields? a. Short inspiration, long expiration, loud, harsh b. Soft sound, long inspiration, short quiet expiration c. Mixed sounds of harsh and soft, long inspiration and long expiration d. loud, long inspiration and short, loud expiration

b

Which description best explains residual volume (RV)? a. Amount of air in the lungs at the end of maximal inhalation b. Amount of air remaining in lungs at the end of full forced exhalation c. Amout of air remaining in the lungs after normal exhalation d. Maximal amount of forced air that can be exhaled after maximal inspiration

b

Which intervention does the nurse use to assist the pt who suffers from chronic xerostomia secondary to past radiation treatments? a. offer small, frequent meals b. suggest a moisturizing spray c. explain fluid restrictions d. teach to wash with mild soap and water

b

Which is an example of third-hand passive smoking? a. Sitting in a car with a person who is smoking b. Exposure to smoke on the clothes of a smoker c. Walking through a group of people smoking outside d. Entering a room where several people have been smoking

b

Which prescribed drug order for an older adult diagnosed with rhinosinusitis would the nurse clarify with the HCP? a. acetaminophen b. diphenhydramine c. montelukast d. cromolyn sodium

b

Which pt is at HIGHEST risk for developing HCA pneumonia? a. any hospitalized pt between the ages of 18 and 65 years b. 32-year-old trauma pt on a mechanical ventilator c. disabled 54-year-old with osteoporosis discharged to home d. any pt who has not received the vaccine for pneumonia

b

Which statement BEST describes pneumonia? a. an infection of just the "windpipe" because the lungs are "clear" of any problems b. a serious inflammation of the bronchioles, alveoli, and interstitial spaces from various causes c. only an infection of the lungs with mild to severe effects on breathing d. an inflammation resulting from lung damage caused by long-term smoking

b

Which test is the most accurate and rapid test for TB? a. chest xray b. nucleic acid amplification test (NAAT) c. tuberculin test (Mantoux test) d. sputum culture

b

63. A patient with a tracheostomy tube is currently alert and cooperative but seems to be coughing more frequently and producing more secre- tions than usual. The nurse determines that there is a need for suctioning. Which nursing intervention does the nurse use to prevent hy- poxia for this patient? a. Allow the patient to breathe room air prior to suctioning. b. Avoid prolonged suctioning time. c. Suction frequently when the patient is coughing. d. Use the largest available catheter.

b b. Avoid prolonged suctioning time

16. The patient is receiving humidified oxygen which places the patient at high risk for which problem? a. Injury related to the moisture in the tube b. Infection related to the condensation in the tubing c. Limited mobility related to reliance on equipment d. Impaired skin integrity related to the mask

b b. Infection related to the condensation in the tubing

A pt with COPD needs instruction in measures to prevent pneumonia. What information does the nurse include? SELECT ALL THAT APPLY a. avoid going outside b. clean all respiratory equipment you have at home c. avoid indoor pollutants such as dust and aerosol d. get plenty of rest and sleep daily e. limit alcoholic beverages to 4-5/week f. be sure to get pneumonia vaccinations

b c d f

Which statements about the precautions of caring for a hospitalized pt with TB are true? SELECT ALL THAT APPLY a. health care workers must wear a mask that covers the face and mouth b. negative air flow rooms are required for these pts c. health care workers must wear a N95 or high-effeiciency particulate air (HEPA) mask d. gown and gloves are included in appropriate barrier protection e. strict contact precautions must be maintained f. careful hand washing is required before and after providing pt care

b c d f

A pt that has active TB. Which drugs will the HCP order during the initial phase of treatment? SELECT ALL THAT APPLY a. bedaquiline fumarate b. isoniazid c. rifampin d. bacille calmette-guerin e. ethambutol f. pyrazinamide

b c e f

Pts who are at high risk for TB would be asked which questions upon assessment? SELECT ALL THAT APPLY a. what does your diet normally consist of? b. do you have an immune dysfunction/HIV? c. do you use alcohol or inject recreational drugs? d. where do you live in the US? e. do you work in a crowded area such as a prison/mental health facility f. have you ever had a bacille Calmette-Guerin (BCG) vaccine?

b c e f

Which signs and symptoms suggest that a pt's rhinosinusitis is bacterial? SELECT ALL THAT APPLY a. facial trauma b. purulent drainage c. headache d. fever e. drop in BP f. no response to decongestants

b d f

3. To improve a patient's oxygenation to a normal level, the amount of oxygen administered is based on which factors? (Select all that apply.) a. Symptom management only b. Pulse oximetry reading c. Respiratory assessment d. The patient's subjective complaints e. Arterial blood gas results

b, c, e b. Pulse oximetry reading c. Respiratory assessment e. Arterial blood gas results

6. A patient requires home oxygen therapy. When the home health nurse enters the patient's home for the initial visit, he observes several issues that are safety hazards related to the pa- tient's oxygen therapy. What hazards do these include? (Select all that apply.) a. Bottle of wine in the kitchen area b. Package of cigarettes on the coffee table c. Several decorative candles on the mantel- piece d. Grounded outlet with a green dot on the plate e. Electric fan with a frayed cord in the bath- room f. Computer with a three-pronged plug

b, c, e b. Package of cigarettes on the coffee table c. Several decorative candles on the mantel- piece e. Electric fan with a frayed cord in the bath- room

7. Before completing the morning assessment, the nurse concludes that a patient is experienc- ing inadequate oxygenation and tissue perfu- sion as a result of respiratory problems. Which assessment findings support the nurse's conclu- sion? (Select all that apply.) a. Inspiratory and expiratory effort is shallow, even, and quiet. b. Patient must take a breath after every third or fourth word. c. Skin is pale, pink, and dry. d. Patient appears strained and fatigued. e. Pulse of 95 beats/min, respiratory rate of 30/min. f. Patient does not want to eat.

b, d, e b. Patient must take a breath after every third or fourth word. d. Patient appears strained and fatigued. e. Pulse of 95 beats/min, respiratory rate of 30/min.

51. A patient with a tracheostomy tube is able to speak and is no longer on mechanical ventila- tion. Which type of tracheostomy tube does this patient have? (Select all that apply.) a. Cuffless tube b. Fenestrated tube with inner cannula re- moved and the red stopper locked in place c. Standard inflated cuffed tube d. Size #6 Shiley inflated cuffed with a Passy- Muir valve e. Size #6 Shiley deflated cuffed that is capped

b, e b. Fenestrated tube with inner cannula re- moved and the red stopper locked in place e. Size #6 Shiley deflated cuffed that is capped

The nurse is caring for a patient with chronic bronchitis and notes the following clinical findings: fatigue, dependent edema, distended neck veins, and cyanotic lips. These assessment findings are consistent with which disease process? a. COPD. b. Cor pulmonale c. Asthma d. Lung cancer

b. Cor pulmonale

The nurse is taking a history for a patient with chronic pulmonary disease. The patient reports often sleeping in a chair that allows his head to be elevated rather than going to bed. The patient's behavior is a strategy to deal with which condition? a. Paroxysmal nocturnal dyspnea b. Orthopnea c. Tachypnea d. Cheyne-Stokes

b. Orthopnea

A patient is experiencing an asthma attack and shows an increased respiratory effort. Which ABG value is more associated with the early phase of the attack? a. PaCO2 of 60 mmHg b. PaCO2 of 30 mmHg c. pH of 7.40 d. PaO2 of 98 mmHg

b. PaCO2 of 30 mmHg

A patient is receiving ipratropium and reports nausea, blurred vision, headache, and inability to sleep. What action does the nurse take? a. Administer a PRN medication for nausea and a mild PRN sedative b. Report these symptoms to the physician as signs of overdose. c. Obtain a physician's request for an ipratropium level. d. Tell the patient that these side effects are normal and not to worry.

b. Report these symptoms to the physician as signs of overdose.

Which statement is true about the relationship of smoking cessation to the pathophysiology of COPD? a. Smoking cessation completely reverses the damage to the lungs b. Smoking cessation slows the rate of the disease progression c. Smoking cessation is an important therapy for asthma, but not for COPD, d. Smoking cessation reverses the effects on the airways but not the lungs.

b. Smoking cessation slows the rate of the disease progression

A patient who has well-controlled asthma has what kind of airway changes? a. Chronic, leading to hyperplasia b. Temporary and reversible c. Acute loss of smooth muscle mass d. Permanent and irreversible

b. Temporary and reversible

A patient admitted for a respiratory workup has baseline pulmonary function tests. After treatment with a bronchodilator the FEV1 increases by 14%. How does the nurse best interpret this value? a. The patient has emphysema b. The patient has asthma c. The patient has chronic bronchitis d. The patient has acute bronchitis

b. The patient has asthma

The patient is diagnosed with early pulmonary fibrosis. Which finding indicates that the patient's disease is progressing? a. The patient is short of breath with exertion b. The patient is becoming increasingly more short of breath c. The patient is experiencing respiratory infections d. The patient is experiencing side effects from his or her drugs.

b. The patient is becoming increasingly more short of breath

The nurse teaches a patient with asthma to perform which intervention before exercising? a. Rest for at least an hour. b. Use the short-acting beta-adrenergic (SABA) medication. c. Dress in extra clothing during cold weather. d. Practice pursed lip breathing.

b. Use the short-acting beta-adrenergic (SABA) medication.

A patient sustained a serious crush injury to the neck and had a tracheostomy tube placed yesterday. As the nurse is performing trache- ostomy care, the patient suddenly sneezes very forcefully and the tracheostomy tube falls out onto the bed linens. What does the nurse do? a. Ventilate the patient with 100% oxygen and notify the physician. b. Quickly and gently replace the tube with a clean cannula kept at the bedside. c. Quickly rinse the tube with sterile solution and gently replace it. d. Give the patient oxygen; call for assistance and a new tracheostomy kit.

bb. Quickly and gently replace the tube with a clean cannula kept at the bedside.

34. To prevent accidental decannulation of a tra- cheostomy tube, what does the nurse do? a. Obtain an order for continuous upper ex- tremity restraints. b. Secure the tube in place using ties or fabric fasteners. c. Allow some flexibility in motion of the tube while coughing. d. Instruct the patient to hold the tube with a tissue while coughing.

bb. Secure the tube in place using ties or fabric fasteners.

9. The nurse is caring for several patients on a general medical-surgical unit. The nurse would question the necessity of oxygen therapy for a patient with which condition? a. Pulmonary edema with decreased arterial Po2 levels b. Valve replacement with increased cardiac output c. Anemia with a decreased hemoglobin and hematocrit d. Sustained fever with an increased meta- bolic demand

bb. Valve replacement with increased cardiac output

A 35-year-old male pt with no health problems state that he had a flu shot last year and asks if it is necessary to have it again this year. What is the BEST response by the nurse? a. No, because once you get a flu shot, it lasts for several years and is effective against many different viruses. b. yes, because the immunity against the virus wears off, increasing your chance of getting the flu c. yes, because the vaccine guards against a specific virus and reduces your chances of acquiring the flu and is only effective for 1 year d. no, flu shots are only for high risk pts and you are not considered to be high risk

c

A patient is admitted to the hospital for treatment of pneumonia. Which nursing assessment finding BEST indicates that the patient is responding to ABX? a. wheezing, O2 at 2L/min, RR 26, , no SOB/chills b. temp 99F, lung sounds clear, pulse ox on 2L/min at 98%, cough with yellow sputum c. cough, clear sputum, temp 99F, pulse ox at 96% on room air d. feeling tired, RR 28 on 2L/min of O2, audible breath sounds

c

A patient who received a bronchoscopy was NPO for several hours before the test. Now a few hours after the test, the pt is hungry and would like to eat a meal. What does the nurse do before allwoing the pt to eat? a. Order a meal b/c the pt is now alrety and oriented b. Check pulse oximetry to be sure oxygen saturation has returned to normal c. Check for a gag reflex before allwoing the pt to eat d. Assess for nausea from the medication given for the test

c

A pt arrives in the ED with a severe crush injury to the face with blood gurgling from the mouth and nose and obvious respiratory distress. The nurse prepares to assist the physician with which procedure to manage the airway? a. performing a needle thoracotomy b. inserting and endotracheal tube c. performing a tracheotomy d. inserting a nasal airway and giving O2

c

A pt has sustained a mandible fracture and the surgeon has explained that the repair will be made using a resorbable plate. The pt discloses to the nurse that he has not told the surgeon about his substance abuse and illicit drug dependence. What is the nurse's BEST response? a. why didn't you talk to the surgeon about this issue? b. you should tell the surgeon, but it's your choice c. it is important for your surgeon to know about this info d. you shouldn't be ashamed; your surgeon will still repair your fracture

c

A pt hospitalized for pneumonia has ineffective airway clearance related to fatigue, chest pain, excessive secretions, and muscle weakness. What nursing intervention helps to correct this problem? a. administer O2 to prevent hypoxemia and atelectasis b. push fluids to greater than 3000 ml/day to ensure adequate hydration c. administer bronchodilator therapy in a timely manner to decrease bronchospasms d. maintain semii-Fowler's position to facilitate breathing and prevent further fatigue

c

A pt is having radiation therapy to the neck and reports a sore throat and difficulty swallowing. Which statement by the nursing student indicates a correct understanding of symptoms relief for this pt? a. the pt should not swallow anything too cold or too hot b. I will give the pt a mouthwash with an alcohol base c. I will help the pt with a saline gargle d. the pt should be reassured that the sore throat is temporary

c

A pt is unable to speak following a cordectomy. Which action is delegated to the UAP to assist the pt in dealing with the communication issues? a. politely tell the pt not to communicate b. teach the pt how to use hand signals c. allow extra time to accomplish ADLs d. give step-by-step instructions during the ADLs and discourage 2 way communication

c

A pt reporting a "sore throat" also has a temp of 101.4F (38.5C), pus behind the tonsils, and swollen lymph nodes. This pt will most likely be treated for which type of bacterial infection? a. staphylococcus b. pneumococcus c. streptococcus d. epstein-barr virus

c

A pt reports experiencing mild fatigue and a dry, harsh cough. There is a possibility of exposure to inhalation anthrax 3-4 days ago, but the pt currently reports feeling much better. What does the nurse advise the pt to do? a. have a cbc to rule out the disease b. monitor for and immediately seek attention for respiratory symptoms c. consult a HCP for diagnostic testing and ABX therapy d. stay at home, rest, increase fluid intake, and avoid public places

c

A pt reports fatigue and shortness of breath when getting up to walk to the bathroom; hoever, the pulse oximetry reading is 99%. The nurse identifies a diagnosis of Activity intolerance. Which lab value is consistent with the pt's subjective symptoms? a. BUN of 15 mg/dL b. WBC of 8000/mm c. Hemoglobin of 9 g/dL d. Glucose 160 mg/dL

c

A pt returns from surgery following a rhinoplasty. The UAP places the pt in a supine position to encourage rest and sleep. Which actions should the nurse take FIRST? a. teach the pt how to use the bed controls to position herself b. explain the purpose of the semi-Fowler's position to the nursing assistant c. place the pt in a semi-Fowler's position and assess for aspiration d. post a notice at the head of the bed to remind personnel about positioning

c

A pt who had a thoracentesis is now experiencing the following clinical monifestations: rapid, shallow respirations, rapid heart rate, and pain on the affected side that is worse at the end of inhalation. What complcation does the nurse suspect this patient has developed? a. Hemoptysis b. Lung abscess c. Pneumothorax d. Lung cancer

c

A pt who presents with symptoms of influenza that started 24 hours ago is seen by the HCP. Which intervention does the nurse expect for this pt? a. prescription for ABX b. admission to an acute care facility c. an order for an antiviral agent such as oseltmivir d. instructions to rest and decrease fluid intake

c

A pt who was in a motor vehicle accident and sustained laryngeal trauma is being treated in the ED with humidified O2 and is being monitored every 15-30 min for respiratory distress. Which assessment finding indicates the URGENT need for further intervention? a. RR 24; PaO2 80-100; no difficulty with communication b. pulse ox 96%; anxious, fatigued, blood in sputum, abdominal breathing c. confused and disoriented, difficulty producing sounds, pulse ox 80% d. anxious, RR 30, talking rapidly about the accident, warm to touch

c

A pt's pulse oximetry reading is 89%. What is the nurses first priority action? a. Recheck the reading w/ a different oximeter b. Apply supplemental oxygen and recheck the oximeter reading in 15 min c. Assess the pt for respiratory distress and recheck the oximeter reading d. Place the pt is the recovery position and monitor frequently

c

After being discharged from the hospital, a pt is diagnosed with TB at the outpatient clinic. What is the correct procedure regarding public health policy in this case? a. contact the infection control nurse at the hospital because the hospital is responsible for follow-up of this case b. there are no regulations because the pt was diagnosed at the clinic and not during the hospitalization c. contact the public health nurse so that all individuals who have come in contact with the patient can be screened d. have the pt sign a waiver regarding the hospital and clinic's liability for treatment

c

The nurse observes that a pt is having difficulty swallowing and has initiated aspiration precautions. Which procedure does the nurse expect the HCP to order for this pt? a. chest x-ray of the neck and chest b. CT scan of the head and neck c. dynamic swallow study under fluoroscopy d. direct and indirect laryngoscopy

c

The nursing student is caring for an older adult pt who sustained a stroke, is confused and is having trouble swallowing. Which statement by the nursing student indicates an understanding of aspiration precautions for this pt? a. i will administer pills as whole tabs as they are easier to swallow b. if the pt coughs, I will discontinue feeding and contact the physician c. I will keep the head of the bed elevated during and after feeding d. I will encourage small amounts of fluids, such as water, tea or juice

c

Upon assessing the lungs, the nurse hears short, discrete popping sounds 'like hair being rolled between fingers near the ear" in the bilateral lower lobes. How is this assessment documented? a. Rhonchi b. Wheezes c. Fine crackles d. Course crackles

c

What is the best positon to assume for a thoracentesis? a. Side-lying, affected side exposed, head slightly raised b. Lying flat with arm on affected side across the chest c. Sitting up, leaning forward on the overbed table d. Prone position with arms above the head

c

What nursing intervention may help to prevent the complications of pneumonia for a surgical pt? a. monitoring chest x-rays and WBC counts for early signs of infection b. monitoring lung sounds every shift and encouraging fluids c. teaching coughing, deep-breathing exercises, and use of incentive spriometry d. encouraging hand hygiene among all caregivers, patients and visitors

c

Which pt is the least likely to be at risk for developing pneumonia? a. pt with a 5-year history of smoking b. renal transplant pt c. post-op pt with bedside commode d. post-op pt with a hip replacement

c

Which sounds in the smaller bronchioles and the alveoli indicate normal lung sounds? a. Harsh, hallow, and tubular blowing b. Nothing' normally no sounds are heard c. Soft, low rustling; like wind in the trees d. Flat, dull tones w/ a moderate pitch

c

Which symptom suggests the possibility of sinus cancer? a. intermittent nasal obstruction b. little to no nasal drainage c. bloody nasal discharge d. lymph node enlargement on opposite side from tumor

c

While percussing a pt's chest and lung fields, the nurse notes a high, loud, musical, drumlike sound similar to tapping a cheek that is puffed out with air. What is the nurse's priority action? a. Document this expected finding using words like "high" "loud" and "hollow" b. Immediately notify the provider because the pt has an airway obstruction c. Assess the pt for air hunger or pain at the end of inhalation and exhalation d. Palpate for crackling sensation underneath the skin or for localized tenderness

c

While playing football at school, a pt injured his nose, resulting in a possible simple fracture. The pt's parents call the nurse seeking advice. What does the nurse tell the parents to do? a. ask the school nurse to insert a nasal airway to ensure patency b. apply an ice pack and allow the pt to rest in a supine position c. seek medical attn within 24 hours to minimize further complications d. monitor the symptoms for 24 hrs and contact the physician if there is bleeding

c

14. A patient is receiving a high concentration of oxygen as a temporary emergency measure. Which nursing action is the most appropriate to prevent complications associated with high- flow oxygen? a. Auscultate the lungs every 4 hours for oxy- gen toxicity. b. Increase the oxygen if the Pao2 level is less than 93 mm Hg. c. Monitor the prescribed oxygen level and length of therapy. d. Decrease the oxygen if the patient's condi- tion does not respond.

c c. Monitor the prescribed oxygen level and length of therapy.

25. A patient with a facemask at 5 L/min is able to eat. Which nursing intervention is performed at mealtimes? a. Change the mask to a nasal cannula of 6 L/ min or more. b. Have the patient work around the face mask as best as possible. c. Obtain a physician order for a nasal can- nula at 5 L/min. d. Obtain a physician order to remove the mask at meals.

c c. Obtain a physician order for a nasal can- nula at 5 L/min.

66. 67. Which clinical finding in a patient with a re- cent tracheostomy is the most serious and re- quires immediate intervention? a. Increased cough and difficulty expectorat- ing secretions b. Food particles in the tracheal secretions c. Pulsating tracheostomy tube in synchrony with the heartbeat d. Set tidal volume on the ventilator not be- ing received by the patient

c c. Pulsating tracheostomy tube in synchrony with the heartbeat

32. A patient returns from the operating room af- ter having a tracheostomy. While assessing the patient, which observations made by the nurse warrant immediate notification of the physi- cian? a. Patient is alert but unable to speak and has difficulty communicating his needs. b. Small amount of bleeding present at the incision. c. Skin is puffy at the neck area with a crack- ling sensation. d. Respirations are audible and noisy with an increased respiratory rate.

c c. Skin is puffy at the neck area with a crack- ling sensation.

The nurse is teaching the pt and family about care of a peritonsillar abscess at home. For what symptoms does the nurse indicate the need for the pt to go to the emergency dept IMMEDIATELY? SELECT ALL THAT APPLY a. persistent cough b. hoarseness c. stridor d. drooling e. N/V f. fever

c d

A patient has been prescribed cromolyn sodium for the treatment of asthma. Which statement by the patient indicates a correct understanding of this drug? a. "It opens my airways and provides short-term relief." b. "It is the medication that should be used 30 mins before exercise." c. "It is not intended for use during acute episodes of asthma attacks." d. "It is a steroid medication, so there are severe side effects."

c. "It is not intended for use during acute episodes of asthma attacks."

A patient is admitted with asthma. Which assessment findings are most likely to indicate that the patient's asthma condition is deteriorating and progressing toward respiratory failure? a. Crackles, rhonchi, and productive cough with yellow sputum b. Tachypnea, thick and tenacious sputum, and hemoptysis c. Audible breath sounds, wheezing, and use of accessory muscles. d. Respiratory alkalosis; slow, shalllow respiratory rate.

c. Audible breath sounds, wheezing, and use of accessory muscles.

For a patient who is a non-smoker, which classic assessment finding is particularly important in diagnosing asthma? a. Cough b. Dyspnea c. Audible wheezing d. Tachypnea

c. Audible wheezing

A patient is fearful that she might develop lung cancer because her father and grandfather died of cancer. She seeks advice about how to modify lifestyle factors that contribute to cancer. How does the nurse advise this patient? a. Not to worry about air pollution unless there is hydrocarbon exposure. b. Quit her job if she has continuous exposure to lead or other heavy metals c. Avoid situations where she would be exposed to secondhand smoke d. Not to be concerned because there are no genetic factors associated with lung cancer

c. Avoid situations where she would be exposed to secondhand smoke

The nurse assesses a patient and finds a dusky appearance with bluish mucous membranes and production of lots of mucous secretions. What illness does the nurse suspect? a. Asthma b. Emphysema c. Chronic bronchitis d. Acute bronchitis

c. Chronic bronchitis

After the nurse has instructed a patient with COPD in the proper coughing technique, which action the next day by the patient indicates the need for additional teaching or intervention? a. Coughing upon rising in the morning. b. Coughing before meals. c. Coughing after meals. d. Coughing at bedtime.

c. Coughing after meals.

Which are the most common early symptoms of pulmonary arterial hypertension a. Shortness of breath and dizziness b. Hypotension and headache c. Dyspnea and fatigue d. Chest pain and orthopnea

c. Dyspnea and fatigue

The nurse is developing a teaching plan for a patient with chronic airflow limitation using the priority patient problem of insufficient knowledge related to energy conservation. What does the nurse advise the patient to avoid? a. Performing activities at a relaxed pace. b. Working on activities that require using arms at chest level or lower. c. Eating three large meals a day. d. Talking and performing activities separately.

c. Eating three large meals a day.

A patient presents to the walk-in clinic with extremely labored breathing and a history of asthma that is unresponsive to prescribed inhalers or medications. What is the first priority nursing action? a. Establish IV access to give emergency medications. b. Obtain the equipment and prepare the patient for intubation. c. Place the patient in high Fowler's position and start oxygen. d. Call 911 and report the patient has probable status asthmaticus.

c. Place the patient in high Fowler's position and start oxygen.

A patient with asthma has been prescribed a fluticasone inhaler. What is the purpose of this drug for the patient? a. Relaxes the smooth muscles of the airway. b. Acts as a bronchodilator in severe episodes. c. Reduces obstruction of airways by decreasing inflammation. d. Reduces the histamine effect of the triggering agent.

c. Reduces obstruction of airways by decreasing inflammation.

The physician's prescriptions indicate an increase in the suction to -20 cm for a patient with a chest tube. To implement this, the nurse performs which intervention? a. Increases the wall suction to the medium setting and observes gentle bubbling in the suction chamber. b. Adds water to the suction and drainage chambers to the level of -20 cm. c. Stops the suction, adds sterile water to the level of -20 cm in the water seal chamber, and resumes the wall suction d. Has the patient cough and deep breathe and monitors the level of fluctuation to achieve -20 cm.

c. Stops the suction, adds sterile water to the level of -20 cm in the water seal chamber, and resumes the wall suction

The nurse is caring for an older adult patient with a history of chronic asthma. Which problem related to aging can influence the care and treatment of this patient? a. Asthma usually resolves with age, so the condition is less severe in older adult patients. b. It is more difficult to teach older adult patients about asthma than to teach younger patients. c. With aging, the beta-adrenergic drugs do not work as effectively. d. Older adult patients have difficulty manipulating handheld inhalers.

c. With aging, the beta-adrenergic drugs do not work as effectively.

23. A patient is being discharged and requires home oxygen therapy with a reservoir-type nasal cannula. He asks the nurse, "Why can't I just take this nasal cannula that I have been using in the hospital?" What is the nurse's best response? a. "The doctor ordered the cannula, so your insurance company should cover the cost." b. "With the used cannula there is a risk of a hospital-acquired infection." c. "This special nasal cannula allows you to decrease the oxygen flow by 50%." d. "This nasal cannula is much better. It is more flexible and comfortable to wear."

cc. "This special nasal cannula allows you to decrease the oxygen flow by 50%."

he nurse is providing discharge instructions for a patient who must perform self-care of a tracheostomy. The patient has been cheer- ful and cooperative during the hospital stay and has demonstrated interest and capability in performing self-care. But now the patient begins crying and refuses to leave the hospital. What is the nurse's best response? a. "You have done so well with your self-care. I am sure that you will be okay." b. "Let me call your family. They can help you to get home and get settled." c. "You have been brave and cheerful, but there is something that is worrying you." d. "We'll delay this teaching until later. Let's choose a scarf for you to wear home."

cc. "You have been brave and cheerful, but there is something that is worrying you."

17. A patient is receiving warmed and humidified oxygen. The respiratory therapist informs the nurse that several other patients on other units have developed hospital-acquired infections and Pseudomonas aeruginosa has been identi- fied as the organism. What does the nurse do? a. Place the patient in respiratory isolation. b. Obtain an order for a sputum culture. c. Change the humidifier every 24 hours. d. Obtain an order to discontinue the hu- midifie

cc. Change the humidifier every 24 hours.

62. An older adult patient sustained a stroke sev- eral weeks ago and is having difficulty swallow- ing. To prevent aspiration during mealtimes, what does the nurse do? a. Hyperextend the head to allow food to en- ter the stomach and not the lungs . b. Give thin liquids after each bite of food to help "wash the food down." c. Encourage "dry swallowing" after each bite to clear residue from the throat. d. Maintain a low-Fowler's position during eating and for 2 hours afterwards.

cc. Encourage "dry swallowing" after each bite to clear residue from the throat.

24. A patient is receiving oxygen therapy through a non-rebreather mask. What is the correct nursing intervention? a. Maintain liter flow so that the reservoir bag is up to one-half full. b. Maintain 60% to 75% Fio2 at 6 to 11 L/min. c. Ensure that valves and rubber flaps are patent, functional, and not stuck. d. Assess for effectiveness and switch to par- tial rebreather for more precise Fio2.

cc. Ensure that valves and rubber flaps are patent, functional, and not stuck.

30. A patient returns from the operating room and the nurse assesses for subcutaneous emphy- sema which is a potential complication associ- ated with tracheostomy. How does the nurse assess for this complication? a. Checking the volume of the pilot balloon b. Listening for airflow through the tube c. Inspecting and palpating for air under the skin d. Assessing the tube for patency

cc. Inspecting and palpating for air under the skin

11. A patient with chronic obstructive pulmonary disease (COPD) is admitted to the hospital with oxygen-induced hypoventilation. What is the respiratory stimulus to breathe for this patient? a. High carbon dioxide (60 to 65 mm Hg) level in the blood that rose over time b. Low level of carbon dioxide concentration in the blood, as sensed by the chemorecep- tors in the brain c. Low level of oxygen concentration in the blood, as sensed by the peripheral chemo- receptors d. Oxygen narcosis which stimulates central chemoreceptors in the brain

cc. Low level of oxygen concentration in the blood, as sensed by the peripheral chemo- receptors

12. The nurse is administering oxygen to a patient who is hypoxic and has chronic high levels of carbon dioxide. Which oxygen therapy pre- vents a respiratory complication for this pa- tient? a. Fio2 higher than the usual 2 to 4 L/min per nasal cannula b. Venturi mask of 40% for the delivery of oxygen c. Lower concentration of oxygen (1 to 2 L/ min) per nasal cannula d. Variable Fio2 via partial rebreather mask

cc. Lower concentration of oxygen (1 to 2 L/ min) per nasal cannula

27. A patient has an inflated cuffed #8 Shiley tra- cheostomy and is on a ventilator. The trache- ostomy tube is pulsating in synchrony with the patient's heartbeat. What does the nurse do first? a. Apply direct pressure to the innominate artery at the stoma site. b. Prepare the patient for immediate surgical repair. c. Notify the physician of the pulsating tube. d. No immediate management is needed un- less there is bleeding.

cc. Notify the physician of the pulsating tube.

64. The nurse is suctioning the secretions from a patient's endotracheal tube. The patient dem- onstrates a vagal response by a drop in heart rate to 54 and a drop in blood pressure to 90/50. After stopping suctioning, what is the nurse's priority action? a. Allow the patient to rest for at least 10 minutes. b. Monitor the patient and call the Rapid Re- sponse Team. c. Oxygenate with 100% oxygen and monitor the patient. d. Administer atropine according to standing orders.

cc. Oxygenate with 100% oxygen and monitor the patient

33. A patient was intubated for acute respiratory failure, and there is an endotracheal tube in place. Which nursing intervention is not ap- propriate for this patient? a. Ensure that the oxygen is warmed and hu- midified. b. Suction the airway, then the mouth, and give oral care. c. Suction the airway with the oral suction equipment. d. Position the tubing so it does not pull on the airway.

cc. Suction the airway with the oral suction equipment.

10. When a patient is requiring oxygen therapy, what is important for the nurse to know? a. Patients require 1 to 10 L/min by nasal cannula in order for oxygen to be effective. b. Oxygen-induced hypoventilation is the priority when the Paco2 levels are un- known. d. The goal is the highest Fio2 possible for the particular device being use

cc. Why the patient is receiving oxygen, ex- pected outcomes, and complications.

staging

classifies clinical aspects of the cancer; determines the exact location of the cancer and its degree of metastasis at diagnosis. stage influences therapy,

ploidy

classifies tumor chromosomes as normal or abnormal; degree of aneuploidy usually increases with the degree of malignancy.

Serous Tranduate

clear discharge - little color - little cellularity

A pt has had surgery for cancer of the neck. Which behavior indicates that the pt understands how to perform self-care to prevent aspiration? a. chooses thin liquids that cause coughing but knows to take small sips b. eats small, frequent meals that include a variety of textures and nutrients c. asks for small, frequent sips of nutrition supplement as a bedtime snack d. positions self upright before eating/drinking anything

d

A pt in the ED with laryngeal trauma has developed shortness of breath with stridor and decreased O2 saturation. What is the PRIORITY action? a. insert an oral/nasal airway b. assess for tachypnea, anxiety and nasal flaring c. obtain equipment for a trach d. apply O2 and stay with the pt

d

A pt is experiencing acute anxiety related to hospitalization stress and an inability to accept changes related to laryngeal cancer. The pt wants to leave the hospital but agrees to try a med to "help me calm down". For which med does the nurse obtain a prn order? a. amitriptyline b. modafinil c. morphine sulfate d. lorazepam

d

A pt returns to the unit after bronchoscopy. In addition to respiratory status assessment, which assessment does the nurse make in order to prevent aspiration? a. presence of pain or soreness in the throat b. time and amt of last oral fluid intake c. type and location of chest pain d. presence or absence of gag reflex

d

A pt who had neck surgery for removal of a tumor reports "not being able to breathe very well." The nurse observes that the pt has decreased chest movement and an elevated pulse. A bronchoscopy is ordered. For what reason did the provider order a bronchoscopy for this pt? a. Reverse and relieve any obstruction caused during the neck surgery b. Assess the function of vocal cords or remove foreign bodies from the larynx c. Aspirate pleural fluid or air from the pleural space d. Visualize the larynx (airway structures) to use as a guide for intubation

d

A pt with a recent diagnosis of sinus cancer states that he wants another course of ABX because he believes he simply has another sinus infection. What is the nurse's BEST response? a. I'll call the physician for the antibiotic prescription b. why are you doubting your doctor's diagnosis? c. let me bring you some info about sinus cancer d. what did the dr. say to you about your condition?

d

After receiving the subcutaneous Mantoux skin test, a pt with no risk factors returns to the clinic in the required 48-72 hours for the test results. Which assessment finding indicates a positive result? a. test area is red, warm, and tender to touch b. there is induration or a hard nodule any size at the site c. induration/hardened area measures 5 mm or greater d. induration/hardened area measures 10 mm or greater

d

Based on the etiology and the main cause of heart failure, which pt has the greatest need for health promotion measures to prevent heart failure? A. pt with alzheimer's B. pt with cystitis C. pt with asthma D. pt with hypertension

d

The hcp has prescrive varenicline (Chantix) for the pt who wishes to quit smoking. What specific priority teaching must the nurse provide for the pt and his family? a. Avoid spending time in enclosed spaces with active smokers b. Make a list of all the reasons that you wish to quit smoking cigarettes c. Plan to reward yourself with the money you save from not smoking cigarettes d. Be sure to report any changes in behavior or thought processes to your hcp

d

The nurse is assessing a pt who reports being struck in the face and head several times. During the assessment, pink-tinged drainage from the nares is observed. Which nursing action provides relevant assessment data? a. have the pt gently blow the nose and observe for bloody mucous b. test the drainage with a reagent to check the pH c. ask the pt to describe the appearance of the face before the incident d. place a drop of the drainage on a filter paper and look for a yellow ring

d

The nurse is caring for a post-op pt who had a neck dissection. Which assessment finding is expected? a. bulky gauze dressing is present that is dry and intact over the site b. the pt can speak normally but reports a sore throat c. permanent gastrostomy tube is present with continuous tube feedings d. the pt has shoulder drop and limited range of motion

d

The nurse is caring for a pt with a nasal fracture. The pt has clear secretions that react positively when tested for glucose. Which complication does the nurse suspect? a. jaw fracture b. facial fracture c. vertebral fracture d. skull fracture

d

The nurse is caring for several pts who had diagnostic testing for respiratory disorders. Which diagnostic test has the highest risk for postprocedure complication of pneumothorax? a. Bronchoscopy b. Laryngoscopy c. Computed tomography of lungs d. Percutaneous lung biopsy

d

The nurse is caring for several pts who require treatment for laryngeal cancer. Which treatment/procedure requires pt education about aspiration precautions? a. total laryngectomy b. laser surgery c. radiation therapy d. supraglottic laryngectomy

d

The nurse is reviewing lab results for a pt who has pneumonia. Which lab value does the nurse expect to see for this pt? a. decreased Hgb b. increased RBCs c. decreased neutrophils d. increased WBCs

d

The nurse is reviewing the lab results for an older adult pt with pneumonia. Which lab value frequently seen in pts with pneumonia may NOT be seen in this pt? a. RBC 4.0-5.0 b. Hgb 12-16 g/dL c. Hct 36%-48% d. WBC 12,000-18,000 cell/microliter

d

The nursing student is preparing patient teaching materials about head and neck cancer. Which statement is accurate and included in the patient teaching info? a. it metastasizes often to the brain b. it usually develops over time c. it is often seen as red edematous areas d. it is often seen as white patchy mucosal lesions

d

Tissue oxygen delivey through dissociation from hemoglobin is based on which factor? a. saturation b. oxgen tension c. unloading from hemoglobin d. tissues' need for oxygen

d

Which assessment finding is an objective sign of chronic oxygen deprivation? a. Continuous cough productive of clear sputum b. Audible inspiratory and expiratory wheeze c. Chest pain that increases with deep inspiration d. Clubbing of fingernails and a barrel shaped chest

d

Which statement by the pt indicates correct understanding of drug therapy for rhinosinusitis? a. a side effect of my antihistamine drug can increase itching b. when i am feeling better I can stop taking my ABX c. I will take acetaminophen tablets every 4 hours to prevent fever d. my decongestant will decrease the swelling so I can breathe

d

13. A patient is at high risk or unknown risk for oxygen-induced hypoventilation. What must the nurse monitor for? a. Signs of nonproductive cough, chest pain, crackles, and hypoxemia b. Change of skin tone from pink to gray col- or after several minutes of oxygen therapy c. Signs and symptoms of hypoventilation rather than hypoxemia d. Changes in level of consciousness, apnea, and respiratory pattern

d d. Changes in level of consciousness, apnea, and respiratory pattern

The nurse is taking a medical history on a new patient who has come to the office for a checkup. the patient states that he was supposed to take a medication called montelukast, but that he never got the prescription filled. What is the best response by the nurse? a. "When were first diagnosed with a respiratory disorder?" b. "Why didn't you get the prescription filled?" c. "Tell me how you feel about your decision to not fill the prescription." d. "Are you having any problems with your asthma?"

d. "Are you having any problems with your asthma?"

A neighbor with asthma is experiencing a severe and prolonged asthma attack that is unresponsive to treatment with a SABA drug. What is the nurse's best action? a. Continue to administer the patient's SABA drug at 5 minute intervals. b. Call the patient's HCP. c. Apply the supplemental oxygen that is in the patient's home. d. Call 911 and get the patient to emergency care ASAP

d. Call 911 and get the patient to emergency care ASAP

What is the purpose of pulmonary function testing, especially airflow rates and lung volume measurements, when classifying COPD? a. Determines the oxygen liter flow rates required by the patient. b. Measures blood gas levels before bronchodilators are administered. c. Evaluates the movement of oxygenated blood from the lung to the heart. d. Distinguishes airway disease (obstructive) from interstitial lung disease (restrictive).

d. Distinguishes airway disease (obstructive) from interstitial lung disease (restrictive).

A patient has chronic bronchitis. The nurse plans interventions for inadequate oxygenation based on which set of clinical manifestations? a. Chronic cough, thin secretions, and chronic infection b. Respiratory alkalosis, decreased PaCO2, and increased PaO2. c. Areas of chest tenderness and sputum production (often with hemoptysis) d. Large amounts of thick secretions and repeated infections.

d. Large amounts of thick secretions and repeated infections.

A patient with cystic fibrosis is admitted to the med-surg unit for an elective surgery. Which infection control is best for this patient? a. It is best to put two patients with CF in the same room b. Standard Precautions including hand-washing are sufficient c. The patient is to be placed on contact isolation d. Measures that limit close contact between people with CF are needed.

d. Measures that limit close contact between people with CF are needed.

Drugs for the treatment of COPD are the same as those used for the management of asthma. Which additional class of drugs would the nurse expect to administer for a patient with COPD? a. Beta-blocker drugs b. Corticosteroids c. Xanthines d. Mucolytics

d. Mucolytics

The nurse is caring for a patient with a chest tube in place. Over the past hour the drainage from the tube was 110 mL. What is the nurse's best action? a. Gently "milk" the tubing to remove clots b. Check the chest tube system for leaks. c. Instruct the patient to cough and deep breathe d. Notify the surgeon immediately.

d. Notify the surgeon immediately.

What is the advantage of using aerosol route for administering short-acting beta2 agonists? a. Achieves a rapid and effective anti-inflammatory action. b. Reduces the risk for fungal infections. c. Increases patient compliance because it is easy to use. d. Provides rapid therapy with fewer systemic side effects.

d. Provides rapid therapy with fewer systemic side effects.

Upon observation of a chest tube setup, the nurse reports to the physician that there is a leak in the chest tube and system. How has the nurse identified this problem? a. Drainage in the collection chamber has decreased b. The bubbling in the suction chamber has suddenly increased c. Fluctuation in the water seal chamber has stopped d. There was onset of continuous vigorous bubbling in the water seal chamber

d. There was onset of continuous vigorous bubbling in the water seal chamber

Lymphocytosis

increased lymphocytes

dietary factors related to cancer

suspected to alter cancer risk, preservatives, contaminants, preparation methods, and additives, sweeteners may have cancer promoting effects.


Conjuntos de estudio relacionados

02 - الآمــرون بالصــــرف (Les Ordonnateurs)

View Set

Image Acq. Unit 4 Quality Factors Radiographic Distortion

View Set

ΒΙΟΛΟΓΙΑ -1ο Κεφάλαιο β( Κύτταρο : η μονάδα της ζωής)

View Set

AWS Certified Cloud Practitioner

View Set

Chapter 39- Vehicle Extrication and Special Rescue Q & A

View Set

Chapter 25 face and neck injuries

View Set

Practice Questions: Comprehensive Pharmacology

View Set